Old and New Inequalities Vol 2

Download as pdf or txt
Download as pdf or txt
You are on page 1of 146

2

Preface
”The last thing one knows when writing a book is what to put first.” Blaise Pascal
Mathematics has been called the science of tautology; that is to say,
mathematicians have been accused of spending their time proving that things are equal
to themselves. This statement is rather inaccurate on two counts. In the first place,
mathematics, although the language of science, is not a science. More likely it is a
creative art, as G. H. Hardy liked to consider it. Secondly, the fundamental results of
mathematics are often inequalities rather than equalities .
In the pages that follow, we present a large variety of problems involving such
inequalities, questions that became famous in (mathematical) competitions or journals
because of their beauty. The most important prerequisite for benefiting from this book
is the desire to master the craft of discovery and proof. The formal requirements are
quite modest. Anyone who knows basic inequalities such as the ones of Cauchy-
Schwarz , H¨older , Schur , Chebyshev or Bernoulli is well prepared for almost
everything to be found here. The student who is not that experienced will also be
exposed in the first part to a wide combination of moderate and easy problems, ideas,
techniques, and all the ingredients leading to a good preparation for mathematical
contests. Some of the problems we chose to discuss are known, but we have included
them here with new solutions which show the diversity of ideas pertaining to
inequalities. Nevertheless, the book develops many results which are rarely seen, and
even experienced readers are likely to find material that is challenging and
informative.
To solve a problem is a very human undertaking, and more than a little mystery
remains about how we best guide ourselves to the discovery of original solutions. Still,
as George P´olya and the others have taught us, there are principles of problem
solving. With practice and good coaching we can all improve our skills. Just like
singers, actors, or pianists, we have a path toward a deeper mastery of our craft.
The authors September 2008

About the authors


Vo Quoc Ba Can is a student at the ”Can Tho” University of Medicine and
Pharmacy. As a high-school student, he participated in many national contests
obtaining several prizes. Though at the moment he is not studying mathematics, his
activity in Inequalities has proved to be quite wide lately. Some of his problems were
published in specialized journals, but the biggest part of them became popular on the
wordwide known MathLinks forum. On the same theme, he (co)authored several
manuscripts, which were (unfortunately) published in Vietnamese.
Cosmin Pohoat ¸˘a is in present a high-school student at the ”Tudor Vianu” High
School in Bucharest, Romania. During his scholar activity he participated in many
(mathematical or not) olympiads and contests. Recently, he was awarded with a Gold
Medal at the Sharygin International Mathematical Olympiad, which took place in
Dubna, Russia from July 29 to August 1, 2008. In the past few years, he had many

3
important contributions in Euclidean Geometry, distinguishing himself in journals like
Forum Geometricorum, Crux Mathematicorum or the American Mathematical
Monthly. In Clark Kimberling’s Encyclopedia of Triangle Centers, a point appears
under his name (X3333 - ”The Pohoata Point”). His main mathematical interests
besides Euclidean Geometry are Graph Theory, Combinatorial Number Theory and, of
course, Inequalities. Beyond mathematics, his activities include computer science,
philosophy, music, football (soccer) and tennis.

Table of contents
1 Problems 8 2 Solutions 22

Chapter 1
Problems
1. Prove that for all positive real numbersa,b the following inequality holds
2a(a + b)3 + b 2(a2 + b2 ) ≤ 3(a2 + b2 ).
2. Consider real numbersa,b,c contained in the interval [1 , 1]. Prove that2 2 ≤ a + b + b
+ c + c + a ≤ 3.1 + c 1 + a 1 + b
3. Leta,b,c be three positive real numbers contained in the interval [0, 1]. Prove that
1 1
ab + 1 +bc + 1 +ca + 1
1
≤5
a + b + c.
4. Letx,y,z be positive real numbers such thatxyz = 1. Show that the following
inequality holds:
1
+ 1 +1 1
(y + 1)2 + z2 + 1 + (z + 1)2 + x2 + 1 ≤ 1. (x + 1)2 + y2 2
5. Leta,b,c be three positive real numbers satisyingabc = 8. Prove that
a − 2 b − 2 c − 2 ≤ 0. + 1 + + 1 +
a b c + 1
6. Leta, b, c be the sidelengths of an acute-angled triangle. Prove that
(a + b + c)(a2 + b2 + c2 )(a3 + b3 + c3 ) ≥ 4(a6 + b6 + c6 ).
7. Leta,b,c be positive real numbers such thatab + bc + ca = 1. Prove that
a b2 + c2 + bc + b c2 + a2 + ca + c a2 + b2 + ab ≥ 3. 8. Find the maximum value of (x3
+ 1)(y3 + 1),
for all real numbersx,y, satisfying the condition thatx + y = 1.
9. Leta,b,c be positive real numbers. Prove that
a +b +c ≥a + b +b + c + 1.
b c a b + c a + b
10. Ifx,y,z are positive real numbers, prove that the following inequality holds: (x + y
+ z)2 (yz + zx + xy)2 ≤ 3(y2 + yz + z2 )(z2 + zx + x2 )(x2 + xy + y2 ).
11. Leta, b, c be positive real numbers such that
a

4
+
b
+
c

ab
1 + 1 + 1
c.
Prove that2 a
+
b
+
c

a + b + c
3 +
abc.
12. Leta,b,c be positive real numbers such that
1 + 1 +1 1

b + c + 1 + c + a + 1 ≥ 1. a + b
Prove thata + b + c ≥ ab + bc + ca.
13. Leta,b,c be real numbers satisfyinga, b, c ≥ 1 anda + b + c = 2abc. Prove that
3

(
a
+
b
+
c
)
2

3 3 3
√ ab − 1 +√ bc − 1 +√ ca − 1.

14. Leta1 , a2 , . . . , an be positive real numbers satisfying the condition thata1 + a2 + . .


. + an = 1. Prove that
n a 1 < √ . 1 + a + . . . + a
j 2 j=1 1 j
15. Positive numbersα,β,x1 , x2 , . . . , xn (n ≥ 1) satisfy the conditionx1 + x2 + . . . + xn
= 1. Prove that
x
3 x3 x3 1 . αx +
1
1 + 2 + . . . + n ≥ βx αx + βx αx + βx n(α + β)
2 2 3 n 1
16. If three nonnegative real numbersa,b,c satisfy the condition
1 1 1
,
a2 + 1 +b2 + 1 +c2 + 1 = 2

5
prove that ab + bc + ca ≤ 3 .2 17. Leta,b,c be positive real numbers. Prove that
2 2 2
b c + 4(a − b)2 a
b
a +
+c +a ≥ a + b + c b + c.
18. Ifx,y,z are positive numbers satisfying the conditionxy + yz + zx = 1, show that
27 √ x + y +√ y + z +√ z + x 2 ≥ 6√ 3.4 (x + y)(y + z)(z + x) ≥

19. Leta,b,c be positive real numbers. Prove that


a +b +c ≥ 3 +(a − c)2
b c a ab + bc + ca.
20. Leta,b,c be nonnegative real numbers satisfyingab + bc + ca = 3. Prove that 1 1
1 + a2 (b + c 1 + 2 1 ) + 1 + b2 (c + a) + 1 + c2 (a + b) ≤ 3 abc.
21. Leta,b,c be positive real numbers such that 2a + b = 1. Prove that
5a3
+ 4b3 + 3c3
bc
≥ 4.
ca ab
22. i) Ifx,y andz are three real numbers, all different from 1, such thatxyz = 1, then
prove that
x2 y2 z2
(x − 1)2 + (y − 1)2 + (z − 1)2 ≥ 1. ii) Prove that equality is achieved for infinitely many
triples of rational numbersx,y andz.
23. Leta,b,c be positive real numbers. Prove that
a + b c 9
c(c + a)2 + a(a + b)2 ≥ 4(ab + bc + ca). b(b + c)2
24. Leta,b,c be nonnegative real numbers such thata + b + c = 1. Prove that
b − c)2 √ √ ≤ √
a + ( b + c 3.4 +
25. Leta,b,c be the sidelengths of a triangle. Prove that
3 a2a + 1 ≥ 2 a2 + (b + c)2.

cyc
a
3 + (b + c)3
cyc
26. Prove that for any real numbersa,b,c the following inequality holds
(b + c − a)2 (c + a − b)2 (a + b − c)2 ≥ (b2 + c2 − a2 )(c2 + a2 − b2 )(a2 + b2 − c2 ). 27.
Leta,b,c be the sidelengths of a given triangle. Prove that
(a + b)(b + c)(c + a) + (−a + b + c)(a − b + c)(a + b − c) ≥ 9abc.
28. Leta,b,c be positive real numbers. Prove that 1
+ 1
+ 1
a + 1 +
1 1 1
b + 1 + c + 1≥ 9 . ab c abc + 1

6
29. Leta,b,c be positive real numbers contained in the interval [0, 1]. Prove that
2a + 2b + 2c + abc ≤ 4. 1 + bc 1 + ca 1 + ab

30. Leta,b,c be nonnegative real numbersa,b,c satisfying


max {b + c − a, c + a − b, a + b − c} ≤ 1.
Prove that a2 + b2 + c2 ≤ 1 + 2abc.
31. Ifx,y,z are real numbers satisfyingxyz = −1, prove that
x
4 + y4 + z4 + 3(x + y + z) ≥y2 + z2 z2 + x2 x2 + y2
+ +
x y z .
32. Leta,b,c,d be positive real numbers satisfying the conditiona + b + c + d = abc +
bcd + cda + dab. Prove that
2a 2b 2c
a + b + c + d + 2d ≥ 8. + 1 + + 1 + + 1 +
a b c d + 1
33. Leta,b,c be nonnegative real numbers. Prove that a2 + 2bc+ b2 + 2ca+ c2 + 2ab b2 +
c2 c2 + a2 a2 + b2 ≥ 3.
34. Leta,b,c be positive real numbers. Prove that bc
c(c + a b(b + c) c + a a + b b + ab) + a(a + b) + ca ≥ a + b + c c. 35. Leta,b,c be positive
real numbers such thatab + bc + ca ≥ 3. Prove that
√b c 3a + √b + c + √c + a ≥ √2 .a + b
36. Letx,y,z,t be positive real numbers such that
1 1 1 1
.
x + 1 +y + 1 +z + 1 +t + 1 = 1
Prove that
1 1 1 1 1 1 1
min + 1 + 1z, + 1 + 1t, + 1 + 1x, + 1 + 1≤ 1x y y z z t t x y ≤ max + 1 + 1z, + 1 + 1t,
+ 1x, 1 + 1 + 1 .x y y z z t t x y
37. Leta1 , a2 , . . . , an be positive real numbers. Prove that
n n 3
☐n ☐n Tn+1 ☐ aTk ☐ ≥ ak ,j
k=1 j=1 k=1
whereTk = k(k+1) is thek-th triangular number.2
38. Leta,b,c,d be positive numbers. Prove that
3(a2 − ab + b2 )(c2 − cd + d2 ) ≥ (a2 c2 − abcd + b2 d2 ).
39. Leta,b,c be real numbers such thata + b + c = 1. Prove that
a b c ≤ 9. + 1 + + 1 +
a2 b2 c2 + 1 10
40. Letn be a positive integer, and letx andy be positive real numbers such thatxn + yn
= 1. Prove that
n 1 + x2k n 1 + y2k 1 4k 4k < (1 − x)(1 − y)
. k=1 1 + x 1 + y k=1
41. Leta,b,c be positive real numbers such thata + b + c + 1 = 4abc. Prove that
1 + 1 + 11 1 1
a
b c

7
≥ 3 ≥ √ab + √bc + √ca.
42. Leta,b ,c be nonnegative real numbers such thata+b+c = 3. Setx = √ a2 − a + 1,
y = √ b2 − b + 1 andz = √ c2 − c + 1. Prove that:xy+yz+zx ≥ 3 andx+y+z ≤ 2 +√ 7.
43. Letn ≥ 2 be a given integer. Determine
(a) the largest realcn such that
1 + 1 + . . . + 1 ≥ cn 1 + a 1 + a 1 + a
1 2 n
holds for any positive numbersa1 , a2 , . . . , an witha1 a2 . . . an = 1. (b) the largest
realdn such that
1 + 1 + . . . + 1 ≥ dn 1 + 2a 1 + 2a 1 + 2a holds for any positive numbersa , a , . . . , a
1 2 n 1 2 n
witha1 a2 . . . an = 1.
44. Leta,b,c be positive real numbers. Prove that
bc + ca + ab ≤ a + b + c a2 + bc b2 + ca c2 + ab b + c c + a a + b.
45. Real numbersa1 , a2 , . . . , an are given. For eachi (1 ≤ i ≤ n) define
di = max {aj | 1 ≤ j ≤ i} − min {aj | i ≤ j ≤ n}
and letd = max {di | 1 ≤ i ≤ n}.
(a) Prove that for any real numbersx1 ≤ x2 ≤ . . . ≤ xn , we have
max {|xd .i − ai | | 1 ≤ i ≤ n} ≥2
46. Let (b) Show that there are real numbersx1 ≤ x2 ≤ . . . ≤ xn such that we have
equality in (a).
a,b,c be nonzero positive numbers. Prove that
a2 b2 c2 4a2 + ab + 4b2 + 4b2 + bc + 4c2 + 4c2 + ca + 4a2 ≤ 1.
47. Leta,b,c be positive numbers such that 4abc = a + b + c + 1. Prove that b2 + c2 c2 +
a2 a2 + b2
a +b +c ≥ 2(ab + bc + ca).
48. Leta,b,c be positive real numbers. Prove that
a3 b3 c3 3. (a + b)3 + (b + c)3 + (c + a)3 ≥ 8
49. Leta,b,c,x,y,z be positive real numbers. Prove that
(a2 + x2 )(b2 + y2 )(c2 + z2 ) ≥ (ayz + bzx + cxy − xyz)2 .
50. Letx,y,z be positive real numbers. Prove that

y
+
z
+
√ √ 4 (x + y + z)z + x x + y
+ ≥ (y + z) (z + x) (x + y).x y z
51. Leta,b,c be nonnegative real numbers such thatabc = 4 anda, b, c > 1. Prove that4
(a − 1)(b − 1)(c − 1)3 a + b + c − 1 ≤√ 4 − 1 .3
52. Leta,b,c be positive real numbers satisfyingabc = 1. Prove that 1) + 1 1
c(b + c) + a(c + a) ≥ 3. b(a + b 2

8
53. Prove that for all positive real numbersa,b,c the following inequality holds: 1 1 + 1
1 1 1 c + a + a + b≥ ab + bc + ca + 2(a2 + b2 + c2 ). a + b + c b + c
54. Leta,b,c be the sidelengths of a triangle. Prove that

c
+
a

b

a
+ b − c √
√b + c − a + √
c + √ a −√b + √ a + √ b −√c ≤ 3. b + √ c −√a
55. Leta,b,c be the sidelengths of a triangle with perimeter 1. Prove that b c a 1 < √a +
b2 + √b + c2 + √c + a2 < 2.
56. Prove that for any positive real numbersa,b andc, we have that
b + c+ c + a+ a + b≥ 6 · a + b + c

3
a b c abc .
57. Leta,b,c be positive real numbers. Prove that
√ c 3a + √ b + √ ca + a2 ≥ √ 2. ab + b2 bc + c2
58. Leta1 ≤ a2 ≤ . . . ≤ an be positive real numbers such that
a
2 + a2 + . . . + a2

1 2 n , a1 + a2 + . . . + an = m,n = 1 n
where 1 ≥ m > 0. Prove that for alli satisfyingai ≤ m, we have
n − i ≥ n(m − ai )2 .
59. Letx,y,z be positive real numbers. Prove that
3 √x √y √z√3
≤ √x + y + z ·y + z +z + x +x + y .2
60. Leta,b,c be positive real numbers. Prove that

a b c
a
2
+ 2
bc
+

b
2
+ 2
ca

9
+

c
2
+ 2
ab

a + b + c √ ab + bc + ca.
61. Leta,b,c be distinct positive real numbers. Prove the following inequality: a2 b + a2
c + b2 a + b2 c + c2 a + c2 b − 6abc≥ 16abc (a + b + c)2. a2 + b2 + c2 − ab − bc − ca
62. Leta,b,c be nonzero positive real numbers. Prove that a3 + abc+ b3 + abc+ c3 + abc≥
a(b3 + c3 )+ b(c3 + a3 )+ c(a3 + b3) b + c c + a a + b a2 + bc b2 + ca c2 + ab .
63. Leta,b,c,d be real numbers with sum 0. Prove the inequality:
(ab + ac + ad + bc + bd + cd)2 + 12 ≥ 6(abc + abd + acd + bcd).
64. Leta,b,c be positive real numbers satisfyinga + b + c = 1. Prove that
1
− 2
2 2 2 8(a2 + b2 + c2 )2 + 1 − 2 + 1 − 2 ≥ (1 − a)(1 − b)(1 − c). a b c

65. Letx1 , x2 , . . . , xn be real numbers from the interval [0, 1] satisfying


x1 x2 . . . xn = (1 − x1 )2 (1 − x2 )2 . . . (1 − xn )2 .
Find the maximum value ofx1 x2 . . . xn .
66. Leta,b,c be three positive real numbers with sum 3. Prove that
1
+ 12 + 12 ≥ a2 + b2 + c2 .a2 b c
67. Leta, b, c be positive real numbers satisfyinga + b + c = 3. Prove that
1
2b + 1 +2c + 1 +2a + 1
a b c ≤ abc.
68. For any three positive numbersa,b,c, prove the inequality 1 1 1(1 + abc) a (1 + b) + b
(1 + c) + c (1 + a)≥ 3.
69. Leta, b, c, d be real numbers such thata2 + b2 + c2 + d2 = 1. Prove that 1 + 1 1 1
1

bc
+
1

cd
+
1

da
≤ 16 1 − ab 3. 70. Letx1 , x2 , . . . , xn be positive real numbers such thatx1 + x2 + . . . +

10
xn = 1. Prove thatn √ n
x
i

1≤ n2
√ n + 1.

i=1 i=1 1 + xi
71. Leta,b,c be positive real numbers. Prove that a
4 + b4 + c4 3abc 2
≥ 3(a2 + b2 + c2). ab + bc + ca + a + b + c
72. Leta,b,c be nonnegative real numbers, from which at least two are nonzero. Prove
that
3
3 a 9
b2 + c2 c2 + a2 a2 + b2
2
+
bc
2 2
+ 3 b + ca + 3 c + ab ≥ a + b +
√ abc
c.
73. Leta1 , a2 , . . . , a100 be nonnegative eral numbers such thata2 +a2 +. . .+a2 = 1.1 2
100 Prove that
a
2 12 . a + a2 a + . . . + a2
1 2 3
2 100 a 1 < 25
74. Leta, b, c be nonnegative real numbers, no two of which are zero. Prove that ab +
ac + 4bc+ bc + ba + 4ca+ ca + cb + 4ab≥ 4.b2 + c2 c2 + a2 a2 + b2
75. Leta, b, c be positive real numbers. Prove that
a + b2 + c3 b + c2 + a3 c + a2 + b3 9.ab + c2 + bc + a2 + ca + b2 ≥ 2
76. Leta,b andc be positive real numbers satisfyinga + b + c = 2. Prove that 2 + bc+ b2
+ ca+ c2 + ab≤ 2 + cyc b2 + c2.2 + cyc b + c ≤ a 1 a21 a
b + c c + a a + b
77. Real numbersai , bi (1 ≤ i ≤ n) satisfyn a2 =n b2 = 1 andn ai bi =i=1 i i=1 i i=1 0. Prove
that2 n 2n
ai + bi ≤ n.
i=1 i=1
78. Leta,b,c be positive real numbers satisfyinga + b + c = 1. Prove that √2
√ ab + √ bc + √ ca ≤ 2 . ab + bc bc + ca ca + ab

79. Leta,b,c be nonnegative real numbers satisfyinga2 + b2 + c2 = 1. Prove that


≤ √
2

11
.
1 +
bc
+
b + c1 ≤ a
1 + ca 1 + ab
80. Leta, b, c be positive real numbers such thata ≤ b ≤ c andabc = 1. Prove that
a + b2 + c3 ≥ 1 + 12 + 13 .a b c
81. Givenk + 1 positive real numbersx0 , . . . , xk and a positive integern, show that
k
(
x
σ
1

+
. . .
+
x
σ
k

)
−n ≤ k −n x −n
i ,
σ i=0
where the sum on the left is taken of thek + 1 distinctk-element subsets of {x0 , . . . , xk
}.
82. Leta,b,c be nonnegative real numbers, such that at least two are nonzero and which
satisfy the conditiona + b + c = 1. Prove that

b
c
4
√ a + √ b + 2c + √ c + 2a ≤ 27 √3 − 1 . a + 2b√ 2

83. For real numbersxi > 1, 1 ≤ i ≤ n,n ≥ 2, such that


x
2 n
i ≥ S = x , for alli = 1, 2, . . . , n
j xi − 1 j=1
find, with proof, supS. 84. Leta, b, c, d be positive real numbers such thata+b+c+d =
abc+bcd+cda+dab. Prove that
a
2
+ 1 +
b
2

12
2 2
+ 1 + c2 + 1 + d2 + 1 ≤ (a + b + c + d)2 .
85. Leta, b, c be the sidelengths of a triangle. Prove that
a2 b − 1 + b2 c − 1 + c2 a − 1 ≥ 0.c a b
86. Letx, y, z be positive real numbers such thatx2 + y2 + z2 ≥ 3. Prove that x5 − x2 y5
− y2 z5 − z2 x5 + y2 + z2 + y5 + z2 + x2 + z5 + x2 + y2 ≥ 0. 87. Leta, b, c be the
sidelengths of a given triangle. Prove that a + b + c + 3 ≥ 2 a + b + b + c + c + a .b c a b + c
c + a a + b
88. Letx,y,z be positive real numbers. Prove that
x + y+ y + z+ z + x− 4 x
y
+
z
+
y + z z x y z + x x + y ≥ 1 − 8xyz . (x + y)(y + z)(z + x) 89. Leta,b,c be real numbers
satisfyinga2 + b2 + c2 = 9. Prove that
3 · min {a, b, c} ≤ 1 + abc.
90. Letx1 , x2 , . . . , x3n be positive real numbers. Prove that
3n 1 + x2 3n n 2n ·k ≥ 1 + x1/n .
k
k=1 1 + xk k=1
91. Given an integern ≥ 2, find the largest constantC(n) for which the inequality
n
xi ≥ C(n) 2xi xj +√ xi xj
i=1 1≤j<i≤n
holds for all real numbersxi ∈ (0, 1) satisfying (1 − xi )(1 − xj ) ≥ 1 for 1 ≤ j <4 i ≤ n.
92. Leta, b, c be nonnegative real numbers, from which at least two are nonzero and
satisfying the conditionab + bc + ca = 1. Prove that
a3 + a + b3 + b + c3 + c ≥ 2√ a + b + c.
93. Leta, b, c , d be nonnegative real numbers such thata + b + c + d = 3. Prove that
ab(a + 2b + 3c) + bc(b + 2c + 3d) + cd(c + 2d + 3a) + da(d + 2a + 3b) ≤ 6√ 3.
94. Forn ∈ N,n ≥ 2, determine
n n
max (1 − xi ), forxi ∈ R+ , 1 ≤ i ≤ n, x2 = 1.i
i=1 i=1
95. For integersn ≥ 2 and reals > 0, show that
n ☐n 1 ☐n
(s + i) 1 .☐s + j ☐< (n + 1) s + k −2i=0 j=0 k=1
96. Let a, b, c be real numbers contained in the interval 0, 3 and also satisfying
the5
conditiona +b +c = 1. Determine the maximum value that the following expression can
reach:

13
P (a, b, c) = a3 + b3 + c3 + 3abc.
4
97. Leta, b, c, d be positive real numbers such thata ≥ b ≥ c ≥ d andabcd = 1.
Prove that 1 1 1 ≥ 3 . a3 + 1 + b3 + 1 + c3 + 1 abc + 1

98. Leta, b, c be nonnegative real numbers, from which at least are nonzero. Prove that
a2 (b + c)2 b2 (c + a)2 c2 (a + b)2
b2 + c2 +c2 + a2 +a2 + b2 ≥ 2(ab + bc + ca).
99. Leta,b,c be nonnegative real numbers, from which at least two are nonzero. Prove
that
(
a
+
b
+
c
)
2 a(b + c) b(c + a) c(a + b) (a + b + c)2 ≥ a2 + bc b2 + ca + c2 + ab a2 + b2 + c2. ab +
+ ≥
bc + ca
100. Leta,b,c be positive real numbers. Prove that
a b2 − ca c2 − ab2 − bc + 4b2 + 4c2 + a2 + 4c2 + 4a2 + b2 ≥ 0. 4a2 + 4b2 + c2
101. Leta, b, c be nonnegative real numbers, from which at least two are nonzero.
Prove that
a(b + c)+ b(c + a) c(a + b) a(b + c) + b(c + a) c(a + b) b2 + c2 c2 + a2 + a2 + b2 ≥ a2 + bc
b2 + ca + c2 + ab.
102. Letn be a positive integer. Find the minimum value of
2n+1 + (b − c)2n+1 + (c − a)2n+1 (a − b)

(a − b)(b − c)(c − a) for distinct real numbersa,b,c withbc + ca ≥ 1 + ab + c2 .


103. Leta, b, c be positive real numbers. Prove that
ab+c bc+a ca+b 3. (b + c)2 + (c + a)2 + (a + b)2 ≥ 4

104. Find the least real numberc such that ifn ≥ 1 anda1 , . . . , an > 0 then
n k n
≤ c ak .k
k=1 j=1 1/aj k=1
105. Leta, b, c be nonnegative real numbers satisfyinga + b + c = 1, and moreover
from which at least two are nonzero. Prove that
a 4b2 + c2 + b 4c2 + a2 + c 4a2 + b2 ≤ 3 .4
106. Leta, b be real numbers such thata + b = 0 and letx, y > 1 be some given
constants. Determine the minimum value of the following expression:
) = (
a
2 + 1)x (b2 + 1)y
f (a, b (a + b)2 .

14
107. It is given that real numbersx1 , x2 , . . . , xn (n > 2) satisfy
n
xi > 1, |xi | ≤ 1 (i = 1, 2, . . . , n).
i=1
Prove that there exists a positive integerk such thatk xi −n xi ≤ 1.i=1 i=k+1
108. Letx, y, z be real numbers with sum 0, which are contained in the interval [−1, 1].
Prove that
1 + x + y2 + 1 + y + z2 + 1 + z + x2 ≥ 3.
109. Letai be nonzero positive real numbers, for alli = 1, . . . , n, satisfying
a1 + 1 + . . . +1 .1 + a2 + . . . + an =a1 a2 an
Prove that
1
+
. . .
+ 1 ≤ 1 . n − 1 + a1
1 +
n − 1 + a2 n − 1 + an 110. Leta, b, c , d be real numbers contained in the interval (0,
k]. Prove that a4 + b4 + c4 + d4 (2k − a)4 + (2k − b)4 + (2k − c)4 + (2k − d)4 abcd≥ (2k
− a)(2k − b)(2k − c)(2k − d).
111. Leta, b, c be positive real numbers. Prove that
a + b c3
b + c + c + a ≥ √ 2 · ab + bc + ca. a + b a2 + b2 + c2
112. Leta, b, c be nonnegative real numbers, from which at least two are nonzero.
a3 b3 c3 3(a2 + b2 + c2 ).a2 + b2 + b2 + c2 + c2 + a2 ≥ 2
113. Leta, b, c, d be nonnegative reals. Prove that
ak 1, 4 , cyc a + b + c≥ min 1, 2k−1 3k
for any nonnegative real numberk.
114. Leta, b, c be positive real numbers such thatabc = 1. Prove that for all (strictly)
positivek’s we have
1 + 1 1
1 + b + ck + 1 + c + ak ≤ 1. 1 + a + bk
115. Leta, b, c be nonnegative real numbers, from which at least two are nonzero.
Prove that
√b ca + √b + c + √c + a ≤ 5√ a + b + c.a + b 4
116. Leta, b, c, d be nonnegative real numbers, from which at least two are nonzero,
and which also satisfy the condition thata + b + c + d = 1. Show that
√ b c da + √ b + c + √ c + d + √ d + a ≤ 3. a + b 2
117. Leta,b,c,d be positive real numbers, from which at least three are nonzero. Prove
that

a b c d
a + b + c + √ b + c + d + √ c + d + a + √ d + a + b ≤ 5√a + b + c + d. 4

15
118. Leta1 , a2 , . . . , an be real numbers satisfyinga2 + a2 + . . . + a2 = 1. Determine1 2
n the maximum value of the following expression:
min |ai − aj | .
i=j
119. Leta2 , a3 , . . . , an be positive real numbers ands = a2 + a3 + . . . + an . Show that
n
a1−1/k √ s.k < s + 2
k=2

Chapter 2
Solutions
Problem 1 Prove that for all positive real numbers a, b the following inequality holds
2a(a + b)3 + b 2(a2 + b2 ) ≤ 3(a2 + b2 ).
(Irish MO 2004)
First proof. By applying twice the AM-GM Inequality , we get
2
a
(
a
+
b
)
2 2 2 2 2 2
3 ≤ 2a(a + b) + (a + b) 2b + a + b 2 , and b 2(a + b ) ≤2 .
Therefore
2a(a + b)3 + b 2(a2 + b2 ) ≤ 2a2 + 2b2 + 2ab ≤ 3(a2 + b2 ).
Second proof. According to the Cauchy-Schwarz Inequality , we have that
2 √ 2 · a2 + b2 2 2a(a + b)3 + b 2(a2 + b2 ) = 2a(a + b) · (a + b) + 2b ≤ 2a(a + b) + 2b2 (a

+ b)2 + a2 + b2 = 4(a2 + ab + b2 )2 ,
and thus
2a(a + b)3 + b 2(a2 + b2 ) ≤ 2a2 + 2b2 + 2ab ≤ 3(a2 + b2 ).
22
Problem 2 Consider real numbers a, b, c contained in the interval [1 , 1]. Prove that 2 2
≤ a + b + b + c + c + a ≤ 3.1 + c 1 + a 1 + b
(Romanian MO 2006)
Proof. We begin by proving the left hand side of the inequality. Sincea, b ≥ 1 , we
have2 a + b ≥ 1, and thusa + b ≥ a + b
1 + c a + b + c.
By adding the other two similar relations to the inequality from above, we obtain
2 = (
a + b) + (b + c) + (c + a)≤ a + b + b + c + c + a a + b + c 1 + c 1 + a 1 + b. For the

16
second inequality, note that the considered expression can be written as
a + c .

cyc 1 + c 1 + a
Asa, c ≤ 1, we have a ≤ aand c ≤ c
1 + c a + c 1 + a c + a,
and so a + c ≤ a + c = 1. 1 + c 1 + a a + c c + a

The other two cyclic relations occur (again) similarly. Summing up the all three we get
the desired result.
Problem 3 Let a, b, c be three positive real numbers contained in the interval [0, 1].
Prove that 1 1
ab + 1 +bc + 1 +ca + 1
1 ≤5
a + b + c.
(Chendi Huang)
Proof. Note that (ab + 1) − (a + b) = (1 − a)(1 − b) ≥ 0, and similarly bc + 1 ≥ b + c
and ca + 1 ≥ c + a. We thus have
a
(a + b + c) + 1 + + 1 +
ab bc ca + 1 ca ab
1 1 1 ≤ + 1 + b + 1 + c + 1 + 1 + 1 + 1
bc
a
≤ b + c + 3 + 1 +
bc ca + b ab + c
=
a
1
bc + 1 − c − b + 5 ca + b ab + c ≤ 1 − c − b + 5c + b b + c
= 5.
Problem 4 Let x, y, z be positive real numbers such that xyz = 1. Show that the
following inequality holds:
1 + 1 +1 1

(y + 1)2 + z2 + 1 + (z + 1)2 + x2 + 1 ≤ 1. (x + 1)2 + y2 2 (Cristinel Mortici, Math.


Reflections)
Proof. (by Bin Zhao) We see that
1 + 1 =1 1

2 + x2 + y2 + 2x ≤ 2(1 + x + xy). (x + 1)2 + y2


and sincexyz = 1, consider the substitutions
b c a
x = a, y = b, z = c.

It follows that1 ) = a
2(a + b + c), 2(1 + x + xy
and adding the other two similar inequalities to the one from above yields
1 + 1 +1 1

17
(y + 1)2 + z2 + 1 + (z + 1)2 + x2 + 1 ≤ 1. (x + 1)2 + y2 2
Problem 5 Let a, b, c be three positive real numbers satisying abc = 8. Prove that
a − 2 b − 2 c − 2 ≤ 0. + 1 + + 1 +
a b c + 1
(Romanian jBMO Team Preparation Tests 2008)
Proof. The inequality in question rewrites as
1 1
1 ≤ 1 . + 1 + + 1 +
a b c + 1
Consider the substitutions
x y z
a = 2y, b = 2z, c = 2x.
Thus, by the Cauchy-Schwarz Inequality we get
1 1 1 = 2xy + y2 + 2yz + z2 + 2zx + x2 a + 1 + b + 1 +y2 z2 x2 c + 1
(
x
+
y
+
z )
2
= 2
x + y2 + z2 + 2xy + 2yz + 2zx = 1.
Problem 6 Let a, b, c be the sidelengths of an acute-angled triangle. Prove that (a + b
+ c)(a2 + b2 + c2 )(a3 + b3 + c3 ) ≥ 4(a6 + b6 + c6 ).
(Vietnamese IMO Team Preparation Tests 2000)
Proof. By the Cauchy-Schwarz Inequality , we have
(a + b + c)(a3 + b3 + c3 ) ≥ (a2 + b2 + c2 )2 ,
and therefore it is enough to prove that
(a + b2 + c2 )3 ≥ 4(a6 + b6 + c6 ).
2

Set x = a2 , y = b2 , z = c2 . Sincea, b, c are the sidelengths of an acute-angled


triangle, (it is well-known and easy to prove that) the numbersx, y, z are theirselves the
sidelengths of a triangle. The inequality to prove now becomes
(x + y + z)3 ≥ 4(x3 + y3 + z3 ).
According to the Triangle Inequality, we proceed as follows:
( x + y + z)3 = x3 + y3 + z3 + 3x2 (y + z) + 3y2 (z + x) + 3z2 (x + y) + 6xyz ≥ x3 +
y3 + z3 + 3x2 (y + z) + 3y2 (z + x) + 3z2 (x + y) ≥ x3 + y3 + z3 + 3x2 · x + 3y2 · y + 3z2
· z
= 4(x3 + y3 + z3 ).
Note that the equality holds if and only if the triangle is degenerated, having
sidelengths of the type (0, t, t), wheret is a positive real number.
Problem 7 Let a, b, c be positive real numbers such that ab + bc + ca = 1. Prove that
a b2 + c2 + bc + b c2 + a2 + ca + c a2 + b2 + ab ≥ 3.
(Jose Luis Diaz-Barrero, College Math. Journal)
First proof. Since (b−c)2 is always a nonnegative real number, by the AM-GM

18
Inequality we get
b2 + bc + c2 3 = 34(b + c)2 + 14(b − c)2 ≥ 4(b + c)2 .
Hence
= a b2 + c2 + bc + b c2 + a2 + ca + c a2 + b2 + ab
√3 √3

√3 a(b + c) + b(c + a) + c(a + b)
2 2 2
√ 3(ab + bc + ca) = √ 3.
Second proof. We see that the inequality in question can be rewritten as
≥ √
3 , b
+
bc
·
f
a
+ ca · f b ab · f c
c a
where f (x) = √ x2 + x + 1. Sincef (x) is convex on the real line andab+bc+ca = 1
with ab,bc, andca positive real numbers then, according to the Jensen Inequality , it
follows that
ab
·
f
c + bc · f a + ca · f b ≥ f ab ·c + bc ·a + ca ·b

b c a b c a = f (ac + ba + cb) = f (1)



= 3.
Problem 8 Find the maximum value of
(x3 + 1)(y3 + 1),
for all real numbers x, y, satisfying the condition that x + y = 1. (Romanian MO 2006)
Proof. Putxy = t; asx + y = 1 we get (x3 + 1)(y3 + 1) = t3 − 3t + 2. Fromx + y = 1, by
the AM-GM Inequality ,
x + y 2
t = xy ≤ = 1.2 4
It is easy to prove that t3 − 3t + 2 ≤ 4 fort ≤ 1/4, with equality if and only ift = −1.
We infer that (x3 +1)(y3 +1) ≤ 4 forx, y ∈ R withx+y = 1 and (ψ3 +1)(−1/ψ3 +1) = 4,
whereψ is one of the roots ofz2 − z − 1 = 0.
Remarks. In fact, for any positive real numbersx andy, the following inequality holds:
x3 + (x + y)3 y3 + (x + y)3 ≤ 4(x + y)6 ,
with equality if and only ifx2 + 3xy + y2 = 0.
Problem 9 Let a, b, c be positive real numbers. Prove that
a b c a + b b + c
+ + ≥ + + 1.b c a b + c a + b

19
(Bielorussian MO 1998)
Proof. Note that the inequality is equivalent to
(a + b + c) a +b +c − 3 ≥ (a + b + c) a + b +b + c − 2 ,b c a b + c a + b
and thus it rewrites as
a2 b2 c2 ab+ bc + ca − 2(a + b + c) ≥ (a + b + c)(a − c)2 b+ c + a + c a b (a + b)(b + c) . Now
since
a2 b2 c2 (a − b)2 + (b − c)2 + (c − a)2 b +c +a − (a + b + c) =b c a ab+ bc + ca − (a + b + c) =
a2 (b − c)2 + b2 (c − a)2 + c2 (a − b)2 c a b 2abc≥ 0, it suffices to prove that (a − b)2 + (b
− c)2 + (c − a)2 (a + b + c)(a − c)2 b c a≥ (a + b)(b + c) . By the Cauchy-Schwarz
Inequality , we get
2 + (b − c)2 (a − c)2 (a − b)

b c≥ b + c and thus, it remains to show that 1 + 1 ≥ a + b + c b + ca (a + b)(b + c) Of


course, this is obvious since it is equivalent with
b(a + b + c) ≥ 0.
a(a + b)(b + c)
Problem 10 If x, y, z are positive real numbers, prove that the following inequality
holds: (x + y + z)2 (yz + zx + xy)2 ≤ 3(y2 + yz + z2 )(z2 + zx + x2 )(x2 + xy + y2 ).
(Indian MO 2007)
Proof. By the AM-GM Inequality , we have that
y2 + yz + z2 3 = 34(y + z)2 + 14(y − z)2 ≥ 4(y + z)2 ,
hence it suffices to show that
81 2 (z + x)2 (x + y)2 ≥ (x + y + z)2 (yz + zx + xy)2 .64(y + z)
This rewrites as
8
(x + y)(y + z)(z + x) ≥ 9(x + y + z)(xy + yz + zx), which is rather well-known, but for
sake of completeness, we record that it can be proved as follows (by making use of the
same AM-GM Inequality ):
(x + y)(y + z)(z + x) = (x + y + z)(xy + yz + zx) − xyz
1
≥ (x + y + z)(xy + yz + zx) − 9(x + y + z)(xy + yz + zx)
8
= 9(x + y + z)(xy + yz + zx)
Remarks. Note that that the inequality in question is valid for any real numbersx,y,z
(and not necessarily for them to be all nonnegative).
Problem 11 Let a, b, c be positive real numbers such that
a
+
b
+
c

ab
1 + 1 + 1
c.

20
Prove that 2
a
+
b
+
c

a + b + c
3 + abc.

(Peruvian IMO Team Selection Tests 2007)


Proof. By the Cauchy-Schwarz Inequality , we have
a
+
b
+
c

a b c
1 + 1 + 1 9

a + b + c, and thus
a + b + c ≥ 3.
Returning to the inequality in question, we see that it can be rewritten as
(a + b + c)2 ≥ 3 + 21 + 1 + 1 .ab bc ca
By the AM-GM Inequality , we get
1
+ 1 + 1 ≤ 1 1 + 1 + 12 1 2 ,ab bc ca 3 a ≤
b c 3(a + b + c)
and so it is suffice to show that
(a + b + c)2 ≥ 3 + 23(a + b + c)2 ,
which is obviously true according to the fact thata + b + c ≥ 3.
Problem 12 Let a, b, c be positive real numbers such that
1 + 1 +1 1

b + c + 1 + c + a + 1 ≥ 1. a + b
Prove that a + b + c ≥ ab + bc + ca.
(Romanian jBMO Team Selection Tests 2007)
Proof. According to the Cauchy-Schwarz Inequality , we get
(a + b + 1)(a + b + c2 ) ≥ (a + b + c)2 ,
which easily rewrites as
1 ≤ c2 + a + b . a + b + 1 (a + b + c)2 By summing cyclically, we obtain
1 + 1 +1 1

b + c + 1 + c + a + 1 ≤a2 + b2 + c2 + 2(a + b + c). a + b (a + b + c)2 But since1 + 1 + 1 1


b + c + 1 + c + a + 1 ≥ 1, a + b
we can see that
a2 + b2 + c2 + 2(a + b + c) ≥ (a + b + c)2 , and thereforea + b + c ≥ ab + bc + ca.

21
Problem 13 Let a, b, c be real numbers satisfying a, b, c ≥ 1 and a + b + c = 2abc.
Prove that √ √
3

(
a
+
b
+
c
)
2

3

ab

1 +
3

bc

1 +
3

ca − 1.
(Bruno de Lima Holanda, Math. Reflections)
Proof. The given inequality is equivalent to
√ √ √
3 3 4a2 b2 c2 ≥3 ab − 1 +3 bc − 1 +√ ca − 1,

which can be rewritten as



3 1 1 14 ≥ 3 1 − a2 b2 c2 + 3 1 − a2 b2 c2 + 1 − a2 b2 c2. abc2 a2 bc ab2 c

Consider the substitutionsx =1 ,y =1 ,z =1 ; then we havex, y, z < 1 andab bc ca


x + y + z = 2. Thus, we are left to prove that

3

4

3 3 3
√ yz − xyz +√ zx − xyz +√ xy − xyz,

or equivalently,

3 4 ≥3 yz(1 − x) +3 zx(1 − y) +3 xy(1 − z).
For this, we proceed as follows, by making use of the H¨older Inequality :
3
3 yz(1 − x) ≤ y z (1 − x) = 4.

cyc cyc cyc cyc

22
Problem 14 Let a1 , a2 , . . . , an be positive real numbers satisfying the condition that a
+ a2 + . . . + an = 1. Prove that
n
aj 1
< √2 .
j=1 1 + a1 + . . . + aj
(Romanian IMO Team Preparation Tests 2008)
Proof. According to the Cauchy-Schwarz Inequality , we have that
n
a
j = 1 + a + . . . + a
1 jj=1 1 + a1 + . . . + a
n √aj ·√
aj
j j=1
☐☐n ☐ ≤ ☐
☐ n aj ☐☐ aj ☐
j
=1
(1 +
a
1
+
. . .
+
a
j
)
2 j=1
n aj<
(1 + a1 + . . . + aj−1 )(1 + a1 + . . . + aj )j=1
= 1n 1 − 1 + a + . . . + a
1 jj=1 1 + a1 + . . . + aj−1
1= 1 − 1 + a1 + . . . + an
1
= √2 .
Problem 15 Positive numbers α, β, x1 , x2 , . . . , xn (n ≥ 1) satisfy the condition x1 + x2
+ . . . + xn = 1. Prove that
x
3 x3 x3 1 . αx +
1
1 + 2 + . . . + n ≥ βx
2 αx2 + βx3 αxn + βx1 n(α + β) (Moldavian IMO Team Selection Tests
2002)
First proof. By the AM-GM Inequality , we have
3
αx
i

23
+
βx
i
+1
+
n2 (α + β) ≥ n(α + β), αxi +
xi + 1 3xi βxi+1 n(α + β)2
for alli = 1, 2, . . . , n. Therefore
n x3 n αxi + βxi+1 + n(α + β) ≥ n(α + β). αx + βx
i=1 i
n
i + 1 3xi i
+1
i
=1
n
(
α
+
β
)
2
i=1
Since n αx + βx 2
i i+1 = 1 n ( α + β ) . i=1 n(α + β)
and n
3xi ) = 3 n ( α + β ) , i=1 n(α + β
we thus have thatn x3 2 ≥ n(α + β), i=1 αxi + βx
i
+3 i+1 n(α + β)
which yields
n 3
x 1. i=1 αxi + βx
i

i+1 n(α + β)
Second proof. By the H¨older Inequality , we have that
n x3 n n 3

ni (αxi + βxi+1 ) ≥ xi = 1,
i=1 αxi + βxi+1 i=1 i=1
and sincen (αxi + βxi+1 ) = α + β,
i=1
we conclude thatn x3 1 .
i=1 αxi + βx
i ≥ n(α + β) Problem 16 If three nonnegative real numbers a, b, c satisfy the
i+1
condition
1 1 1
,
a2 + 1 +b2 + 1 +c2 + 1 = 2

24
prove that
ab + bc + ca ≤ 3 .2 (Iranian MO 2005)
Proof.
Set the following substitutions:
x
=
1 1 1
a2+1 , y =b2+1 , z =c2+1 ; then 1 > x, y, z > 0 andx + y + z = 2. We furthermore get
a2 = 1 − x= y + z − x≥ 0x 2x
b2 = 1 − y= z + x − y≥ 0y 2y
c2 = 1 − z= x + y − z≥ 0,z 2z
and thereforex, y, z are the sidelengths of a triangle. Next, putm =y+z−x , n =2 z+x−y , p =
x+y−z
. We then have thatm + n + p = 2 and2 2
m n p
a =
n + p, b = p + m, c = m + n.
Now, by using the AM-GM Inequality ,
2
ab
= 2
n +
m m
p + m n + p m + p n + p · n = 2 m + p · n ≤ m + n p, and similarly, get
2
bc

n + p p, 2ca ≤ m + p m + n m + m + n n + p. Thus
2(ab + bc + ca) ≤ m + n + n + p + m + p = 3.m + p n + p m + n m + p m + n n + p
Problem 17 Let a, b, c be positive real numbers. Prove that a2 b2 c2 + 4(a − b)2 b
a +
+c +a ≥ a + b + c b + c.
(Balkan MO, 2005)
Proof. The inequality in question is equivalent with a2 b2 c2 4(a − b)2 b + b − 2a +c + c
− 2b +a + a − 2c ≥a + b + c,
or
(a − b)2 + (b − c)2 + (c − a)2 4(a − b)2 b c a≥ a + b + c.
The Cauchy-Schwarz Inequality yields
(b − c)2 + (c − a)2 2 ≥ (a − b)2 ,
c a (c + a) ≥ (|b − c| + |c − a|)
and thus (b − c)2 + (c − a)2 (a − b)2

c a c + a .
Now, note that it suffices to prove that
(a − b)2 + (a − b)2 4(a − b)2 b c + a≥ a + b + c, which can be rewritten as1 + c + a ≥ a + b +

25
c.
This obviously follows from the same Cauchy-Schwarz Inequality . 1 4
b
Problem 18 If x, y, z are positive numbers satisfying the condition xy + yz + zx = 1,
show that
27 √ x + y +√ y + z +√ z + x 2 ≥ 6√ 3.4 (x + y)(y + z)(z + x) ≥
(Turkish IMO Team Selection Tests 2006)
Proof. By the Cauchy-Schwarz Inequality ,
√ x + y +√ y + z +√ z + x 2 ≤ 3 (x + y + y + z + z + x) = 6(x + y + z),
and hence, in order to prove the left inequality, it suffices to show that
8
(x + y)(y + z)(z + x) ≥ 9(x + y + z).
For this, we proceed as follows:
(x + y)(y + z)(z + x) = (x + y + z)(xy + yz + zx) − xyz
1
≥ (x + y + z)(xy + yz + zx) − 9(x + y + z)(xy + yz + zx)
8
= ) = 89(x + y + z).9(x + y + z)(xy + yz + zx
For the right hand side of the inequality, we can make use of the AM-GM Inequality
in combination with the Minkowski Inequality :

x + y +√ y + z +√ z + x 2 ≥ 3 (x + y)(x + z) + (y + z)(y + x) + (z + x)(z + y)
= 3 x2 + 1 + y2 + 1 + z2 + 1
≥ 3 (x + y + z)2 + (1 + 1 + 1)2
≥ 3 3(xy + yz + zx) + 9 = 6√ 3.
Problem 19 Let a, b, c be positive real numbers. Prove that
a +b +c ≥ 3 +(a − c)2 b c a ab + bc + ca. (Vo Quoc Ba Can)
Proof. The inequality in question is equivalent to
(ab + bc + ca) a +b +c ≥ 3(ab + bc + ca) + (a − c)2 ,b c a
or
(a2 + b2 + c2 − ab − bc − ca) + ab2 bc2 ca2 2 .c +a +b − ab − bc − ca ≥ (a − c)
This rewrites as
1 2 + (b − c)2 + (a − c)2 ] +a(b − c)2 b(c − a)2 c(a − b)2 2 ,2[(a − b) + + ≥ (a − c)
c a b
or (a − b)2 (b + 2c) + (b − c)2 (c + 2a) (a − c)2 (a − 2b)
≥ .b c b
By the Cauchy-Schwarz Inequality , we get
(a − b)2 (b + 2c) + (b − c)2 (c + 2a)≥ [(a − b) + (b − c)]2 b
c
b+2c
b +c
c+2a
(
a

26
c
)
2
= (b + 2c)(c + 2a),2ab + 2bc + 2c2
and thus it is suffice to show that
(b + 2c)(c + 2a) ≥ a − 2b 2ab + 2bc + 2c2 a , or equivalently,
4(ab2 + bc2 + ca2 − 3abc) + 4b2 c + 11abc ≥ 0, which is obviously true according to
the fact that
ab2 + bc2 + ca2 ≥ 3abc (by the AM-GM Inequality ).
Remarks. Moreover, for any positive real numbersa,b,c, the constantk = 1 is the largest
k for which the following inequality is valid:
a + b + c ≥ 3 + k(a − c)2
b c a ab + bc + ca.
The proof for this last comment is left to the reader.
Problem 20 Let a, b, c be nonnegative real numbers satisfying ab + bc + ca = 3. Prove
that 1 1
1 + a2 (b + c 1 + 2 1 ) + 1 + b2 (c + a) + 1 + c2 (a + b) ≤ 3 abc. (MathLinks Contest
2008)
Proof. Note that the inequality is equivalent to
a2 (b + c)) + 3 ≥ 3. cyc 1 + a2 (b + c 1 + 2abc
By the Cauchy-Schwarz Inequality , we have
36 18, cyc 1 + a2 (b + c)
a2(b + c) ≥ (b + c)[1 + a2 (b + c)] =
cyc 9 + a (1 − abc)
cyc
and hence it suffices to prove that
6 +1 ≥ 1. 9 + a (1 − abc)1 + 2abc
cyc
For this, we proceed by denoting withp,r the termsa + b + c, andabc, respectively. In
this case, the last inequality becomes
6 ) +1 ≥ 1, 9 + p(1 − r 1 + 2r which now rewrites as
(1 − r)(3 − pr) ≥ 0.
This one is obviously valid, since by the AM-GM Inequality we have ab
+
bc
+
ca
3/2
r = abc ≤3 = 1.
and (ab + bc + ca)2 pr = abc(a + b + c) ≤ = 3.
3
Problem 21 Let a, b, c be positive real numbers such that 2a + b = 1. Prove that
5a3 + 4b3 + 3c3

bc

27
≥ 4.
ca ab
(Tran Van Luan)
Proof. We make use of the AM-GM Inequality , as follows:
5a3 + 4b3 + 3c3 a3 b3 b3 c3 c3 a3 bc
= 3
+ + + + 2 +
ca ab bc ca ca ab ab bc
6ab + 2bc + 4ca
≥ c a b
=
4
ab+ ca+ 2 ab+ bc
c b c a
≥ 8a + 4b = 4.
Problem 22 i) If x, y and z are three real numbers, all different from 1, such that xyz =
1, then prove that
x2 y2 z2
( x − 1)2 + (y − 1)2 + (z − 1)2 ≥ 1. ii) Prove that equality is achieved for infinitely
many triples of rational numbers x, y and z.
(IMO 2008)
Proof. (i) Sincexyz = 1, there exist three nonnegative real numbersa,b, andc such that
x = a , y = b , z = c ; the inequality becomesb c a
a2 b2 c2
(a − b)2 + (b − c)2 + (c − a)2 ≥ 1. By Cauchy-Schwarz Inequality , we get
a
2
2
(a − b)2 (a − c)2 ≥ a(a − c)
a − b
cyc cyc cyc 2
≥ a2 − ab .
cyc cyc
Moreover, we have
(a − b)2 (a − c)2 = (a − b)2 (a − c)2 + 2 (a − b)(a − c) · (b − c)(b − a)
cyc cyc cyc
2 2
= (a − b)(a − c) = a2 − ab .
cyc cyc cyc
This proves (i).
(ii) As we have seen in the proof of (i), the equality holds if and only if
a b c a−b ) = b−c c−a

(b − c)(b − a) = (c − a)(c − b). (a − b)(a − c


This rewrites as
a ) =b c

(b − c)2 (b − a) = (c − a)2 (c − b), (a − b)2 (a − c

28
or b +c +a = 3.
a b c
Returning to the notations withx,y,z, this is equivalent with
1
+ 1 + 1 = 3.x y z
Now, note that in the above equation we can choose (x, y, z) = n , −n(n + 1), −n+1 ,
where n is an arbitrary rational number. This shows(n+1)2 n2
that the equality in our problem is achieved for infinitely many triples of rational
numbers
x,y andz.
Problem 23 Let a, b, c be positive real numbers. Prove that a + b c 9 c(c + a)2 + a(a +
b)2 ≥ 4(ab + bc + ca). b(b + c)2
(Ho Phu Thai, Math. Reflections)
Proof. By applying the Cauchy-Schwarz Inequality and then following the Nesbitt
Inequality , we get
a ab ≥a 2 b
+
c ≥ 9 . cyc b(b + c)2 4cyc cyc
Thereforea ≥ 9 4(
ab
+ bc + ca ) . cyc b(b + c)2
Problem 24 Let a, b, c be nonnegative real numbers such that a + b + c = 1. Prove that
b − c)2 √
a + ( b + √ c ≤ √3.4 +
(Chinese Girls MO 2008)
Proof. Setx = √ bc; then we have
√ b +√ c = b + c + 2√ bc = 1 − a + 2√ bc = √ 1 − a + 2x,
and
b − c)2 + (b + c)2 − 4bc = 1
a + ( (1 + a)2 − 4x2.4 = a 4 2
Therefore, the inequality in question now rewrites as
(1 + a)2 − 4x2 + 2√ 1 − a + 2x ≤ 2√ 3.
By the Cauchy-Schwarz Inequality , we have
(1 + a)2 − 4x2 + 2√ 1 − a + 2x ≤ 3 [(1 + a)2 − 4x2 + 2(1 − a + 2x)],
and hence it suffices to prove that
(1 + a)2 − 4x2 + 2(1 − a + 2x) ≤ 4, or equivalently, a2 − 4x2 + 4x − 1 ≤ 0.
This can be also rewritten as
(1 − 2x − a)(1 + a − 2x) ≥ 0,
which is obviously true since
1 + a − 2x ≥ 1 − 2x − a = 1 − 2√ bc − a ≥ 1 − (b + c) − a = 0.
Problem 25 Let a, b, c be the sidelengths of a triangle. Prove that a3 a2 a
3 + (b + c)3 + 1 ≥ 2 a2 + (b + c)2.
cyc cyc
(Pham Quang Vu)
Proof. The inequality is equivalent to

29
a 2aa2 1 − a2 + (b + c)2 ≥ 0,
3 3 +
cyc a + (b + c) a + b + c
which furthermore rewrites as a(b + c)(b + c − a)3
[
a
3 + (b + c)3 ][a2 + (b + c)2 ] ≥ 0.

cyc
This last equivalent form is obviously valid, according to the Triangle Inequality. The
equality holds if and only if (a, b, c) = (t, t, 0) for some positivet.
Remarks. In fact, the original inequality holds for any nonnegative real
numbersa,b,c, not necessarily for them to be the sidelengths of a triangle. Indeed, we
may rewrite the inequality as
a3 a3 2a2 2a2 a
3 + (b + c)3 − a3 + b3 + c3 + a2 + b2 + c2 − a2 + (b + c)2 ≥ 0,
cyc
and hence, in order to prove this one, it suffices to show that
a3 a3 2a2 2a2 a3 + (b + c)3 − a3 + b3 + c3 + a2 + b2 + c2 − a2 + (b + c)2 ≥ 0, or 2 − 2 a a a2
+ (b + c)2 ≥ a3 + b3 + c3 − a3 + (b + c)3. a2 + b2 + c2
This can be rewritten into 4bc ≥ 3abc(b + c) (a3 + b3 + c3 ) [a3 + (b + c)3 ], (a2 + b2 +
c2 )[a2 + (b + c)2 ]
or equivalently,
a3 + b3 + c3 3a(b + c)[a2 + (b + c)2 ].a2 + b2 + c2 ≥ 4[a3 + (b + c)3]
Without loss of generality, we can assume thatb + c = 2 and furthermore putx = bc;
then 1 ≥ x ≥ 0. The last inequality now becomes
) = 8 +
a
3 − 6x 3a(a2

f (x + 4) ≥ 0.
2 3
4 + a − 2x 2(a + 8)
On the other hand,f (x) is a monotic function, hence
f (x) ≥ min {f (1), f (0)} ,
and since
f
(1)
=
a3 + 2 − 3a(a2 + 4) = 2a6 − 3a5 + 2a3 − 24a + 32 a2 + 2 2(a3 + 8) 2(a3 + 8)(a2 + 2)
=
2(a − a2 − 1)2 + (a3 + 1)(a − 1)2 + 5a3 − 5a2 − 22a + 29 2(a3 + 8)(a2 + 2)
3

3 2
≥ 5a − 5a − 22a + 29 02(a3 + 8)(a2 + 2) >
together with
f
(0)

30
=
a3 + 8 − 3a(a2 + 4) = 2a6 − 3a5 + 8a3 − 48a + 128 a2 + 4 2(a3 + 8) 2(a3 + 8)(a2 + 4)
2(a3 − a2 − 1)2 + (a3 + 1)(a − 1)2 + 11a3 − 5a2 − 46a + 125= 2(a3 + 8)(a2 + 4)
11
a
3 − 5a2
≥ − 46a + 125 > 0, 2(a3 + 8)(a2 + 4)
it now follows that
f (x) ≥ min {f (1), f (0)} ≥ 0, Note that this time the equality holds if and only ifabc =
0.
Problem 26 Prove that for any real numbers a, b, c the following inequality holds
(b + c − a)2 (c + a − b)2 (a + b − c)2 ≥ (b2 + c2 − a2 )(c2 + a2 − b2 )(a2 + b2 − c2 ).
(Japanese MO 2001)
Proof. The inequality is equivalent to
3(a2 + b2 + c2 )(b + c − a)2 (c + a − b)2 (a + b − c)2 ≥ ≥ 3(a2 + b2 + c2 )(b2 + c2 − a2 )
(c2 + a2 − b2 )(a2 + b2 − c2 ).
By the Cauchy-Schwarz Inequality , we have
3(a2 + b2 + c2 ) ≥ (a + b + c)2 ,
and thus it is suffice to prove that
(a + b + c)2 (b + c − a)2 (c + a − b)2 (a + b − c)2 ≥ ≥ 3(a2 + b2 + c2 )(b2 + c2 − a2 )(c2 +
a2 − b2 )(a2 + b2 − c2 ).
Since
2
(a + b + c)2 (b + c − a)2 (c + a − b)2 (a + b − c)2 = 2 a2 b2 − a4
cyc cyc
and
3(a2 + b2 + c2 )(b2 + c2 − a2 )(c2 + a2 − b2 )(a2 + b2 − c2 ) = 3 2 a4 b4 − a8 ,
cyc cyc
the last inequality is equivalent to
2
2 xy − x2 ≥ 3 2 x2 y2 − x4 ,
cyc cyc cyc cyc
wherex = a2 ,y = b2 ,z = c2 . This rewrites as
4 x2 (x − y)(x − z) ≥ 0,
cyc
which is a particular case of the Schur Inequality .
Problem 27 Let a, b, c be the sidelengths of a given triangle. Prove that
(a + b)(b + c)(c + a) + (−a + b + c)(a − b + c)(a + b − c) ≥ 9abc.
(Virgil Nicula and Cosmin Pohoat¸˘a, Math. Reflections)
Proof. Rewrite the inequality as
(a + b)(b + c)(c + a) − 8abc ≥ abc − (−a + b + c)(a − b + c)(a + b − c). The left hand
side equals a(b − c)2 ,
cyc

31
and the right hand side equals
1 · (−a + b + c)(b − c)2 .
2 cyc
Consider the substitutionsa = y + z,b = z + x,c = x + y. In this case, the inequality is
equivalent to
(x + y)(x − y)2 ≥ z(x − y)2 .
cyc cyc
After expanding, this rewrites as
x3 + y3 + z3 + 3xyz ≥ xy(x + y) + yz(y + z) + zx(z + x),
which is, again, a particular case of the Schur Inequality .
Problem 28 Let a, b, c be positive real numbers. Prove that 1
+ 1
+ 1
a + 1 +
1 1 1
b + 1 + c + 1≥ 9 . ab c abc + 1 (Walther Janous)
Proof. By the AM-GM Inequality , we have
1 1
a + 1 +b + 1 +c + 1
1≥ 3 3 (a + 1)(b + 1)(c + 1) 3 =
≥ √ 3 abc 1 + 1 1 + 1 1 + 1
a b c
9 ,3 abc 3 + 1 + 1 + 1
a b c
hence
1 1 1 9
a b c3 3
1 + 1 + 1 + 1 + + 1 + ≥ √
a b c + 1 abc 1+1+1 + 1
a b c

≥9 .√abc √ 3 3 abc + 1
It is thus enough to prove that
√ √
abc + 1 ≥3 abc3 abc + 1 , which is true since for allx > 0, we have
x3 + 1 − x(x + 1) = (x − 1)2 (x + 1) ≥ 0.
Problem 29 Let a, b, c be positive real numbers contained in the interval [0, 1]. Prove
that
2a + 2b + 2c + abc ≤ 4.
1 + bc 1 + ca 1 + ab
(adapted after Polish MO 2005)
Proof. We have
2(1 + bc) − (1 + b)(1 + c) = (1 − b)(1 − c) ≥ 0,
and therefore
2a − 4 ≤ 4a 4b 4c (1 +
b

32
)(1 +
c
) +
(1 +
c
)(1 +
a
) +
(1 + a )(1 + b ) − 4 cyc 1 + bc
=
4(a2 + b2 + c2 − 1 − ab − bc − ca − abc) (1 + a)(1 + b)(1 + c)
4(a + b + c − 1 − ab − bc − ca − abc)≤ (1 + a)(1 + b)(1 + c)
4(a − 1)(b − 1)(c − 1) − 8abc= (a + 1)(b + 1)(c + 1)
8abc≤ −
(1 + a)(1 + b)(1 + c)
8abc
≤ − (1 + 1)(1 + 1)(1 + 1) = −abc.
This yields
2a + 2b + 2c + abc ≤ 4. 1 + bc 1 + ca 1 + ab
The equality holds if and only if (a, b, c) ∈ {(1, 1, 1), (0, 1, 1), (1, 0, 1), (1, 1, 0)}.
Problem 30 Let a, b, c be nonnegative real numbers a, b, c satisfying
max {b + c − a, c + a − b, a + b − c} ≤ 1.
Prove that a2 + b2 + c2 ≤ 1 + 2abc. (Chendi Huang)
Proof. Without loss of generality, assume thata ≤ b ≤ c. Since
b + c ≤ a + 1 ≤ b + 1,
we getc ≤ 1.
1 . Ifa ≤ bc, we have
( a2 + b2 + c2 ) − (1 + 2abc) = a2 + (b + c)2 − 2(a + 1)bc − 1 ≤ a2 + (a + 1)2 − 2(a
+ 1)bc − 1 = 2(a + 1)(a − bc) ≤ 0.
2 . Ifa ≤ bc, we have
bc − a ≤ bc − (b + c − 1) = (1 − b)(1 − c).
Hence
( a2 + b2 + c2 ) − (1 + 2abc) = (a − bc)2 − (1 − b2 )(1 − c2 ) ≤ (1 − b)2 (1 − c)2 − (1
− b2 )(1 − c2 ) = −2(1 − b)(1 − c)(b + c) ≤ 0.
This completes our proof.
Problem 31 If x, y, z are real numbers satisfying xyz = −1, prove that
x
4 + y4 + z4 + 3(x + y + z) ≥y2 + z2 z2 + x2 x2 + y2
+ +
x y z .
(Iranian MO 2005)
Proof. According to the condition from the hypothesis, we have
3(x + y + z) = −3xyz(x + y + z),
and thus

33
y2 + z2 z2 + x2 x2 + y2
2 2 2 2 2 2
x +y +z = −yz(y + z ) − zx(z + x ) − xy(x + y )
= −x3 (y + z) − y3 (z + x) − z3 (x + y).
The inequality in question is now equivalent with
x4 + y4 + z4 − 3xyz(x + y + z) ≥ −x3 (y + z) − y3 (z + x) − z3 (x + y), which rewrites as
x4 + y4 + z4 + x3 (y + z) + y3 (z + x) + z3 (x + y) − 3xyz(x + y + z) ≥ 0,
or (x + y + z)(x3 + y3 + z3 − 3xyz) ≥ 0,
which is obviously true according to
(x+y+z)2 (x2 +y2 +z2 −xy−yz−zx) ≥ 0, and to the well-knownx2 +y2 +z2 ≥ xy+yz+zx.
Problem 32 Let a, b, c, d be positive real numbers satisfying the condition a + b + c + d
= abc + bcd + cda + dab. Prove that
a + b + c + d +
2a 2b 2c 2d ≥ 8. + 1 + + 1 + + 1 +
a b c d + 1
(Vo Quoc Ba Can)
Proof. Putx = a + b + c + d = abc + bcd + cda + dab; then, by the AM-GM
Inequality , we havex ≥ 4.
By the MacLaurin Inequality , we have now that
2 ab ≥ 3 a abc = 3x,
sym cyc cyc
and moreover, according to the Cauchy-Schwarz Inequality ,
2a 2x2 2x2 2x2 2x.
cyc
a + 1 ≥ x + a2 = x + x2 − 2 ab ≥ x2 + x − 3x = x − 2
cyc sym
We thus need to prove that 2x x +x − 2 ≥ 8, which is obviously true, since it rewrites as
(x − 4)2 ≥ 0.
Problem 33 Let a, b, c be nonnegative real numbers. Prove that a2 + 2bc+ b2 + 2ca+ c2
+ 2ab b2 + c2 c2 + a2 a2 + b2 ≥ 3. (Russian MO 1999)
Proof. Without loss of generality, we may assume thatc = min {a, b, c}; then we have
a2 + 2bc ≥ a2 + c2 , b2 + 2ca ≥ b2 + c2 , c2 + 2ab 2ab,b2 + c2 b2 + c2 c2 + a2 a2 + c2 a2 + b2 ≥ a2
+ b2
and since
a2 + c2 b2 + c2 a − b= a2 + c2 a+ b2 + c2 b b2 + c2 + a2 + c2 − b a b2 + c2 − b a2 + c2 − a (a
− b)(ab − c2 ) + (b − a)(ab − c2 )= b(b2 + c2) a(a2 + c2)
(
a

b
)
2
(ab − c2 )(a2 + b2 + c2 = + ab) ≥ 0, ab(a2 + c2 )(b2 + c2 ) it is thus enough to prove that
a + b + 2ab a2 + b2 ≥ 3. b a

34
This rewrites asa +b − 2 ≥ 1 − 2ab a2 + b2, b a
or equivalently,
( a − b)2 (a − b)2
ab≥ a2 + b2,
which is obviously valid, according to the fact thata2 + b2 ≥ 2ab ≥ ab. The equality
holds if and only if (a, b, c) ∈ {(0, t, t), (t, 0, t), (t, t, 0)}, for some positive numbert.
Remarks. In the same manner, one can prove that a2 + 2bc+ b2 + 2ca+ c2 + 2ab b2 + c2
c2 + a2 a2 + b2 ≥ 3. The details are left to the reader.
Problem 34 Let a, b, c be positive real numbers. Prove that
bc
c(c + a b(b + c) c + a a + b b + ab) + a(a + b) + ca ≥ a + b + c c. (Moldavian MO 1999)
Proof. The inequality is equivalent to
ab) + c+ bc ca
a(a + b) + a + b(b + c) + b ≥ 3, c(c + a c + a a + b b + c
or a2 + bc ) +b2 + ca ) + c2 + ab ≥ 3. a(a + b b(b + c c(c + a)
By the AM-GM Inequality , we have
a2 + bc) + b2 + ca) + c2 + ab ≥ 3 3 (a2 + bc)(b2 + ca)(c2 + ab) a(a + b b(b + c c(c + a)
abc(a + b)(b + c)(c + a) ,
hence it suffices to prove that
(a + bc)(b2 + ca)(c2 + ab) ≥ abc(a + b)(b + c)(c + a).
2

Since (a2 + bc)(b2 + ca) − ab(a + c)(b + c) = c(a + b)(a − b)2 , we get
(a2 + bc)(b2 + ca) ≥ ab(a + c)(b + c),
and similarly, (b2 +ca)(c2 +ab) ≥ bc(a+b)(c+a) and (a2 +bc)(c2 +ab) ≥ ac(a+b)(b+c).
Thus
(a2 + bc)2 (b2 + ca)2 (c2 + ab)2 ≥ a2 b2 c2 (a + b)2 (b + c)2 (c + a)2 , and so
(a2 + bc)(b2 + ca)(c2 + ab) ≥ abc(a + b)(b + c)(c + a).
Problem 35 Let a, b, c be positive real numbers such that ab + bc + ca ≥ 3. Prove that
√b c 3a + √b + c + √c + a ≥ √2 .a + b
(Pham Huu Duc, Math. Reflections)
Proof. From the H¨older Inequality , we have
2 3
√a a(a + b) ≥ a ,
cyc a + b cyc cyc
and so it suffices to show that
2(a + b + c)3 ≥ 9(a2 + b2 + c2 + ab + bc + ca),
or equivalently, 2(a + b + c)3 ≥ 9 (a + b + c)2 − ab − bc − ca .
This rewrites as 2(a + b + c)3 ≥ 9 (a + b + c)2 − 3 , or
(2a + 2b + 2c + 3)(a + b + c − 3)2 ≥ 0, which is obviously true.
Problem 36 Let x, y, z, t be positive real numbers such that
1 1 1 1

35
.
x + 1 +y + 1 +z + 1 +t + 1 = 1
Prove that
1 1 1 1
min + 1 + 1z, + 1 + 1t, + 1 + 1x, + 1 + 1≤ 1x y y z z t t x y
1 + 1 + 1 1 + 1 + 1 1 + 1 + 1 1 + 1 + 1
≤ max z, t, x, .x y y z z t t x y
(Pham Van Thuan)
Proof.
Set
a
=
1 1 1 1

x +1 ,b =y+1 ,c =z+1 ,d =t+1 ; then we have 1 ≥ a, b, c, d ≥ 0 and a + b + c + d = 1.


Without loss of generality, we may now assumea ≥ b ≥ c ≥ d. In this case, the
inequality now becomes
b + c + d ≤ 1 ≤ a + b + c
1 − b 1 − c 1 − d 1 − a 1 − b 1 − c, which is equivalent with
c d a b c a + c + d
b + a + b + d + a + b + c ≤ 1 ≤ b + c + d + c + d + a + d + a + b. On the other hand, sincea
≥ b ≥ c ≥ d, we have that
b c c d d a + c + d
b ≤ b + c + d, a + b + d ≤ b + c + d, a + b + c ≤ b + c + d, and therefore

b + c d b c d a + b + d + a + b + c ≤ b + c + d + b + c + d + b + c + d = 1. a + c + d
Similarly, we get
a b b c c b + c + d
a ≥
a + b + c, c + d + a ≥ a + b + c, d + a + b ≥ a + b + c. Hence
a + b c a b c c + d + a + d + a + b ≥ a + b + c + a + b + c + a + b + c = 1. b + c + d
Problem 37 Let a1 , a2 , . . . , an be positive real numbers. Prove that n n ☐n ☐n Tn+1 ☐
aTk ☐ ≥ ak ,j 3
k=1 j=1 k=1
where Tk = k(k+1) is the k-th triangular number. 2
(Jose Luis Diaz-Barrero, Math. Reflections )
Proof. The inequality is equivalent to
n ☐n ☐n Tn+1 ln

a
Tk ☐ ≥ n ln a3

k . j
k=1 j=1 k=1
Consider the function☐ n ☐ f
(
x
) = ln

36
☐ a
x
j ☐
j=1
defined on the real line. We then get
2 j=1 j j=1 j j=1 j f (x) =
n ax ln2 a n ax −n ax ln a
j j ,n ax 2
j=1 j
and since by the Cauchy-Schwarz Inequality we have that ☐n ☐☐n ☐ ☐n ☐2 ☐ ax
ln2 aj ☐☐ ax ☐ ≥ ☐ ax ln aj ☐ ,j j j
j=1 j=1 j=1
it follows thatf (x) is convex for all realsx. Thus, according to the Jensen Inequality ,
we get
n ☐ n ☐ n n Tk ,☐ aTk ☐ = f (T ) ≥ nfk=1 ln
k j nk=1 j=1 k=1
and because
n 1n n = 1 n(n + 1)(2n + 1) n(n + 1)
+ Tk = 2 k2 + k
k=1 k=1 2 6 2 k=1
= n(n + 1)(n + 2) = 1nTn+1,6 3
we conclude that
n n☐ n ☐ n Tk
= nf Tn+1 = n ln ak .☐ aTk ☐ ≥ nfk=1 Tn+1 ln
3 j

k=1 j=1 n 3 k=1


Problem 38 Let a, b, c, d be positive numbers. Prove that
3(a2 − ab + b2 )(c2 − cd + d2 ) ≥ (a2 c2 − abcd + b2 d2 ).
(Titu Andreescu, Math. Reflections )
Proof. We have
2 − ab + b2 )(c2 − cd + d2 ) = (a − b)2 + a2 + b2 (c − d)2 + c2 + d2 4(a

≥ (a − b)2 (c2 + d2 ) + (c − d)2 (a2 + b2 ) + (a2 + b2 )(c2 + d2 ), hence, it suffices to


prove that
3 (a − b)2 (c2 + d2 ) + (c − d)2 (a2 + b2 ) + (a2 + b2 )(c2 + d2 ) ≥ 4(a2 c2 + b2 d2 ),
or equivalently, 3 (a − b)2 (c2 + d2 ) + (c − d)2 (a2 + b2 ) ≥ a2 c2 + b2 d2 − 3a2 d2 −
3b2 c2 .
In fact, we can just prove the following sharper version:
3 (a − b)2 (c2 + d2 ) + (c − d)2 (a2 + b2 ) ≥ a2 c2 + b2 d2 − a2 d2 − b2 c2 .
This rewrites as 3 (a − b)2 (c2 + d2 ) + (c − d)2 (a2 + b2 ) ≥ (a2 − b2 )(c2 − d2 ),
which follows from the AM-GM Inequality :
3 (a − b)2 (c2 + d2 ) + (c − d)2 (a2 + b2 ) ≥ 3 |(a − b)(c − d)| 2(a2 + b2 ) · 2(c2 + d2 ) ≥ 3
|(a − b)(c − d)| (a + b)(c + d) = 3 a2 − b2 (c2 − d2 )
≥ (a2 − b2 )(c2 − d2 ).
Remarks. Actually, we can prove another refinement:
3(a2 − ab + b2 )(c2 − cd + d2 ) ≥ 2(a2 c2 − abcd + b2 d2 ).

37
Proof . Seta = xb, c = yd; then the inequality becomes
3(x − x + 1)(y2 − y + 1) ≥ 2(x2 y2 − xy + 1),
2

or f (x) = (y2 − 3y + 3)x2 − (3y2 − 5y + 3)x + 3y2 − 3y + 1 ≥ 0. We havey2 − 3y + 3 >


0, and since
∆f = (3y2 − 5y + 3)2 − 4(y2 − 3y + 3)(3y2 − 3y + 1) = −3(y2 − 3y + 1)2 ≤ 0,
we can conclude that
f (x) ≥ 0.
Problem 39 Let a, b, c be real numbers such that a + b + c = 1. Prove that
a b c ≤ 9 . + 1 + + 1 +
a2 b2 c2 + 1 10
(Polish MO 1997)
Proof. In order to prove this inequality, we shall consider two cases: Case 1. If min {a,
b, c} ≥ −3 , then we have4
36a + 3− a + 1 = (4a + 3)(3a − 1)2 2 50(a2 + 1)≥ 0, 50 a
and so
a ≤ 36a + 3 a2 + 1 50 .
Hence
a b 36 9
+ 1 + c
≤ 50(a + b + c) + 50 = 3650 + 50 =9.
a2 + 1 +b2 c2 + 1 10
3 ; without loss of generality, we can assume thatc < −3 Case 2. If min {a, b, c} <
−4 4 (note that this not necessarily means thatc = min {a, b, c}).
From the AM-GM Inequality , we have
b ≤ 1, b2 + 1 2
and thus, the initial inequality is proved if we can show that
a ≤ 2. a2 + 1 5

If a ≤ 1 ora ≥ 2, we are done. So, it suffices to consider the case 2 ≥ a ≥ 1 .


Similarly,2 2
we can show separately that it is enough to show the case when 2 ≥ b ≥ 1 . Therefore,
we2
can combine these two remarks and can resume to proving the problem under the
condition 2 ≥ a, b ≥ 1 . In this case,2
−3 > c = 1 − a − b ≥ −3,4
which means thatc ≤ − 3 .c2 + 1 10
Hence a b
+ 1 + c ≤ 1 2 + 13
− 10 =7< 9. + 1 +
a2 b2 c2 + 1 10 10
The equality holds if and only ifa = b = c = 1 . This completes our proof.3
Problem 40 Let n be a positive integer, and let x and y be positive real numbers such
that xn + yn = 1. Prove that
n
1 + x2k n 1 + y2k 1.
4k 4k < (1 − x)(1 − y)
k=1 1 + x 1 + y k=1

38
(IMO Shortlist 2007, proposed by Estonia)
Proof. The inequality
2
1 + t 1
1 + t4< t
holds for allt ∈ (0, 1) because it is equivalent to
0 < t4 − t3 − t + 1 = (1 − t)(1 − t3 ).
Applying it tot = xk and summing overk = 1, 2, . . . , n, we get
n 1 + x2k n 1 xn − 1 yn 1 +
=
x
4k< x xn (x − 1) = xn (1 − x).
k
k=1 k=1
Similarly, we obtain thatn 1 + x2k xn
< n
y (1 − y), k=1 1 + x4k
and therefore, by multiplying them up, yields
n 1 + x2k n 1 + y2k 1 4k 4k < (1 − x)(1 − y)
. k=1 1 + x 1 + y k=1
Problem 41 Let a, b, c be positive real numbers such that a + b + c + 1 = 4abc. Prove
that
1 + 1 + 1 1 1 1
≥ 3 ≥ √
a
b c
+ √bc + √ca.ab
(Daniel Campos Salas, Math. Reflections)
Proof. From the AM-GM Inequality , we have
(ab + bc + ca)2 ≥ 3abc(a + b + c), (ab + bc + ca)3 ≥ 27a2 b2 c2 ,
hence (ab + bc + ca)2 (ab + bc + ca)3 a + b + c ≤
3abc , 1 ≤27a2b2c2 . This implies that 4
abc

(ab + bc + ca)2 + (ab + bc + ca)3 3abc 27a2 b2 c2 , or equivalently,2 1 1 + 1 + 1 31 1
3 ab c
Now, we putt =
+ 1 + 1+ 27 a b c
1 + 1 + 1 , and the last inequality now becomes
a b c
≥ 4.
t2 t3 2 ≥ 0.3 + − 4 = (t − 3)(t + 6)
27
Thust = + + 1 ≥ 3. For the right hand side of the inequality in question, we begin
1 1
bya b c
rewriting the condition from the hypothesis as
1 + 1 + 1 1
+ = 4,ab bc ca abc
and we proceed by making use of the contradiction method. In this case, let us assume
that the following inequality holds:

39

1
1 1
+ √bc + √ca> 3.ab
Then, there existsk greater than one such that
1
√1 + √bc + √1 = 3k.ab ca
Set
x
=
1 1
1
k √ bc , y =k √ ca , z =k √ ; sox + y + z = 3, and we deduce thatab
4 =1 + 1 + 1 +1 = k2 (x2 + y2 + z2 ) + k3 xyz > x2 + y2 + z2 + xyz.ab bc ca abc
Furthermore, assume thatz = min {x, y, z}, and becausex + y + z = 3, the numberz
satisfiesz ≤ 1. Therefore
x2 + y2 + z2 + xyz = z2 + (x + y)2 + (z − 2)xy x + y
2
≥ z2 + (x + y)2 + (z − 2)2
3 − z
2
= z2 + (3 − z)2 + (z − 2)2
1
=2 + 4 ≥ 4,4(z + 2)(z − 1)
which contradicts the previous inequality emerged from our assumption. In this case,
we must have that1 1
√1 + √bc + √ca ≤ 3.ab
This proves our inequality.
Problem 42 Let a, b , c be nonnegative real numbers such that a + b + c = 3. Set x
= √ a2 − a + 1, y = √ b2 − b + 1 and z = √ c2 − c + 1. Prove that: xy + yz + zx ≥ 3 and x
+ y + z ≤ 2 +√ 7.
(adapter after the Vietnamese MO 2007)
Proof. By the AM-GM Inequality , we have
x
=
a
2 − a + 1 ≥ a2 −a2 + 1 + 1 = a2 + 1

2 2 ,
and similarly,
y
=
b
2 − b + 1 ≥ b2 −b2 + 1 + 1 = b2 + 1

40
2 2 .
Therefore, by the Cauchy Schwarz Inequality , we now get
1
xy ≥ 1 (a2 + 1)(b2 + 1) = 1 (a2 + 1)(1 + b2 ) ≥ 2(a + b).2 2
Analogously, we establish that
1 1
yz ≥ 2(b + c), and zx ≥ 2(c + a).
Summing up all of them yields
1
xy + yz + zx ≥ ) + 12(b + c) + 12(c + a) = a + b + c = 3.2(a + b
This proves the first inequality. For the second, we begin by assuming (without
loss of generality) thata ≥ b ≥ c. In this case, 2b ≤ a + b ≤ a + b + c = 3, and therefore c
≤ b ≤ 3 .2
Since
4(m2 − m + 1) − (m + 2)2 = m(3m − 8), we get
m2 − m + 1 ≤ 1 + 1m, 2
for all real numbersm satisfying 3 ≥ m ≥ 0.2
From this, we have
y + z = b2 − b + 1 + c2 − c + 1 ≤ 2 + 12(b + c) = 2 + 12(3 − a),
hence it suffices to prove that
1 ≤ √7,2(3 − a) + a2 − a + 1
or equivalently,
1 ≤ √7 − a2 − a + 1.2(3 − a)
This rewrites as1 (3 − a)(a + 2) ,2(3 − a) ≤ √7 +√a2 − a + 1
or √ 7 + a2 − a + 1 ≤ 2(a + 2), which is obviously true according to the fact that

7 + a2 − a + 1 − 2(a + 2) < 3 + a2 + 2a + 1 − 2(a + 2) = −a ≤ 0.
The equality holds if and only if (a, b, c) ∈ {(3, 0, 0), (0, 3, 0), (0, 0, 3)}.
Problem 43 Let n ≥ 2 be a given integer. Determine (a) the largest real cn such that
1 + 1 + . . . + 1 ≥ cn 1 + a 1 + a 1 + a holds for any positive numbers a , a , . . . , a with
1 2 n 1 2 n
a1 a2 . . . an = 1. (b) the largest real dn such that
1 + 1 + . . . + 1 ≥ dn 1 + 2a 1 + 2a 1 + 2a holds for any positive numbers a , a , . . . , a
1 2 n 1 2 n
with a1 a2 . . . an = 1. (Italian MO 2007)
Proof.
(a) Let us see what happens if
a
1
=
a
2
=
. . .
=

41
a
n

1
=
x
and
a
n
=
1
xn−1 , where x is an arbitrary positive real number. In this case, the inequality becomes
n−1n − 1 x ≥ cn, + 1 +
x xn−1 + 1
and by settingx near to infinity, we getcn ≤ 1. Now, we are left to show that indeed we
have 1 + 1 + . . . + 1 ≥ 1. 1 + a 1 + a 1 + a
1 2 n
For this, proceed by assuming (without loss of generality) thata1 ≤ a2 ≤ . . . ≤ an ; then
a1 a2 ≤ 1, and therefore
1 + 1 ≥ 1 + 1 = 1 + a 1 .
1 + a1 1 + a2 1 + a1 1 +1 1 + a1 a1 + 1 = 1a1
This proves (a).
(b) Ifn = 2 then the inequality becomes
1 + 1 ≥ d2, i.e. 1 + a1 ≥ d2.
1 + 2a1 1 + 2a2 1 + 2a1 a1 + 2
This rewrites as 2(a1 − 1)2 + 1) + 2 ≥ d ,
2 3(a1 + 2)(2a1
and it easily follows thatd
Ifn ≥ 3, we seta
2 is the ”sharpest choice”.2 = 3 1 , wherex is an arbitrary = a = . . . = a
1 2 n−1 = x andan =xn−1
positive real number. In this case, the inequality becomes
n − 1 + xn−1
n−1 + 2 ≥ dn, 1 + 2x x

and thus, by letting (again)x near to infinity, we get getdn ≤ 1. We are now left to prove
that 1 + 1 + . . . + 1 ≥ 1. 1 + 2a 1 + 2a 1 + 2a
1 2 n
Without loss of generality, assume thata1 ≤ a2 ≤ . . . ≤ an ; thena1 a2 a3 ≤ 1, and
therefore there exists a positive numberk such thatk < 1 and satisfyinga1 a2 a3 = k3 .
Now, seta1 = knp , a2 = kpm , a3 = kmn , and thusm2 n2 p2
m2 n2 p2 1 + 2a
1 + 1 + 1 = m2 + 2knp + n2 + 2kpm + p2 + 2kmn 1 + 2a 1 + 2a
1 2 3
m
2 n2 p2
≥ m2 + 2np + n2 + 2pm + p2 + 2mn (
m
+

42
n
+
p
)
2
≥ m2 + 2np + n2 + 2pm + p2 + 2mn = 1. This yields 2 ,ifn = 2
.n = 3d 1, ifn > 2
Problem 44 Let a, b, c be positive real numbers. Prove that bc + ca + ab ≤ a + b + c
a2 + bc b2 + ca c2 + ab b + c c + a a + b.
(Pham Huu Duc, Math. Reflections)
Proof. The inequality is equivalent to a
+
b
+
c
+
a
2 2 2 + 2
b c b + ca + c2 + ab ≥ 3. b + c c + a a + b a2 + bc
From the Cauchy-Schwarz Inequality , we have
2 b2 c2 a b c a
+ + +
b + c c + a a + b a2 + bc + b2 + ca + c2 + ab a
2 2 2 2 2 2 =
b c a b c a(b + c) + b(c + a) + c(a + b) + a2 + bc + b2 + ca + c2 + ab (
a
+
b
+
c
+
a
+
b
+
c
)
2
≥ a(b + c) + b(c + a) + c(a + b) + a2 + bc + b2 + ca + c2 + ab 4(

a
+
b
+
c
)
2
= 2
a + b2 + c2 + 3(ab + bc + ca),
hence it suffices to prove that

43
4(a + b + c)2 ≥ 3(a2 + b2 + c2 ) + 9(ab + bc + ca),
which is obviously true according to the well-knowna2 + b2 + c2 ≥ ab + bc + ca.
Problem 45 Real numbers a1 , a2 , . . . , an are given. For each i (1 ≤ i ≤ n) define
di = max {aj | 1 ≤ j ≤ i} − min {aj | i ≤ j ≤ n}
and let d = max {di | 1 ≤ i ≤ n}.
(a) Prove that for any real numbers x1 ≤ x2 ≤ . . . ≤ xn , we have
max {|xd .i − ai | | 1 ≤ i ≤ n} ≥2
(b) Show that there are real numbers x1 ≤ x2 ≤ . . . ≤ xn such that we have equality in
(a).
(IMO 2007, proposed by New Zealand)
Proof. (a) Assume thatd = dm for some indexm, and letk andl (k ≤ m ≤ l) be the
indeces such thatdm = ak − al . Thendm = ak − al ≤ (ak − xk ) + (xl − al ) hence ak − xk ≥
d/2 orxl − al ≥ d/2. The claim follows immediately.
(b) LetMi = max {aj : 1 ≤ j ≤ i} andmi = min {aj : i ≤ j ≤ n}. Setxi = (mi + Mi )/2.
Clearly,mi ≤ ai ≤ Mi and both (mi ) and (Mi are nondecreasing. Furthermore,
d
−i mi − Mi = xi − Mi ≤ xi − ai .2 = 2
Similarly,xi − ai ≤ di /2, and therefore,
max {|xi − ai | | 1 ≤ i ≤ n} ≤ max di : 1 ≤ i ≤ n .2
Thus, the equality in (a) holds for the sequence{xi }.
Problem 46 Let a, b, c be nonzero positive numbers. Prove that
a2 b2 c2 4a2 + ab + 4b2 + 4b2 + bc + 4c2 + 4c2 + ca + 4a2 ≤ 1. (Zhao Bin, Math.
Reflections)
Proof. Setx = b , y = c , z = a ; thenxyz = 1 and the inequality becomesa b c
1 1 1
√ √ ≤ 1.
4x2 + x + 4 +4y2 + y + 4 + 4z2 + z + 4
We have (x + 1)2 (4x2 + x + 4) − 4(x2 + x + 1)2 = x(x − 1)2 ≥ 0, hence it suffices to
prove that
x + 1 y + 1 z + 1 ≤ 2, 2
x + x + 1 + y2 + y + 1 + z2 + z + 1 or equivalently x2 y2 z2
x2 + x + 1 + y2 + y + 1 + z2 + z + 1 ≥ 1. Sincexyz = 1, there exist positive rela
numbersm,n,p such thatx = m2 , y = pm , z =np n2 mn . The last inequality now
becomesp2
m4 n4 p4 m4 + m2 np + n2 p2 + n4 + n2 pm + p2 m2 + p4 + p2 mn + m2 n2 ≥ 1. On the
other hand, by the Cauchy-Schwarz Inequality , we have
4 n4 p4m + n4 + n2 pm + p2 m2 + p4 + p2 mn + m2 n2 m4 + m2 np + n2 p2

(
m
2 + n2 + p2 )2 ≥ m4 + n4 + p4 + m2 n2 + n2 p2 + p2 m2 + mnp(m + n + p), and therefore
our inequality follows according to the fact that

44
(m2 + n2 + p2 )2 ≥ m4 + n4 + p4 + m2 n2 + n2 p2 + p2 m2 + mnp(m + n + p),
which obviously holds sincem2 n2 + n2 p2 + p2 m2 ≥ mnp(m + n + p).
Problem 47 Let a, b, c be positive numbers such that 4abc = a + b + c + 1. Prove that
2 + c2 c2 + a2 a2 + b2b + + ≥ 2(ab + bc + ca).
b c a
(Andrei Ciupan, Math. Reflections)
Proof. The inequality in question is equivalent to
2 + c2 c2 + a2 a2 + b2b + + + a + b + c ≥ a + b + c + 2(ab + bc + ca),
b c a
or (a2 + b2 + c2)(ab + bc + ca) ≥ a + b + c + 2(ab + bc + ca).
abc
From Problem 41 , we have
ab + bc + ca = 1 + 1 + 1
a + b + c ≥ 3 and ≥ 3,abc a b c
and thus, it suffices to prove that
3(a2 + b2 + c2 ) ≥ a + b + c + 2(ab + bc + ca),
which can be rewritten as
(a − b)2 + (b − c)2 + (c − a)2 + a2 + b2 + c2 ≥ a + b + c.
This is obviously valid, since
1
a2 + b2 + c2 ≥ 3(a + b + c)2 ≥ a + b + c.
Note that the equality holds if and only ifa = b = c = 1.
Problem 48 Let a, b, c be positive real numbers. Prove that
a3 b3 c3 3. (a + b)3 + (b + c)3 + (c + a)3 ≥ 8
(Vietnamese IMO Team Selection Tests 2005)
First proof. By Cauchy-Schwarz Inequality , we get
a
2
3 a 2
a(a + b) ≥a + b ,
cyc
(
a
+
b
)
3
cyc cyc
and since a(a + b) = a2 + ab ≤ 2 a2 ,
cyc cyc cyc cyc
it suffices to prove that
a2 2 3a2,
cyc a + b ≥ 4 cyc
or equivalently,22a ≥ 3 a2.
cyc a + b cyc
Since 2a a + b2 a2 − b2 a2 + b2
2 2

45
+
b
= a + b+ a + b= a + b+ (a − b)
cyc cyc cyc cyc cyc
a
2 + b2
= b ,
a + cyc
note that the previous inequality rewrites as
2 + b2 a
≥ 3 a2,
cyc a + b cyc
or equivalently a2 + b2 a + b≥ 3 a2 − a,
cyc a + b − 2 cyc cyc
i.e. (a − b)2 (a − b)2
cyc .
cyc
2(
a
+
b
)

3 a2 + a
cyc cyc
From the Cauchy-Schwarz Inequality , we have
3 a2 ≥ a,
cyc cyc
hence it is enough to show that
(
a

b
)
2 ≥ cyc a , 2(a + b) 2
2 (a − b) cyc
cyc
which is obviously valid since it can be written as 2 c(a − b) ≥ 0. cyc (a + b)(a + b + c)
Second proof. By the Cauchy-Schwarz Inequality , we get
a
3 2
c3 (a + b)3 ≥ c3/2 a3/2 ,
cyc
(
a
+
b

46
)
3
cyc cyc
and therefore we are left to prove that
2
8 c3/2 a3/2 ≥ 3 c3 (a + b)3 .
cyc cyc
We continue by setting the substitutionsx = √ ab, y = √ bc, z = √ ca. In this case, the
inequality to prove becomes
8(x3 + y3 + z3 )2 ≥ 3(x2 + y2 )3 + 3(y2 + z2 )3 + 3(z2 + x2 )3 ,
or equivalently,
(x6 + y6 ) − 9 x2 y2 (x2 + y2 ) + 16 x3 y3 ≥ 0,
cyc cyc cyc
which is valid because
x6 + y6 − 9x2 y2 (x2 + y2 ) + 16x3 y3 = (x2 + 4xy + y2 )(x − y)4 ≥ 0.
Third proof. Using the Power Mean Inequality , we get
1
a
3
1
/
3
1
1 a2 2 3 ≥ 3 cyc (a + b)2 , 3 cyc (a + b)
hence we must show that
a2 b2 c2 3. (a + b)2 + (b + c)2 + (c + a)2 ≥ 4

Consider the substitutionsx =b , y = c , z = a . Thenxyz = 1, and thus the previousa b c


inequality becomes 1 + 1 + 1 ≥ 3. (1 + x)2 (1 + y)2 (1 + z)2 4
Since
1 + 1 − 1 = xy(x − y)2 + (xy − 1)2 (1 + x)2 (1 + y)2 1 + xy (1 + xy)(1 + x)2 (1 + y)2
≥ 0,
we have that 1 + 1 ≥ 1 = z , (1 + x)2 (1 + y)2 1 + xy z + 1
and therefore
1
+
1
+
1
z
≥ z + 1 +1 = (z − 1)2 + 3 ≥ 3. (1 + x)2 (1 + y)2 (1 + z)2 (1 + z)2 4(z + 1)2 4
4
Fourth proof. Like in the third proof , we reach the point where we must show that
a2 b2 c2 3. (a + b)2 + (b + c)2 + (c + a)2 ≥ 4
This time we proceed as follows: from the Cauchy-Schwarz Inequality , we have
a2 22 = 1 (a + b)2 ,

cyc

47
(
a
+
b
)
2 2
2 (a + b) (a + c) ≥ a(a + b)
cyc cyc 4 cyc
and therefore, we are left to prove that
2
(a + b)2 ≥ 3 (a + b)2 (a + c)2 ,
cyc cyc
which is immediate, according to the AM-GM Inequality .
Problem 49 Let a, b, c, x, y, z be positive real numbers. Prove that
(a2 + x2 )(b2 + y2 )(c2 + z2 ) ≥ (ayz + bzx + cxy − xyz)2 .
(Titu Andreescu, Math. Reflections)
Proof. Consider the substitutionsm = a , n = b , p = c . Then, the inequality is
equivalentx y z
to 2a2 b2 c2 a
+b +c − 1 ,x2 + 1y2 + 1z2 + 1 ≥x y z
or
(m2 + 1)(n2 + 1)(p2 + 1) ≥ (m + n + p − 1)2 . Ifm + n + p ≥ mn + np + pm, then
(m + n + p − 1)2 = 1 + m2 + n2 + p2 + 2(mn + np + pm − m − n − p) ≤ 1 + m2 + n2 + p2
,
and since (m2 + 1)(n2 + 1)(p2 + 1) ≥ 1 + m2 + n2 + p2 ,
the inequality follows.
Alternatively, ifmn + np + pm ≥ m + n + p, then by the AM-GM Inequality , we have
(
m
+
n
+
p
)
2
m + n + p ≤ mn + np + pm ≤3 ,
and som + n + p ≥ 3. Without loss of generality, assume now thatm ≥ n ≥ p (note that
nowm ≥ 1). Furthermore, putt = n+p , and therefore we get2
(n2 + 1)(p2 + 1) ≥ 1 + n2 + p2 ≥ 1 + 2t2 .
It suffices to prove that
(1 + m2 )(1 + 2t2 ) ≥ (m + 2t − 1)2 , which is true, because it can be rewritten as
(m − 1) t2 m + (t − 1)2 + 1 ≥ 0.
Problem 50 Let x, y, z be positive real numbers. Prove that

y

48
+
z
+
√ √ 4 (x + y + z)z + x
+ x + y≥ (y + z) (z + x) (x + y).x y z
(Darij Grinberg)
Proof. According to the H¨older Inequality , √y + z √ √
+
z + x+ x + y2 x y z (y + z)2 x2 + (z + x)2 y2 + (x + y)2 z2 ≥ 8(x+y+z)3 . Thus, it is enough
to prove that 4(x + y + z)3 16(x + y + z)2 x2 y2 + y2 z2 + z2 x2 + x2 yz + xy2 z + xyz2 ≥
(x + y)(y + z)(z + x), which reduces to
x x2 y + xy2 + 2xyz ≥ 4 x2 y2 + 4 x2 yz,
cyc cyc cyc cyc cyc
and after expanding rewrites as
xy(x2 + y2 ) ≥ 2 x2 y2 .
cyc cyc
The last inequality follows from the cyclic multiplication ofx2 +y2 ≥ 2xy,y2 +z2 ≥ 2yz,
andz2 + x2 ≥ 2zx.
Problem 51 Let a, b, c be nonnegative real numbers such that abc = 4 and a, b, c > 1.
Prove that √ 4 (a − 1)(b − 1)(c − 1) a + b + c − 1 ≤3 4 − 1 .3
(Marian Tetiva, Math. Reflections)
Proof. Seta = x + 1, b = y + 1, c = z + 1; we then havex, y, z ≥ 0, and the first
condition becomes
3 = x + y + z + xy + yz + zx + xyz. The inequality in question now rewrites as
P
=
xyz
(
x
+
y
+
z
)

3
3
√ 4 4 − 1 .
Since 3 = x + y + z + xy + yz + zx + xyz, we have that
√ 2 √
3

3 = 1 −
23 4 − 1 (x + y + z) +4 − 1 (x + y + z) + xyz
3 3
+xy + yz + zx

49
√ √4 − 1 3xyz(x + y + z) 3(xy + yz + zx) + 23
3

1

4 − 1 23
3
+xy + yz + zx
√ √4 − 1 3xyz(x + y + z) 27xyz(x + y + z) + 23
3

1

4

1
2 3
4

3 + 3xyz(x + y + z)

3
√4 − 1 √
3P +√ 3P ,3 27P + +23
=
1

4

1
2 √
4

3
and therefore,

3
4 − 141 −
√ 2 √ 27P + 23 4 − 1 √ 3P +√ 3P ≤ 3.
3 3
3
We now conclude that√ 4 P ≤ 3 4 − 1 .

Note that the equality holds if and only ifa = b = c =3 4.
Problem 52 Let a, b, c be positive real numbers satisfying abc = 1. Prove that
1 ) +1 1

c(b + c) + a(c + a) ≥ 3. b(a + b 2


(Kazakhstan, Zhautykov Olympiad 2008)
Proof. Puta = x/y,b = y/z,c = z/x, wherex,y,z become now arbitrary positive real
numbers. In this case, according to the Cauchy-Schwarz Inequality ,
1 ) + c(b + c) + a(c + a)= z2 + xy + y2 z21 1 x2

50
b(a + b x2 + yz + y2 + zx
=
x y z z x + x2 y + x2 y + y2 z + y2 z + z2 x
3 3 3 2

x
3 + y 3 + z 3 2

2 2 2
≥ 2 (x2 y + y2 z + z2 x).
On other hand, since from the AM-GM Inequality we have that
x
3
+
x
3 3
y + x3 y 3
2 2 2 2 ≥ 3x2 y (and the other two similar relations),

we get
x
3
+
x
3 y 3 + x3 y 3 = x3 + y 3 + z 3 2
2 2 2 2 2 2 2
≥ 3 x2 y + y2 z + z2 x ,
cyc
and therefore1 ) + 1 1 c(b + c) + a(c + a) ≥ 3. b(a + b 2
Remarks. The inequality in question is sharpening a problem from the jBMO 2002:
Prove that for all positive real numbers a, b, c the following inequality takes place: 1) +
1 1 27
c(b + c) + a(c + a) ≥ 2(a + b + c)2. b(a + b
Problem 53 Prove that for all positive real numbers a, b, c the following inequality
holds: 1 1 + 1 1 1 1 c + a + a + b≥ ab + bc + ca + 2(a2 + b2 + c2 ). a + b + c b + c
(Pham Huu Duc, Math. Reflections)
First proof. The inequality is equivalent to
a(b + c) + bc+ b(c + a) + ca+ c(a + b) + ab≥ a +b +c + (a + b + c)(ab + bc + ca),b + c c + a a
+ b 2(a2 + b2 + c2)
or
ab + bc + ca ≥ (a + b + c)(ab + bc + ca).a + b b + c c + a 2(a2 + b2 + c2)
This can be rewritten as
2ab(a + b + c) + 2bc(a + b + c) + 2ca(a + b + c)≥ (a + b + c)2 (ab + bc + ca),a + b b + c c +
a a2 + b2 + c2
or equivalently,
2(ab + bc + ca) + 2abca + b +b + c +c + a ≥ ab + bc + ca + 2(ab + bc + ca)21 1 1 ,a2 + b2 + c2
i.e. 1 )2
+ b + c + c + a ≥ 2(ab + bc + ca ab
+

51
bc
+
ca
+ 2
abc
1 1
a + b a2 + b2 + c2 .
From the Cauchy-Schwarz Inequality , we have
1 + 1 1 9 b + c + c + a ≥ 2(a + b + c), a + b
hence it suffices to prove that
ab + bc + ca +a + b + c ≥ 2(ab + bc + ca)29abc ,a2 + b2 + c2
or (ab + bc + ca) 2(ab + bc + ca) − a2 − b2 − c2 9abc
≥ a2 + b2 + c2 .a + b + c
If 2(ab+bc+ca) ≤ a +b +c , we are done. Alternatively, if 2(ab+bc+ca) ≥ a2 +b2 +c2 ,
2 2 2

following the Schur Inequality , we get


9abc ≥ 2(ab + bc + ca) − a2 − b2 − c2 ,
a + b + c
and therefore, the problem reduces to proving that
2(
ab
+
bc
+
ca
)

a
2 2 2 2 2 2 ,
2 − b2 − c2 ≥ (ab + bc + ca) 2(ab + bc + ca) − a − b − c a + b + c which is
obviously true, because of the well-knowna2 + b2 + c2 ≥ ab + bc + ca.
Second proof. As in the first proof , we reach the point where we are left to show that
ab + bc + ca ≥ (a + b + c)(ab + bc + ca).a + b b + c c + a 2(a2 + b2 + c2)
This time, from the Cauchy-Schwarz Inequality , we have
ab + bc + ca ≥ (ab + bc + ca)2
ab(a + b) + bc(b + c) + ca(c + a), a + b b + c c + a
and so, it suffices to prove that
2(a2 + b2 + c2 )(ab + bc + ca) ≥ (a + b + c) [ab(a + b) + bc(b + c) + ca(c + a)] ,
which is valid, since it can be written as
ab(a − b)2 + bc(b − c)2 + ca(c − a)2 ≥ 0.
Problem 54 Let a, b, c be the sidelengths of a triangle. Prove that
√ c + a − b √ a +


√b + c − a + √ c +√ a −√b + √ a +√ b − c ≤ 3. b +√ c −√a b −√c

(IMO Shortlist 2006, proposed by South Korea)


Proof. (by Valentin Vornicu) Sincea,b,c are the sidelengths of a triangle, the

52
numbers x = √ a, y = √ b, z = √ c are also the sidelengths of a triangle (note that the
numbers −x + y + z,x − y + z andx + y − z are positive). In this case, according to the
CauchySchwarz Inequality ,
−x2 + y2 + z2 2 −x2 + y2 + z2 −
x
+
y
+
z
≤ 3 (−x + y + z)2,
cyc cyc
and thus, it is suffice to prove that −x2 + y2 + z2
(

x
+
y
+
z
)
2 ≤ 3.
cyc
Moving all into one side and distributing the 3 to each of the fractions equally, the last
inequality rewrites as
0 ≤1x2 + yz − xy − xz =(−x + y + z)2 (x − y)(x − z),
cyc
(

x
+
y
+
z
)
2
cyc
which is valid, according to the generalized Schur Inequality , because ifx ≥ y ≥ z
then 1 ≥ 1 1
(x − y + z)2 ≥ (x + y − z)2. (−x + y + z)2
Problem 55 Let a, b, c be the sidelengths of a triangle with perimeter 1. Prove that
1 < √c ab + √b + c2 + √c + a2 < 2.a + b2
(Vo Quoc Ba Can)
Proof. We will first prove the left hand side of the inequality. Sincea, b, c are
contained in the interval (0, 1), we have that

53
a + b2 < a + b < a + b + c = 1
b + c2 < b + c < a + b + c = 1
c + a2 < c + a < a + b + c = 1,
and thus
√c ab + √b + c2 + √c + a2 > b + c + a = 1.a + b2
In order to prove the right hand side of the inequality in question, note that the
inequality
√ 2bb < a + b + ca + b2
is valid, because it can be rewritten as
3a2 + 2a(b + c) + 3b2 − 2bc − c2 > 0.
Similarly, have that c 2c a 2a√ b + c2< a + b + c,√ c + a2< a + b + c, and therefore, we
conclude that
√ c a 2b 2c 2ab + √ b + c2 + √ c + a2< a + b + c + a + b + c + a + b + c = 2. a + b2
Problem 56 Prove that for any positive real numbers a, b and c, we have that
b + c+ c + a+ a + b≥ 6 · a + b + c
a
b
c

3 abc .
(Pham Huu Duc, Math. Reflections)
Proof. The inequality is equivalent to b + c+ 2 (a + b)(a + c)≥ 6(a + b + c)
√ abc . 3
cyc a cyc bc
From the Cauchy-Schwarz Inequality , we have
(a + b)(a + c)≥ a +√ bc= a + 1,bc √bc √bc
hence it suffices to prove that
b
+
c
+
c
+
a
+
a
+
b
a
+ 2

bc
+
b + c+ 6 ≥ 6(a + b + c) a

54
b
c

ca


3
ab abc .
By the Chebyshev Inequality , get
a b + c ≥ 3(a + b + c) √ + √ + √1 ≥a + b + c √ 1 1 1
bc ca
bc
+
√ca √ab ab √
3
abc ,
and thus we are left to show that
b
+
c
+
c
+
a
+
a
+
b
+ 6

4(
a
+ b + c)

3
a b c abc .
Furthermore, puta = x3 , b = y3 , c = z3 ; the inequality to prove becomes y3 + z3 z3 +
x3 x3 + y3 4(x3 + y3 + z3 ),x3 + y3 + z3 + 6 ≥ xyz
or equivalently,
yz(y3 + z3 )+ zx(z3 + x3 )+ xy(x3 + y3 )+ 6xyz ≥ 4(x3 + y3 + z3).x2 y2 z2
This can be rewritten into
xy
x
y
2 2 2
2z2 x y − xyz x ≥ x x − xy ,
cyc cyc cyc cyc cyc cyc cyc
or (

55
y

z
)
2 x2 xyx
y
2z2 − x − y − z ≥ 0,
cyc cyc cyc
which is true since
x
2 xyx y y

2
z
2

x
2
(xy + yz + zx)x2z2 + z x
2 y2 cyc cyc
=
x
(
y
+
z
)
y + z+ y2 z2 z2 y2 z+ y y
2 z2
≥ 2x +z +y
≥ 2x + y + z.
Remarks. In fact, we have the following refiniment, proposed later by Vo Quoc Ba
Can in the same magazine:
Let a, b, c be positive real numbers. Prove that
b + c+ c + a+ a + b≥ 16(a + b + c)3 3(a + b)(b + c)(c + a). a b c
Proof. From the H¨older Inequality , we have
2
1≥ 13 b + c, cyc acyc a2 (b + c)cyc a
hence it suffices to prove that
13 16(a + b + c)3 1. cyc a≥ 3(a + b)(b + c)(c + a) cyc a2 (b + c)
Seta = 1 , b = 1 , c = 1 ; the inequality becomesx y z
(
x
+
y
+

56
z
)
3
3 ≥ 16(xy + yz + zx) x
3(x + y)(y + z)(z + x) y + z + y + z . z + x x + y We know that
8
(x + y)(y + z)(z + x) ≥ 9(x + y + z)(xy + yz + zx),
and therefore, we are left to show that
2
(x + y + z)3 ≥ 6(xy + yz + zx) x + y + z ,x + y + z y + z z + x x + y
or equivalently,
(x + y + z)4 2 + y2 + z2 ) + 6xyz 1 1 1
y + z +z + x +x + y .xy + yz + zx ≥ 6(x
By the Cauchy-Schwarz Inequality , we have
6
xyz
1 1 1
y
+
z
+
z
+
x
+
x
+
y

6
xyz
1 +1 +1 +1 +1 +1 4x 4y 4y 4z 4z 4x = 3(xy + yz + zx),
and so the problem reduces to proving
(x + y + z)4 2 + y2 + z2 ) + 3(xy + yz + zx),
xy + yz + zx ≥ 6(x
which is obviously true, because it can be rewritten into 2 + y2 + z2 − xy − yz − zx)2
(x≥ 0.xy + yz + zx
Problem 57 Let a, b, c be positive real numbers. Prove that
3
√ a + √ b + √ c ≥ √ 2. ab + b2 bc + c2 ca + a2 (Cosmin Pohoat¸˘a and Michael Rozenberg)

First proof. Consider the substitutionsa = x2 ,b = y2 ,c = z2 . By the H¨older


Inequality , we get
cyc y x
x2
2
+
y

57
2
cyc cyc
x + y xy ≥ (x + y + z)3 ,
2 2

and thus, it remains us to show that


2 2
2 (x + y + z)6 ≥ 9 xy x2 + y2 .
cyc cyc
On the other hand, from the Cauchy-Schwarz Inequality , we have x2 + y2 ≤ 6(x2 +
y2 + z2 ),
cyc
and therefore, the problem reduces to proving
(x + y + z)6 ≥ 27 (xy + yz + zx)2 x2 + y2 + z2 ,
which is true, according to the following identity:
(x + y + z)6 − 27(x2 + y2 + z2 )(xy + yz + zx)2
= (x2 + y2 + z2 + 8(xy + yz + zx))(x2 + y2 + z2 − xy − yz − zx)2 .
Second proof. This time, consider the substitutionsx = b , y = c , z = a ; thenxyz = 1a b c
and the inequality becomes
1
+ 1) +1 1 3
y (y + 1) + z(z + 1) ≥ √ 2. x(x
Furthermore, sincexyz = 1, we can putx =np , y = pm , z = mn , wherem, n, p arem2 n2 p2
positive real numbers. The inequality can be now rewritten as
m2 n2 p2 3 np(m2 + np) + pm(n2 + pm) + mn(p2 + mn) ≥ √ 2. By the H¨older
Inequality , we get
2m2 2 np(m2 + np) ≥ m2 + n2 + p2 3 ,
cyc
np
(
m
2
+ np)m
cyc
and thus, it suffices to prove that
2 m2 + n2 + p2 3 ≥ 9mnp m3 + 3mnp .
cyc
This is equivalent with
2 m6 + 6 m4 (n2 + p2 ) − 9mnp m3 − 15m2 n2 p2 ≥ 0,
cyc cyc cyc
and since from the AM-GM Inequality , we have
2 m6 ≥ 6m2 n2 p2
cyc
6 m4 (n2 + p2 ) ≥ 12mnp m3
cyc cyc
3mnp m3 ≥ 9m2 n2 p2 ,

58
cyc
the conclusion follows immediately.
Problem 58 Let a1 ≤ a2 ≤ . . . ≤ an be positive real numbers such that
a
2 + a2 + . . . + a2

1 2 n , a1 + a2 + . . . + an = m,n = 1 n
where 1 ≥ m > 0. Prove that for all i satisfying ai ≤ m, we have
n − i ≥ n(m − ai )2 .
(Iranian IMO Team Selection Test 2005)
First proof. By the Cauchy-Schwarz Inequality , we have
a1 + a2 + . . . + ai + ai+1 + ai+2 + . . . + anm = n
=
a1 + a2 + . . . + ai + ai+1 + ai+2 + . . . + an n n
a
1
+
a
2
+
. . .
+
a
i
(n − i)(a2
+
i
+1
+ a2 + . . . + a2 ) ≤
i+2 n
n n
(n − i)(a2 + a2 + . . . + a2 )

a
i
+
i+1 i+2 n
n
(n − i)(n − a2 − a2 − . . . − a2 )= ai + 1 2 i
n
≤ a n(n − i),i + n
and therefore
m

59

a
i

n
− i
n .
This means thatn − i ≥ n(m − ai )2 , which yields our conclusion.
Second proof. Letbk = m − ak for allk = 1, 2, . . . , n; we then have that
n n
m ≥ b1 ≥ b2 ≥ . . . ≥ bn , bk = 0, b2 = n(1 − m2 ).k
k=1 k=1
Sinceai ≤ m, note thatm ≥ b1 ≥ b2 ≥ . . . ≥ bi ≥ 0, and thus we are left to show that n
− i
b
2

i n .
By the Cauchy-Schwarz Inequality , we get
b
2 1 (b + b + . . . + b )2 , + b2
1 2 i 1
2
2 i + . . . + b ≥i
and
b
2 + b2 + . . . + b2 ≥1
n

i
(
b
i
+1
+
b
i
+2
+ . . . + b
n
)
2 i+1 i+2 n
=
1
2
n − i (−b1 − b2 − . . . − bi )
=
1

60
2
n − i (b1 + b2 + . . . + bi ) .
Summing up these two inequalities, it follows that
n
(1

m
2 ) ≥1 + 2 1 2 2 nib21 1 i i ii n − i, and therefore
n − i (b1 + b2 + . . . + bi ) ≥ +n − i · i b =
2
b2 (n − i)(1 − m ).i ≤ i
On the other hand, we also haveb2 ≤ m2 , and so we can write thati
2
b2 ≤ min m2, (n − i)(1 − m ).i i
Note that this proves our inequality, since (it is easy to check that) (
n

i
)(1

m
2 )≤ n − i
min m2 ,i n .
Problem 59 Let x, y, z be positive real numbers. Prove that
3 √x √y √z√3 ≤ √x + y + z ·
y + z +z + x +x + y .2
(Byron Schmuland, Math. Reflections)
First proof. Since the inequality in question is homogenous, we can assume (without
loss of generality) thatx + y + z = 1; then the inequality becomes
√x √
+ 1 − y + 1 − z ≥ 3√3y √z . 1 − x 2
By the AM-GM Inequality , we have
x(1 − x)2 = 1 · 2x · (1 − x) · (1 − x) ≤ 1 2x + (1 − x) + (1 − x) 3 4 , 2 2 3= 27 hence√ 2 x(1 −
x) ≤3 √3 . Thus √ y√ z x y z 1 − x
√x + 1 − y + 1 − z= √ x(1 − x) +√ y(1 − y) +√ z(1 − z) 3
√3 + 3√3 + 3√3 = 3√3

≥2 x2 y2 z2 .
Second proof. By H¨older Inequality , we get
√x 2 4
x
(
y
+
z
)
y

61
+
z
cyc cyc
x2 (y + z) ≥ x ,
cyc cyc
hence it suffices to prove that
2(x + y + z)5 ≥ 27(xy + yz + zx) x2 (y + z) + y2 (z + x) + z2 (x + y) .
From the Schur Inequality , we have
x3 + y3 + z3 + 3xyz ≥ x2 (y + z) + y2 (z + x) + z2 (x + y),
and thus
(x + y + z)(x2 + y2 + z2 + xy + yz + zx) ≥ 3 x2 (y + z).
cyc
In this case, the inequality reduces to proving that
2(x + y + z)4 ≥ 9(xy + yz + zx)(x2 + y2 + z2 + xy + yz + zx),
but this rewrites as
(x2 + y2 + z2 − xy − yz − zx)(2x2 + 2y2 + 2z2 + xy + yz + zx) ≥ 0,
which is valid, according to the well-knownx2 + y2 + z2 ≥ xy + yz + zx.
Problem 60 Let a, b, c be positive real numbers. Prove that
√ b ca + √ 2
b + 2ca + √ c2 + 2ab ≤ a + b + c ca.a2 + 2bc√ ab + bc +
(Ho Phu Thai, Math. Reflections)
Proof. By the Cauchy-Schwarz Inequality , we have
√ a b c2 a + b2 + 2ca + c2 + 2ab ,
cyc a2 + 2bc ≤ (a + b + c)a2 + 2bc
hence it suffices to prove that a + b2 + 2ca + c2 + 2ab ≤ a + b + cb c
a2 + 2bc ab + bc + ca. This is equivalent with
a a⇔ ab + bc + ca − a2 + 2bc≥ 0,
cyc
or 1 a(a − b)(a − c) ≥ 0.
ab + bc + ca cyc a2 + 2bc Without loss of generality, assume
now thata ≥ b ≥ c. In this case, it follows that
a(a − b)(a − c)≥ a(a − b)(a − c)+ b(b − a)(b − c) a2 + 2bc a2 + 2bc b2 + 2cacyc
2 2 2
= c(a − b) (2a + 2b − 2ac − 2bc + 3ab)≥ 0.(a2 + 2bc)(b2 + 2ca)
Problem 61 Let a, b, c be distinct positive real numbers. Prove the following
inequality: a2 b + a2 c + b2 a + b2 c + c2 a + c2 b − 6abc≥ 16abc (a + b + c)2. a2 + b2 +
c2 − ab − bc − ca
(Iurie Boreico and Ivan Borsenco, Math. Reflections)
Proof. The inequality is equivalent to
a
2 b + a2 c + b2 a + b2 c + c2 a + c2 b− 6abc ≥16abc
2 2 2
(a + b + c)2 a + b + c − ab − bc − ca .
Since
a2 b + a2 c + b2 a + b2 c + c2 a + c2 b − 6abc = (b + c)(a − b)(a − c)
cyc

62
and
a2 + b2 + c2 − ab − bc − ca = (a − b)(a − c) + (b − c)(b − a) + (c − a)(c − b),
the inequality in question can be rewritten into 16abc (a − b)(a − c) b + c −(a + b + c)2 ≥
0.
cyc
Now, note that
b
+
c

(
a
+
b
+
c
)
2

b
+
c

(
a
+
b
+
c
)
2
= (
b + c)(a − b − c)2 16abc 4a(b + c)2
(a + b + c)2 ≥ 0, and moreover, ifa ≥ b ≥ c, then
b + c − 16abc 16abc16abc ≤ c + a −
(a + b + c)2 ≤ a + b − (a + b + c)2. (a + b + c)2
Thus, the generalized Schur Inequality yields 16abc (a − b)(a − c) b + c −(a + b + c)2 ≥
0.
cyc
Problem 62 Let a, b, c be nonzero positive real numbers. Prove that a3 + abc+ b3 +
abc+ c3 + abc≥ a(b3 + c3 )+ b(c3 + a3 )+ c(a3 + b3) b + c c + a a + b a2 + bc b2 + ca c2 +
ab . (Pham Huu Duc and Cosmin Pohoat¸˘a)
Proof. The inequality is equivalent to
a3 + abc − a2 ≥a(b3 + c3) − (b2 − bc + c2 ) + a2 − ab,

cyc b + c cyc a2 + bc cyc cyc

63
or a b2 − bc + c2
+ cyc b + c a2 + bc − 1 (a − b)(a − c) ≥ 0.
This rewrites as a(a − b)(a − c) + (b − c)2 (b + c) (a − b)(a − c)≥ 0, cyc (b + c)(a2 + bc)
and furthermore as a(a − b)2 (a − c)2 (b − c)2 (a − b)(a − c)≥ 0, cyc (b + c)(a2 + bc)+ a2 +
bccyc
or equivalently,
a (a − b)2 (a − c)2 (a − b)2 (b − c)2 (c − a)2 (a2 + b2 + c2 )≥ 0. cyc (b + c)(a2 + bc) −
(a2 + bc)(b2 + ca)(c2 + ab) Without loss of generality, we may now assume thata ≥ b ≥
c. Thus, it is suffice to prove that
a(a − b)2 (a − c)2 b(b − a)2 (b − c)2 (a − b)2 (b − c)2 (c − a)2 (a2 + b2 + c2 )≥ 0,(b + c)(a2 +
bc) + (c + a)(b2 + ca) − (a2 + bc)(b2 + ca)(c2 + ab)
i.e. a(b2 + ca) ) + (a − c)(a2 − c2 ) −a2 + b2 + c2b(a2 + bc) ≥ 0. (b − c)(b2 − c2 c2 + ab
Note that this last inequality is valid, because
a(b2 + ca)) + (a − c)(a2 − c2) − a2 + b2 + c2 b(a2 + bc)
(b − c)(b2 − c2 c2 + ab ≥
a(b2 + ca)+ b(a2 + bc)− a2 + b2 + c2
b3 a3 c2 + ab
=
a2 + b2 c(a5 + b5 )− a2 + b2 + c2
ab+ a3 b3 c2 + ab

a2 + b2 a2 + b2 + c2
ab− c2 + ab
2 2 2
= c (a − ab + b )≥ 0.ab(ab + c2)
The equality holds if and only ifa = b = c orabc = 0.
Remarks. Note that this inequality is sharpening the well-known a3 + abc+ b3 + abc+
c3 + abc≥ a2 + b2 + c2.b + c c + a a + b
However, the inequality
a(b3 + c3 )+ b(c3 + a3 )+ c(a3 + b3 )≥ a2 + b2 + c2,a2 + bc b2 + ca c2 + ab
which holds for any positive real numbersa,b,c is not trivial. We leave it as an exercise
for the reader.
Problem 63 Let a, b, c, d be real numbers with sum 0. Prove the inequality:
(ab + ac + ad + bc + bd + cd)2 + 12 ≥ 6(abc + abd + acd + bcd).
(Kazakhstan, Zhautykov Olympiad 2006)
First proof. Sinced = −a − b − c, our inequality becomes
(a2 + b2 + c2 + ab + bc + ca)2 + 12 + 6(a + b + c)(ab + bc + ca) ≥ 6abc, or equivalently,
(a2 + b2 + c2 + ab + bc + ca)2 + 12 + 6(a + b)(b + c)(c + a) ≥ 0.
This rewrites as
1 (a + b)2 + (b + c)2 + (c + a)2 2 + 12 + 6(a + b)(b + c)(c + a) ≥ 0.
4
Setx = b+c y = c+a ,z = a+b ; then the previous inequality goes into2 , 2 2

64
(x2 + y2 + z2 )2 + 48 + 24xyz ≥ 0.
Now, since the AM-GM Inequality , we have
(x2 + y2 + z2 )2 ≥ 9 |xyz|4/3
and because 24xyz ≥ −24 |xyz| , it suffices to prove that
wheret = |xyz| 9t4 + 48 − 24t3 ≥ 0, 1/3 . This is true, since
9t4 + 48 − 24t3 = 3(3t2 + 4t + 4)(t − 2)2 ≥ 0.
Second proof. From the Rolle Theorem, we know that there exist real numbersx, y, z
such that☐
☐ x + y + z = 3 (a + b + c + d) = 0
☐ 4
ab + ac + ad + bc + bd + cd = 2(xy + yz + zx)

☐ abc + abd + acd + bcd = 4xyz.
Therefore, the inequality in question is equivalent to
(xy + yz + zx)2 + 3 ≥ 6xyz.
Without loss of generality, assume now thatz = min {x, y, z}; then sincex + y + z = 0,
we havez = −x − y ≤ 0, and so our inequality becomes
(x2 + xy + y2 )2 + 3 + 6xy(x + y) ≥ 0.
Let us now puts = x + y ≥ 0, p = xy. In this case, the previous inequality rewrites as
(s2 − p)2 + 3 + 6sp ≥ 0,
or f (p) = p2 + 2(3s − s2 )p + s4 + 3 ≥ 0. We have now that
∆f = (3s − s2 )2 − s4 − 3 = −3(2s + 1)(s − 1)2 ≤ 0,
and we thus conclude that f (p) ≥ 0.
Problem 64 Let a, b, c be positive real numbers satisfying a + b + c = 1. Prove that
1 2 2 8(a2 + b2 + c2)22 1 − 2 + 1 − 2 ≥
(1 − a)(1 − b)(1 − c). a − 2 + b c
(Vo Quoc Ba Can, Math. and Youth magazine)
First proof. The Cauchy-Schwarz Inequality yields
1 2 22 1 − 2 +1 − 2 [a2 (b + c)2 + b2 (c + a)2 + c2 (a + b)2 ] = − 2 +
a b c
(
b
+
c

a
)
2 + (c + a − b)2 + (a + b − c)2

=2 (b + c)2 + b2 (c + a)2 + c2 (a + b)2 ]a2 b2 c2 [a


≥ [(b + c − a)(b + c) + (c + a − b)(c + a) + (a + b − c)(a + b)]2 = 4(a2 + b2 + c2 )2 .
We thus need to prove that
(1 − a)(1 − b)(1 − c) ≥ 2[a2 (b + c)2 + b2 (c + a)2 + c2 (a + b)2 ],
or equivalently,
(a + b)(b + c)(c + a)(a + b + c) ≥ 2[a2 (b + c)2 + b2 (c + a)2 + c2 (a + b)2 ].
This is obviously true, since

65
ab(a − b)2 + bc(b − c)2 + ca(c − a)2 ≥ 0.
Note that the equality holds if and only ifa = b = c = 1 .3
Second proof. Setx = a2 + b2 + c2 . It is easy to see that 1 ≤ x ≤ 1, and thus then3
(x − 1)(3x − 1) ≤ 0, from which we get 4x − 1 ≥ 3x2 . Moreover, by the AM-GM
Inequality ,
b2 c ≥ abc a = abc
cyc cyc
hence
(4x − 1) b2 c2 ≥ 3abcx2 .
cyc
Furthermore 4x − 1 = (b + c − a)2 ,
cyc
and therefore
(b + c − a)2 b2 c2 ≥ 3abcx2 .
cyc cyc
Without loss of generality, we now assume thata ≥ b ≥ c. Since
2 c2 ≤ c2 a2 ≤ a2 b2 b

and (b + c − a)2 ≤ (c + a − b)2 ≤ (a + b − c)2 , following the Chebyshev Inequality ,


we get
(b + c − a)2 b2 c2 ≤ 3 b2 c2 (b + c − a)2
cyc cyc cyc
= 3 b2 c2 (1 − 2a)2
cyc
=
3
a
2
b
2
c
2
1
2
− 2 ,
cyc a
and so 1
2

2

x
2 8x2 8x2 ≥ (a + b)(b + c)(c + a) = (1 − a)(1 − b)(1 − c). a abc
cyc

66
Problem 65 Let x1 , x2 , . . . , xn be real numbers from the interval [0, 1] satisfying
x1 x2 . . . xn = (1 − x1 )2 (1 − x2 )2 . . . (1 − xn )2 .
Find the maximum value of x1 x2 . . . xn . (Chendi Huang)
Proof. Set
S
=
n

x1 + x2 + . . . + xn and T = √x1x2. . . xn.n


We have 0 ≤ T ≤ S ≤ 1 and
Tn = x1 x2 . . . xn = (1 − x1 )2 (1 − x2 )2 . . . (1 − xn )2
=
(1 − x1 ) + (1 − x2 ) + . . . + (1 − xn )2n n = (1 − S)2n ≤ (1 − T )2n . ThusT ≤ (1 − T )2 ,
from which (together with the fact that 0 ≤ T ≤ 1) we get

5 − 1
T ≤2 .
Since the equality occurs when the numbersx1 , x2 , . . . , xn are all equal, we get that √
max x5 − 1 .1 x2 . . . xn = max Tn =2
Problem 66 Let a, b, c be three positive real numbers with sum 3. Prove that
1 + 1 + 1 ≥ a2 + b2 + c2 .
a2 b2 c2
(Romanian IMO Team Selection Tests 2006)
First proof. From the AM-GM Inequality , we get
1 + 1 + 1 1 + 1 + 13

a
2
b2 c2 ab bc ca
= abc,
hence it suffices to prove that
3 ≥ abc(a2 + b2 + c2 ).
Again, the AM-GM Inequality gives us that
(ab + bc + ca)2 ≥ 3abc(a + b + c) = 9abc,
and therefore, we are left to show that
(a2 + b2 + c2 )(ab + bc + ca)2 ≤ 27,
which is obviously valid, since
(
a
2 2 2 3
2 + b2 + c2)(ab + bc + ca)2 ≤ a + b + c + 2(ab + bc + ca) 3
(
a
+
b
+
c

67
)
6
=27 = 27.
Second proof. Rewrite the inequality as
a2 b2 ≥ a2 b2 c2 a2 , i.e. a2 b2 (1 + c + c2 + c3 )(1 − c) ≥ 0.
cyc cyc cyc
Note now that ifab ≤ 2 anda ≥ b then
a2 (1 + b + b2 + b3 ) ≥ b2 (1 + a + a2 + a3 ).
Thus, we conclude that ifab, bc, ca are all smaller than 2, then the Chebyshev
Inequality yields
a2 b2 (1 + c + c2 + c3 )(1 − c) ≥ a2 b2 (1 + c + c2 + c3 ) (1 − c) = 0,
cyc cyc cyc
and we are done. Otherwise, if (for example)ab ≥ 2, thena+b ≥ 2√ 2, and soc ≤ 3−2√ 2
andc2 ≤ 1/9. Therefore
1 + 1 + 1 > 9 > a2 + b2 + c2 .
2 2 a2 b c
Remarks. Actually, we can show more:
For any two positive integers n, p satisying, n ≥ 4 and p ≥ 4, the proposition P(n, p) is
false:
n 1 n n
≥ xip for xi ∈ R, xi > 0, i = 1, . . . , n , xi = n.
i=1 xip i=1 i=1
This variation was considered as a problem in the Romanian IMO Team Selection
Tests from 2007. We continue with its proof.
Proof. (by Dan Schwarz) Notice first that it is enough to find a set of valuesxi forn = 4
such thatn
E =
n 1
− xip < 0
i=1 xip i=1
as then for any n > 4 we can extend this set of values by taking the extran − 4
ones to be equal to 1.
Now that we have reduced it to the casen = 4, it makes sense to look for ”simple”
cases:
• somexi very small - it yieldsE > 0, no good;
• allxi equal - it yields common value 1, for whichE = 0, no good;
• let’s then try taking the smallest threexi equal to some value 0 < x < 1; the fourth one,
denoted byy, will be 1 < y < 4,y = 4 − 3x.
Then,
E = 3/xp + 1/yp − 3xp − yp = (1/xp )[3 + (x/y)p − 3x2p − (xy)p ] = (1/xp )[3 − (xy)p ] +
(1/yp )[1 − 3(xy)p ].
It seems natural now to look for the maximum possible value for xy; it is not
difficult to see thatxy = (1/3)(3x)(4 − 3x) ≤ (1/3)(2)2 = 4/3 (by the AM-GM
Inequality ), with equality for 3x = 4 − 3x i.e.x = 2/3 andy = 2. Then, as 4/3 > 1 andp

68
≥ 4, we have (4/3)p ≥ (4/3)4 = 256/81 > 3, henceE < 0 for the set of values (2/3, 2/3,
2/3, 2 and 1 repeatedn − 4 times).
You might now wonder what can we say about the propositions P(4, 3) and P(3, 4). As
a matter of fact, they are true. However, we will omit their proof here.
Problem 67 Let a, b, c be positive real numbers satisfying a + b + c = 3. Prove that
1
+ 1 +2c + 1 +2a + 1
a b c

2b abc.
(Vo Quoc Ba Can)
Proof. Firstly, we will show that
ab2 + bc2 + ca2 + abc ≤ 4.
Indeed, assume (without loss of generality) thatb is betweena andc; then
a(b − a)(b − c) ≤ 0,
and thus
ab2 + bc2 + ca2 + abc = b(a + c)2 + a(b − a)(b − c)
= b(3 − b)2 + a(b − a)(b − c)
= 4 + (b − 4)(b − 1)2 + a(b − a)(b − c) ≤ 4.
Returning to our problem, we see that the inequality in question is equivalent with
a2bc
≤ 1,
cyc 2b + 1
or
ca2 − 2a2bc ≥ ab2 − 2 cyc 2b + 1cyc This can be rewritten intoca2 2 − 2,
cyc
2
b
+ 1
≥ ab
cyc
and since from the Cauchy-Schwarz Inequality , we get
2 2
ca2 ab2
ca2 cyc + 1) =cyc abc, 2b + 1 ≥ ca2 (2b ab2 + 6
cyc
cyc cyc
it shall be enough to prove that
2
ab2 ≥ ab2 − 2 ab2 + 6abc ,
cyc cyc cyc
or equivalently,
6abc ≥ (3abc − 1) ab2
cyc
Notice that if 3abc ≤ 1, the inequality is trivial. Otherwise, if 3abc ≥ 1, by making use

69
of the fact that
ab2 ≤ 4 − abc, cyc
we conclude the problem reduces to proving that
6abc ≥ (3abc − 1)(ab2 + bc2 + ca2 ).
This is obviously true, since from the AM-GM Inequality , we have thatab2 + bc2 +
ca2 ≥ 3abc.
Problem 68 For any three positive numbers a, b, c, prove the inequality
(1 + abc) 1 11 ) + b (1 + c) + c (1 + a)≥ 3. a (1 + b

(BMO 2006)
Proof. (by Darij Grinberg) Note that in general we haveabc = 1, so we cannot
apply the substitutionsa = v ,b = w ,c = u with positive realsu,v,w. However, we can
substituteu v w
a = k · v ,b = k · w ,c = k · u , wherek,u,v,w are positive real numbers. Thenabc = k3 ,u v
w
and our inequality simplifies to
1 +
k
3 u) + v w
k (w + ku) + k (u + kv)≥ 3. k (v + kw
This can be rewritten as
1 + k3 u + w + ku + w≥ 3.k · v
v + kw u + kv Note that this is a generalization of the Nesbitt Inequality . For proving
it, we defineA = v + kw,B = w + ku,C = u + kv, and then we can readily find
C
+
k
2
u = B − kA,1 + k3
2 C − kB
v = A + k ,1 + k3
2
w = B + k A − kC.1 + k3
Thus, the inequality we are intending to prove rewrites now as
1 + k3 C + k2 B − kA+ A + k2 C − kB+ B + k2 A − kC≥ 3.k · A (1 + k3) B (1 + k3) C (1 + k3)
In other words, 1 · C + k2 B − kA+ A + k2 C − kB+ B + k2 A − kC≥ 3.k A B C
Multiply withk and get C + k2 B − kA+ A + k2 C − kB+ B + k2 A − kC≥ 3k,A B C
and so B + k2C + C + k2A + A + k2B − 3k ≥ 3k.
C B A C B A
Hence B + k2C + C + k2A + A + k2B ≥ 6k,
C B A C B A
which follows from the AM-GM Inequality , since
x + k2y ≥ 2k.
y x
Remarks. This inequality seems to have appeared in many different places
before. For example: USA IMO team training 1993, Armenian MO 1996, Indian IMO

70
Team Selection Tests 1997, Crux Mathematicorum 1998 (proposed by Mohammed
Aassila).
Problem 69 Let a, b, c, d be real numbers such that a2 + b2 + c2 + d2 = 1. Prove that 1
+ 1 1 1
1

bc
+
1

cd
+
1

da

16
1 − ab 3.
(Vo Quoc Ba Can)
First proof. The inequality is equivalent to
2

1 1 1 1 1

ab
+
2

1

bc
+
2

1

cd
+
2

1 − da
≥ 8
3
i.e. 1 − 2ab + 1 − 2bc + 1 − 2cd + 1 − 2da 8
.1 − ab 1 − bc 1 − cd 1 − da ≥ 3 Since 1 − 2ab

71
= a2 + b2 + c2 + d2 − 2ab ≥ a2 + b2 − 2ab = (a − b)2 ≥ 0, by the Cauchy-Schwarz
Inequality , we have
2
1 − 2ab (1 − 2ab)(1 − ab) ≥ (1 − 2ab)

cyc 1 − ab cyc cyc


= 4[2 − (a + c)(b + d)]2 ,
hence it suffices to prove that 3[2 − (a + c)(b + d)]2 ≥ 2 (1 − 2ab)(1 − ab),
cyc
or equivalently,
3[2 − (a + c)(b + d)]2 ≥ 2[4 − 3(a + c)(b + d) + 2(a2 + c2 )(b2 + d2 )].
This can be rewritten into
3(a + c)2 (b + d)2 − 6(a + c)(b + d) + 4 − 4(a2 + c2 )(b2 + d2 ) ≥ 0,
or in other words
3[1 − (a + c)(b + d)]2 + 1 − 4(a2 + c2 )(b2 + d2 ) ≥ 0,
which is valid, because the AM-GM Inequality gives
4(a + c2 )(b2 + d2 ) ≤ (a2 + b2 + c2 + d2 )2 = 1.
2

The equality holds if and only ifa = b = c = d = ±1 .2


Second proof. For all real numbersx smaller than 1 , we have2
12 + 10 = (4x − 1)2 (2x − 1)≤ 0,1 − x − 32x
9 9(1 − x)
and therefore1 ≤ 32x2 + 10.1 − x 9
According to this inequality, combined with the (obvious) fact that max {ab, bc, cd,
da} ≤ 1 , yields2
1 + 1 1 1 32
2 b 2 + b 2 c 2 + c 2 d 2 + d 2 a 2 ) + 40
1 − bc + 1 − cd + 1 − da ≤ 9 (a 1 − ab 9
32
= 2 + c 2 )(b 2 + d 2 ) + 409 (a
9 8
≤ 2 + b 2 + c 2 + d 2 ) 2 + 409(a
9
= 9 + 40 = 16 .
3
Problem 70 Let x1 , x2 , . . . , xn be positive real numbers such that x1 +x2 +. . .+xn = 1.
Prove that n √ n
x
i

1≤ n2

n + 1.
i=1 i=1 1 + xi

72
(Chinese TST 2006)
First proof. From the AM-GM Inequality , we get
√ n + 1 nn √ 2n n n
√ xi + √ 1 √ x
i

1

4
n
n
+ 1
i=1 i=1 1 + xi i=1 i=1 1 + xi
,
and thus it is enough to show that
n
√nn √ xi + √1 ≤ 2nn + 1 . i=1 n + 1 1 + xi
We shall now make use of the following auxiliary result:
Lemma . For any positive real numberx, the following inequality holds:
√n √ x + √1 ≤ n(n + 1) n2 x + 3n + 4n + 1 1 + x 2(n + 1)2
Proof . This rewrites as
√n √ x + √1 2 n (n2 x + 3n + 4)2 ,n + 1 1 + x ≤4(n + 1)3
and since from the Cauchy-Schwarz Inequality , we have
2 n 1 √n √ x + √1 ≤ (nx + 1) + 1 +
n x + 1n + 1 1 + x
= (nx + 1)(nx + 2n + 1),(n + 1)(x + 1)
the problem reduces to showing that
4(n + 1)2 (nx + 1)(nx + 2n + 1) ≤ n(x + 1)(n2 x + 3n + 4)2 .
On the other hand, the AM-GM Inequality gives us that
(n2 x + 3n + 4)2 ≥ 4(n2 x + n + 2)(2n + 2) = 8(n + 1)(n2 x + n + 2),
and therefore, we are left to prove that
(n + 1)(nx + 1)(nx + 2n + 1) ≤ 2n(x + 1)(n2 x + n + 2), which is valid, since it reduces
to (1 − n)(t − 2)2 ≤ 0.
Returning to our inequality, we now conclude that
n n √ n
xi + √ 1≤ n(n + 1)
√ n + 1 1 + x 2(n + 1)2 n2xi + 3n + 4
i
i=1 i=1
= 2nn .n + 1
Second proof. Putyi = xi + 1, for alli = 1, 2, . . . , n. In this case,
yi ≥ 1 (i = 1, 2, . . . , n) n , yi = n + 1
i=1
and therefore the inequality in question becomes
n n 1 n2 y − 1 √
≤ i y √

73
i
n + 1 ,
i=1 i=1
yj − 1 n2
≤√
or equivalently,n
n j=1 √ y n + 1.
i
i=1
From the Cauchy-Schwarz Inequality , we get
n √yi − 1 √ n √ 1 ·√y − 1 +√ y − 1 · √ 1 + · · · +√ y − 1 · √ 1 i=1 √ =n √n n
1 2 n y
1 y1
√ n ·1 + (y − 1) + · · · + (y − 1) (y − 1) + 1 + · · · + 1 ≤
2 n 1
n √ n n
y1
= −ny 2n + 1 ,1 + 2(n + 1) −ny1
and similarly one can prove that
n
j=1 √ y ny y − 1
i i j
≤ −ny 2n + 1 ∀i = 1, 2, . . . , n,i + 2(n + 1) − for anyi = 1, 2, . . . , n. This yields
n
n yj − 1n
j=1 ≤ −ny + 2(n + 1) − 2n + 1 √
i yi i=1 nyii=1
n 2n + 1 ≤ n −ny + 2(n + 1) −
i nyii=1
=
n
n
(
n
+ 1)

2n + 1n 1
n i=1 yi
☐ ☐
☐ n(n + 1) − 2n + 1 · n2 ☐ ≤ n☐ ☐ ☐ ☐
n n yi
i=1
=
n
n
(
n
+ 1)

74

2n + 1· n2 n2 n n + 1= √ n + 1.
Problem 71 Let a, b, c be positive real numbers. Prove that a
4 + b4 + c4 3abc 2
≥ 3(a2 + b2 + c2). ab + bc + ca + a + b + c
(Vo Quoc Ba Can, Cosmin Pohoat¸˘a)
Proof. Since 3(ab + bc + ca) ≤ (a + b + c)2 , it suffices to prove that 3(a4 + b4 + c4 )+
3abc 2
a + b + c ≥ 3(a2 + b2 + c2), (a + b + c)2
or equivalently,
2
a4 + b4 + c4 + abc(a + b + c) ≥ 9(a2 + b2 + c2 )(a + b + c)2 .
According to the Schur Inequality , we have that
3(a3 + b3 + c3 + abc) ≥ 2(a3 + b3 + c3 ) + a2 (b + c) + b2 (c + a) + c2 (a + b) = a3 + b3 +
c3 + (a + b + c)(a2 + b2 + c2 ).
On the other hand, from the Chebyshev Inequality ,
a3 + b3 + c3 ≥
1 2 + b2 + c2 ),3(a + b + c)(a and thus
1
3(a3 + b3 + c3 + abc) ≥ 3(a + b + c)(a2 + b2 + c2 ) + (a + b + c)(a2 + b2 + c2 )
4
=2 + b2 + c2 ).3(a + b + c)(a
From the same Schur Inequality , but this time applying it in the third degree case, it
now follows that
a4 + b4 + c4 + abc(a + b + c) ≥ a3 (b + c) + b3 (c + a) + c3 (a + b),
and therefore we conclude
2[ a4 + b4 + c4 + abc(a + b + c)]
≥ a4 + b4 + c4 + a3 (b + c) + b3 (c + a) + c3 (a + b) + abc(a + b + c) = (a + b + c)(a3 +
b3 + c3 ) + abc(a + b + c)
= (a + b + c)(a3 + b3 + c3 + abc)
4
≥2 + b2 + c2 ).9(a + b + c)(a
This completes our proof.
Problem 72 Let a, b, c be nonnegative real numbers, from which at least two are
nonzero. Prove that
3
3 a 9
b2 + c2 c2 + a2 a2 + b2
2
+
bc
2 2
+ 3 b + ca + 3 c + ab ≥ a + b +

75
√ abc
c.
(Pham Huu Duc, Math. Reflections)
First proof. By AM-GM Inequality , we get 3 a (a2 + bc)√ abc b2 + c2
2 + bc 3
= 3 b(a2 + bc) · c(a2 + bc) · a(b2 + c2 ) 3(
a
2
+
bc
)

3 abc≥
b(a2 + bc) + c(a2 + bc) + a(b2 + c2 )

3(
a
2 + bc) 3
= abc , a2 (b + c) + b2 (c + a) + c2 (a + b) and so, the problem reduces to

3 a2 (b + c) + b2 (c + a) + c2 (a + b)
a2 + b2 + c2 + ab + bc + ca ≥ a + b + c. In other words, this rewrites as
a(a − b)(a − c) + b(b − c)(b − a) + c(c − a)(c − b) ≥ 0. which is a particular case of the
Schur Inequality .
Second proof. By the AM-GM Inequality , we have
2 2 2 2 2 2
3 a + bc + 3 b + ca + 3 c + ab ≥ 3 9 (a + bc)(b + ca)(c + ab),b2 + c2 c2 + a2 a2 + b2 (a2 +
b2)(b2 + c2)(c2 + a2)
hence, it suffices to show that
(a + b + c)9 (a2 + bc) ≥ 39 a3 b3 c3 (a2 + b2 )(b2 + c2 )(c2 + a2 ).
cyc
Since, from the same AM-GM Inequality we have
ab(a2 + b2 1 ) = 1 · 2ab · (a2 + b2 ) ≤ 8(a2 + b2 + 2ab)2 = 18(a + b)4 ,
2
we are left to show that
(
a
+
b
+
c
)
9 (a2 + bc) ≥ 39

2
4 4 4
9 abc(a + b) (b + c) (c + a) .
cyc
Now since

76
(a2 + bc)(b2 + ca) − ab(a + c)(b + c) = c(a + b)(a − b)2 ≥ 0,
it follows that (a2 + bc)2 ≥ a2 b2 c2 (a + b)2 (b + c)2 (c + a)2 ,
cyc
or equivalently,
(a2 + bc)2 ≥ abc(a + b)(b + c)(c + a).
cyc
In this case, it remains us to show that
(
a
+
b
+
c
)
9 ≥ 39
3 3 3
29 (a + b) (b + c) (c + a) .
but this follows directly to the AM-GM Inequality .
Remark. Notice that with the help of the Cauchy-Schwarz Inequality we can also
prove that the following stronger inequality holds:
2 2 2 2 2 2 b + c a
3 b + c 3 c + a 3 a + b a +
2
+
bc
+
b
2
+
ca
+
c
2
+
ab


3
abc .
Problem 73 Let a1 , a2 , . . . , a100 be nonnegative eral numbers such that a2 + a2 + . . .
+1 2 a2 = 1. Prove that 100
a
2 12 . a + a2 a + . . . + a2
1 2 3
2 100 a1 <25
(IMO Shortlist 2007, proposed by Poland)
Proof. According to the Cauchy-Schwarz Inequality ,

77
1 a (a2 + 2a a ) + a (a2 + 2a a ) + . . . + a (a2 + 2a a )
1 1 2 2 2 3 100 100 1 3 100 1 99
1 100 1

1
100 2 2
≤3 a2 · (a2 + 2ak+1 ak+2 )2 .k k
k=1 k=1
Thus, it is suffice to show that
100
(a2 + 2ak+1 ak+2 )2 ≤ 2.k
k=1
Each term of this last sum can be seen as
a4 + 4a2 a2 +2 + 4a2 (ak+1 · ak+2 ) ≤ (a4 + 2a2 a2 + 2a2 a2 ) + 4a2 a2 .k k+1 k k k k k+1 k k+2
k+1 k+2
The required inequality now follows from
2
100 100
(a4 + 2a2 a2 + 2a2 a2 ) ≤ a2 = 1,k k k+1 k k+2 k
k=1 k=1
and
100
a2 a2 ≤ (a2 + a2 + . . . + a2 ) · (a2 + a2 + . . . + a2 )k k+1 1 3 99 2 4 100
k=1

1
100 2
a
2
4
k k=1
=1 .4
Problem 74 Let a, b, c be nonnegative real numbers, no two of which are zero. Prove
that ab + ac + 4bc+ bc + ba + 4ca+ ca + cb + 4ab≥ 4.b2 + c2 c2 + a2 a2 + b2
(Vasile Cˆırtoaje)
Proof. Without loss of generality, we may now assume thata ≥ b ≥ c; then
ab + ac + 4bc≥ ab + 5c2
b2 + c2 b2 + c2,
2 bc + ba + 4ca ab + 5c
≥ ,c2 + a2 a2 + c2
ca + cb + 4ab≥ 4ab
a2 + b2. a2 + b2
Thus, we proceed as follows.
ab + ac + 4bc+ bc + ba + 4ca+ ca + cb + 4ab− 4b2 + c2 c2 + a2 a2 + b2
ab

78
+ 5
c
2
ab + 5c2
≥ 4ab − 4 b2 + c2 + a2 + c2 + a2 + b2
ab
+
c
2 ab + c2 1 2
2 + b2 + c2 − 2(a − b) =
b
2
+
c
2
+
a
2
+
c
2

2
+ 4
c
1
a2 + c2 a2 + b2 2 (ab − c2 )) + 4 c2 (a2 + b2 + 2c2 ) − 2(a − b)2 =
(a − b)
(a2 + c2 )(b2 + c2 (a2 + c2 )(b2 + c2 ) a2 + b2
2 2 + 4c2 (a2 + b2 + 2c2 )
= (a − b)2 ab − c
(a2 + c2 )(b2 + c2 ) − a2 + b2 ( a2 + c2 )(b2 + c2 )
(a − b)2 [ab(a − b)2 − c2 (3a2 + 3b2 + 2c2 )] + 4c2 (a2 + b2 + 2c2 )= (a2 + b2)(a2 + c2)(b2 +
c2) (a2 + c2)(b2 + c2)
c2(a − b)2(3a2 + 3b2 + 2c2
) + 4c2(a2 + b2 + 2c2)≥ − (a2 + b2 )(a2 + c2 )(b2 + c2 a2 + c2 )(b2 + c2
)
c2(a − b)2(3a2 + 3b2 + 2c2) c2(3a2 + 3b2 + 2c2)≥ − (a2 + b2 )(a2 + c2 )(b2 + c2 ) + (a2 + c2 )(b2 +

c2 ) 2
abc
2 (3a2 + 3b2 + 2c2
= ) ≥ 0. (a2 + b2 )(a2 + c2 )(b2 + c2 )

Problem 75 Let a, b, c be positive real numbers. Prove that a + b2 + c3 b + c2 + a3 c +


a2 + b3 9.ab + c2 + bc + a2 + ca + b2 ≥ 2
(Vo Quoc Ba Can)
Proof. Combining the Cauchy-Schwarz Inequality with the Cˆırtoaje Inequality ,

79
we get b2 c2 a2 (a2 + b2 + c2 )2
ab + c2 + bc + a2 + ca + b2 ≥ ab3 + bc3 + ca3 + a2 b2 + b2 c2 + c2 a2

(a + b2 + c2 )2
2
1 (a2 + b2 + c2 )2 + a2 b2 + b2 c2 + c2 a2
3

(a2 + b2 + c2 )2
1 (a2 + b2 + c2 )2 + 1 (a2 + b2 + c2 )2 = 3,

3 3 2
and a 2 2 + 2 ≥ 2 2 2 , 2
+ b c (a + b + c) bc + a ca + b 2(a b + b c + c a) ab + c
c3 a3 b3 (a2 + b2 + c2 )2
ab + c2 + bc + a2 + ca + b2 ≥ a3 + b3 + c3 + 3abc.
This yields
a + c3 b + a3 c + b3 (a + b + c)2 (a2 + b2 + c2 )2 ab + c2 + bc + a2 + ca + b2 ≥ 2(a2 b + b2
c + c2 a) + a3 + b3 + c3 + 3abc (
a
+
b
+
c
)
2 (a2 + b2 + c2 )2 ≥ 2 2(a2 b + b2 c + c2 a) (a3 + b3 + c3 + 3abc) 4(
a
+
b
+
c
)(
a
2 + b2 + c2 ≥

a3 + b3 + c3 + 2(a2 b + b2 c + c2 a) + 3
) abc,

and therefore, it suffices to prove that


4(a + b + c)(a2 + b2 + c2 ) ≥ 3 a3 + b3 + c3 + 2(a2 b + b2 c + c2 a) + 3abc ,
or equivalently,
a(a − b)2 + b(b − c)2 + c(c − a)2 + 3(ab2 + bc2 + ca2 − 3abc) ≥ 0,
which is obviously true, since the AM-GM Inequality givesab2 + bc2 + ca2 ≥ 3abc.
Note that the equality holds if and only ifa = b = c = 1.
Problem 76 Let a, b and c be positive real numbers satisfying a + b + c = 2. Prove that
+ bc+ b2 + ca+ c2 + ab≤ 2 + cyc b2 + c2.2 + cyc b + c ≤ a 1 a21 a
b + c c + a a + b

80
(Cosmin Pohoat¸˘a)
Proof. Homogenizing, the left hand side of the inequality becomes a + b + c+ a(a + b +
c)≤ 2(a2 + bc) 2cyc b + ccyc b + c ,
which is equivalent to
3 a2 + 2bc b2 + 2ca c2 + 2ab
+ +
2(a + b + c) ≤ b + c c + a a + b .
This can be rewritten as
2
(b − c)2 (b + c) ≥ (b − c)2 (c + a)(a + b)cyc b + c ,cyc
which reduces to proving that
(b − c)2 Sa + (c − a)2 Sb + (a − b)2 Sc ≥ 0,
where
Sa = 2(b+c)2 −(c+a)(a+b), Sb = 2(c+a)2 −(a+b)(b+c), Sc = 2(a+b)2 −(b+c)(c+a).
Without loss of generality, we may now assume thata ≥ b ≥ c. In this case, we have
that
Sb = 2(c + a)2 − (a + b)(b + c) ≥ 2(c + a)(c + b) − (a + b)(b + c) = (b + c)(a − b + 2c) ≥
0,
Sc = 2(a + b)2 − (b + c)(c + a) ≥ 0, and Sa + Sb = (a − b)2 + 2c(a + b + 2c) ≥ 0.
Therefore,
(b − c)2 Sa + (c − a)2 Sb + (a − b)2 Sc ≥ (b − c)2 Sa + (a − c)2 Sb ≥ (b − c)2 (Sa + Sb ) ≥
0.
For the right hand side of the inequality, by the Cauchy-Schwarz Inequality , it is
enough to prove that
2
a2 + bc≤ 1 + (a2 + b2 + c2 )2 a4 + b4 + c4 a
2 b2 + b2 c2 + c2 a2 = 3 + a2 b2 + b2 c2 + c2 a2. b + c
cyc
This rewrites as
2
a2 + bc a4 + b4 + c4 b
+
c
− a ≤a2b2 + b2c2 + c2a2 − 1,
cyc
or equivalently, 2 (a − b)(a − c)≤ (a2 − b2 )(a2 − c2 ). cyc b + ccyc a2 b2 + b2 c2 + c2 a2
In other words,
x(a − b)(a − c) + y(b − c)(b − a) + z(c − a)(c − b) ≥ 0,
where
x = 2(a + b)(a + c) − b + c ≥ 0, a2 b2 + b2 c2 + c2 a2
because
( a + b)(a + c)(b + c)(a + b + c) ≥ 4(a2 b2 + b2 c2 + c2 a2 ). Similarly, we gety ≥ 0
andz ≥ 0. Without loss of generality, we assume now thata ≥ b ≥ c. Then it follows that

81
2 2 a2b2 + b2c2 + c2a2x − y = (a + b)(a + c) − (b + c)(b + a)+ c + a − b + c
a2 − b2 2(b − a)= a2 b2 + b2 c2 + c2 a2 + (a + c)(b + c)
= (a − b) 2a + b −
(a + c)(b + c) , a2 b2 + b2 c2 + c2 a2
which is nonnegative again because
(a + b)(a + c)(b + c)(a + b + c) ≥ 4(a2 b2 + b2 c2 + c2 a2 ).
Hence, the conclusion now follows from the generalized Schur Inequality .
Remarks. The left hand side of this problem is equivalent with
a + b + c1 a≤ a2 + bc+ b2 + ca+ c2 + ab 2 2 +cyc b + c b + c c + a a + b ,
sharpening then a well-known inequality by Hojoo Lee, i.e. Problem 2581, Crux
Mathe
maticorum : a2 + bc b2 + ca c2 + ab
+ + ≥ a + b + c.b + c c + a a + b
This part is also present in [Ci].
Problem 77 Real numbers ai , bi (1 ≤ i ≤ n) satisfy n a2 =n b2 = 1 and
n
i=1 i i=1 i
ai bi = 0. Prove that i=1
n 2 n 2
ai + bi ≤ n.
i=1 i=1
(BMO Shortlist 2007, proposed by Romania)
First proof. The simplest, by far, solution avails itself of methods used to compute
Fourier coefficients. PutA :=n ai andB :=n bi ; theni=1 i=1
n
0 ≤ (1 − Aai − Bbi )2
i=1
n
= (1 + A2 a2 + B2 b2 − 2Aai − 2Bbi + 2ABai bi )i i
i=1
n n n n n n
= 1 + A2 a2 + B2 b2 − 2A ai − 2B bi + 2AB ai bii i
i=1 i=1 i=1 i=1 i=1 i=1
= n + A2 + B2 − 2A2 − 2B2 + 0 = n − (A2 + B2 ).
This proves our inequality.
Second proof. With the notations from the solution above, consider the vector 1 =
(1, 1, . . . , 1). Denote byO the origin of a coordinates system inRn , and byP, Q, I the
other ends (thanO) of vectorsa, b, 1. We now are within a 3-dimensional subspace
embedded in Rn . LetJ be the foot of the perpendicular fromI on the plane (OP Q),
andP , Q be the feet of the perpendiculars fromJ on the lines (OP ), (OQ). From the
three-perpendicularstheorem it follows thatIP ⊥ OP andIQ ⊥ OQ (also, of course,OP
⊥ OQ ). But thenA =< a, 1 >= OP ,B =< b, 1 >= OQ ,
OP 2 + OQ 2 = P Q 2 = OJ2 = OI2 − IJ2 ≤ OI2 = ||1||2 = n,
and therefore, A2 + B2 ≤ n.

82
Problem 78 Let a, b, c be positive real numbers satisfying a + b + c = 1. Prove that √2
√ ab + √ bc + √ ca ≤
2 . ab + bc bc + ca ca + ab
(Chinese MO 2006)
First proof. From the Cauchy-Schwarz Inequality , we get
√b ≤ √a√2b
√ ab = √a √ a,
cyc ab + bc cyc c + a cyc c +
hence it suffices to prove that
√2a√b 2b√c 2c√a ≤ 1.c + √a + √a + √b + √b + √c
Furthermore, set√ a = x, √ b = y, √ c = z. We thus have to prove that
2x2y + 2y2z + 2z2x 2 2 2
≤ x + y + z ,z + x x + y y + z
or equivalently,
x2 +y2 +z2 + 2xy − 2x2y + 2yz − 2y2z + 2zx − 2z2x ≥ 2(xy +yz +zx).z + x x + y y + z
In other words,
x
2 + y2 + z2 + 2xyz1 1 1

x + y +y + z +z + x ≥ 2(xy + yz + zx).
Now since the Cauchy-Schwarz Inequality gives us that 1 1 1 9xyz2xyz x + y + y + z
+ z + x≥ x + y + z, the problem reduces to proving that
x
2
+ y2 + z2 +9xyz
x + y + z ≥ 2(xy + yz + zx),
which is simply a particular case of the Schur Inequality written for the third degree.
Second proof. By the Cauchy-Schwarz Inequality , we have

ab
2 ab a(ab + bc + ca)2 = ab + bc + ca c + a ab + bc
· cyc cyc
ab a(ab + bc + ca)≤ ab + bc + cacyc c + acyc
= a(ab + bc + ca),
cyc c + a
and thus, it is enough to prove that a
(
ab
+
bc
+
ca
)
1

2(
a
+

83
b + c )
2
c + acyc
, or equivalently,
2ab2 + 2bc2 + 2ca2 2 2 2
+ b + c .a + b b + c c + a ≤ a
This can be rewritten into
2
ab
2 2bc2 2ca2

2b2 −a + b + 2c2 −b + c + 2a2 −c + a ≥ a2 + b2 + c2 ,


or in other words,
2
b3
c3 c3 2
a + b + + b2 + c2 .b + c +c + a ≥ a
This last inequality is valid, since from the Cauchy-Schwarz Inequality we get
2
a
+
b
+
c3 c3 2(a2 + b2 + c2 )2 2(a2 + b2 + c2 )2 b3
b + c + c + a≥ a2 + b2 + c2 + ab + bc + ca ≥ 2(a2 + b2 + c2 ) = a2 + b2 + c2 .
Third proof. By same Cauchy-Schwarz Inequality , we have
2 √a + b · (a + b)(a + c) ab + bc= a2b√ ab
cyc
cyc
2
a2 b≤ (a + b) (a + b)(a + c)cyc cyc
a
2
= 2b ,
cyc (a + b)(a + c)
hence it suffices to prove that a2 b 4(a + b)(a + c) ≤ a + b + c,
cyc
which is equivalent to 4a2 b(b + c) + 4b2 c(c + a) + 4c2 a(a + b) ≤ (a + b)(b + c)(c + a)
(a + b + c),
this last one being true, because it can be written as ab(a − b)2 + bc(b − c)2 + ca(c − a)2
≥ 0.
Fourth proof. The inequality in question is equivalent to

ab = a + b· 4a2 b
(
a

84
+
b
)
2 (a + c) .

cyc ab + bccyc 2
Consider the concave functionf (a) = √ a, defined on the real line. According to the
Jensen Inequality , we have that
a + b· f 4a2 b 4a2 b (a + b)2 (a + c)≤ f a + b · (a + b)2 (a + c)cyc 2cyc 2
2a2 b= f (a + b)(a + c)cyc
2a2 b= (a + b)(a + c),
cyc
and so, the problem reduces to showing that
2a2 b ≤ 1. cyc (a + b)(a + c) 2
The solution for this last inequality can be seen in the third proof .
Problem 79 Let a, b, c be nonnegative real numbers satisfying a2 + b2 + c2 = 1. Prove
that
1 ≤ a + b + c ≤ √2. 1 + bc 1 + ca 1 + ab
(Faruk Zejnulahi, Crux Math.)
First proof. For all real numbersx contained in the interval 0, 1 , we have2 2
x

1
=
x
(1

2
x
)
1 − 3 1 + x 3(1 + x) ≥ 0, and therefore
1 ≤ 1 − 2x.1 + x 3
Now, notice that max{ab, bc, ca} ≤ 1 . In this case, we get2
a + b + c ≤ a 1 − 2 bc + b 1 − 2 ca + c 1 − 2 ab
1 + bc 1 + ca 1 + ab 3 3 3

= a + b + c − 2abc = a(1 − 2bc) + b + c ≤ [a2 + (b + c)2 ][(1 − 2bc)2 + 1] = 2(2b2


c2 − 2bc + 1)(1 + 2bc) = 2[1 − 2b2 c2 (1 − 2bc)] ≤ √2.
For the right hand side of the inequality, note that
1 − 1 + x = x2 1 + x 1 + x ≥ 0, and therefore1 ≥ 1 − x, for all positive real numbersx.

Hence1+x
a + b + c ≥ a(1 − bc) + b(1 − ca) + c(1 − ab)
1 + bc 1 + ca 1 + ab
= a + b + c − 3abc
= a2 + b2 + c2 + 2(ab + bc + ca) − 3abc = 1 + 2(ab + bc + ca) − 3abc,

85
and since√ 2
1 + 2u − 1 =1 + √u ≥2u √ 3 − 1 u,2u + 1 1 + √3 =
we conclude that
1 + 2(ab + bc + ca) − 3abc ≥ 1 + √ 3 − 1 (ab + bc + ca) − 3abc
≥ 1 + √ 3 − 1 (ab + bc + ca)3/2 − 3abc √

3

1
3
3

a
2
b
2
c
2 3/2
≥ 1 + − 3abc
= 1 + 3 2 −√3 abc ≥ 0.
Second proof. First, we will show that
(a + b + c)2 ≤ 2(1 + bc)2 ,
or equivalently, 2a(b + c) ≤ 1 + 2bc + 2b2 c2 . This rewrites as 2a(b + c) ≤ a2 + b2 + c2
+ 2bc + 2b2 c2 , which is obviously true since
(a − b − c)2 + 2b2 c2 ≥ 0.
Thus, we now have that a
+ b + c ≤
√ 2a√ 2b√ 2c a + b + c + a + b + c + a + b + c = √2, 1 + bc 1 + ca 1 + ab
and since the AM-GM Inequality gives us
2 2 2 2
a + abc ≤ a + a(b + c )= a + a(1 − a )= 1 − (a − 1) (a + 2)≤ 1,2 2 2
it follows that
1 +
a + b + c= a + abc + b2 c2 a2
bc 1 + ca 1 + ab b + abc + c + abc ≥ a2 + b2 + c2 = 1.
Problem 80 Let a, b, c be positive real numbers such that a ≤ b ≤ c and abc = 1. Prove
that
a + b2 + c3 ≥ 1 + 12 + 13 .a b c
(Vo Quoc Ba Can)
Proof. From the hypothesis, we deduce thatc ≥ 1, bc ≥ 1. Notice that the our inequality
is equivalent to
1
+ 1
≥ 1 −
b4

86
(c3 a − 1)a c3 b2 .
Now we see thatc3 a − 1 ≥ c2 ba − 1 = c − 1 ≥ 0. Ifb ≥ 1, the inequality proves itself
trivial since 1 + 1 1 − b4
3 ≥ 0 ≥
(c3 a − 1)a c b2 .
Otherwise, if 1 ≥ b, then we have
c3 a + b4 − 2 ≥ a(c3 + b3 ) − 2 ≥ abc(b + c) − 2 = b + c − 2 ≥ 2 √ bc − 1 ≥ 0,
and therefore, c3 a − 1 ≥ 1 − b4 ≥ 0.
Now, ifb2 ≥ a, then our inequality is proved. So, it is now suffice prove our inequality
only in the case whena ≥ b2 . We have that
c3 a − 1 − c(1 − b4 ) ≥ c3 b2 + cb4 − c − 1
= c3 b2 + cb4 − c · (abc)4/3 − (abc)5/3 ≥ c3 b2 + cb4 − c · (b2 c)4/3 − (b2 c)5/3 = b2 c(c2 +
b2 − b4/3 c2/3 − b2/3 c4/3 ) ≥ 0,
and thus
c3 a − 1 ≥ c(1 − b4 ) ≥ 0. This yields
c2 c − a≥ b − a≥ 0.a − 1 = b
b2 ab2 ab2 This completes our proof.
Problem 81 Given k + 1 positive real numbers x0 , . . . , xk and a positive integer n,
show that k
(
x
σ
1

+
. . .
+
x
σ
k

)
−n ≤ k −n x − n
i ,
σ i=0
where the sum on the left is taken of the k + 1 distinct k-element subsets of {x0 , . . . ,
xk }. (Emre Alkan, Amer. Math. Monthly )
Proof. (by C˘alin Popescu) The problem is a special case of the following result:
given k + 1 positive real numbersx0 , . . . , xk and a positive integern, then for anyj ∈
{1, . . . , k + 1},
(xσ1 + . . . + xσn )−n ≤ j−n−1 k k σ j − 1i=0 x
− n i ,

where the sum on the left is taken over thek+1 distinct j-element subsets ofj

87
{x0 , . . . , xk }.
The Jensen Inequality applied to the convex real-valued functionx → x−n (x > 0)
gives
x
σ
1

+
. . .
+
x
σ
−n

j


x
−n + . . . + x−n
σ 1 σ j
j j or, equivalently,
(
x
σ
1

+
. . .
+
x
σ
j

)
−n ≤ j−n−1 x−n + . . . + x−n

σ 1 σ j

j
−n−1 k k
j

1
i=0
x
− n i .
Problem 82 Let a, b, c be nonnegative real numbers, such that at least two are nonzero
and which satisfy the condition a + b + c = 1. Prove that
4
√ a b c
√27 √3 − 1 .
a + 2b + √b + 2c + √c + 2a ≤ √2

88
(Vo Quoc Ba Can, Pham Kim Hung)
Proof. Ifc ≥ b ≥ a, then by the Cauchy-Schwarz Inquality , we have
a2 a a√ a + 2b≤ a a + 2b= a + 2b,cyc cyc cyc cyc
hence it suffices to prove that
a

4

≤27 √3 − 1 2 √ 3 − 9 cyc a + 2b√ 2= 6


or equivalently,
1 √ 3 − 9, 1 + 2x ≤ 6
cyc
wherex = b , y = c , z = a . Furthermore, following the fact thatxyz = 1, we geta b c
1
1= 3 + 2xy + 1 − 2(x − 1)(y − 1)(4xy − 1)
2x + 1 +
1 1
2y + 1 3(2x + 1)(2y + 1)(2xy + 1)
1
≤1 + 1 = 13 + z ,3 +2xy z + 2
and therefore
1

1 z 3z 3 + + 2 + + 1 = 4 − ≤ 4< 6√3 − 9.
z 2z (z + 2)(2z + 1)
cyc 2x + 1 3
If
a

b

c
, set
f
(
a, b, c
) =

a
a
+2
b
, and consider the two cases:
cyc
Case 1. Ifa ≥ 4b, then
(a + c)2 a2 c(a2 + 4ab + ac + 2bc) ≥ 0, a + c + 2b − a + 2b = (a + 2b)(a + 2b + c) and so

a + 2b

89
a ≤ √ a + c
a + c + 2b. Moreover
√c b cb + √c + 2a ≤ √b + 2c + √c + 8b ≤ √b,b + 2c
which gives that
f
(
a, b, c
)


a + c

a + c + 2b + b = f (a + c, b, 0). Case 2. If 4b ≥ a, then
c 2 a2 c [2a(4b − a) + c(a + 2b)]a + 2 −
a + 2b = 2(a + 2b)(2a + 4b + 3c) ≥ 0, a + 2b + 3c
2
and thusa ≤ a + c

a + 2b
2 ,
a
+ 2
b
+
3c
2
and

b c b c
b + 2c + √ c + 2a ≤ √ b + 2c + √ c + 2b ≤ b +c, 2 which yields
f
(
a, b, c
)

a
+
c
2
+ b +
c c , b +c , 0 . a

+ 2
b
+
f a +
3c 2 = 2 2
2

90
From these cases, we now get a very interesting conclusion. That is, in order to
prove our original inequality, we can resume to show it in the case when one variable
is equal to zero. Assume, without loss of generality, that this one isc. In this case,

4

f (a, b, 0) = f (1 − b, b, 0) = √
1 − b √ 27 √ 3 − 1
+ b ≤ .1 + b √2
From this, the conclusion follows immediately. The equality holds if and only if (a, b,
c) = (
2
(
√ 3 −1) −√
2 3
√ 0)3 , √ 3 ,
Remarks. Note that the inequality in questions is a refinement of the following one,
proposed by Pham Kim Hung in Math. Reflections:
√b c 3a + √b + 2c + √c + 2a ≤ 2(a + b + c).a + 2b
Problem 83 For real numbers xi > 1, 1 ≤ i ≤ n, n ≥ 2, such that
n x2
i ≥ S = x , for all i = 1, 2, . . . , n
j xi − 1 j=1
find, with proof, sup S. (Romanian IMO Team Preparation Tests 2008)
Proof. (by Dan Schwarz) Forn = 1,S is unbounded to the right, since for the pairs v,
v
v2 v2
v−1 , wherev > 1, we haveS = , and then limv→1 = ∞.v−1 v−1
Forn > 2, we will prove the tight upper boundS ≤n2 , with equality if and only if x
i
=
n n−1

n −1 , for alli = 1, 2, . . . , n.
Consider the functionf : (1, +∞) → [4, +∞), defined byf (x) =x2 . If is straightx−1
forward to prove thatf is strictly decreasing on (1, 2]. Let now
M
=
n
maxxi . i=1
Thus, 2M ≥ S > M + (n − 1),
M − 1
and thereforeM <n − 1 ≤ 2,
n − 2
which implies thatxi ∈ (1, 2) for alli.
Now, form > 1, eitherxi ≥ m for somei, and thenS ≤xi
2 ≤m2 (according to the
xi−1 m−1

91
monotonicity of f), orxi ≤ m for alli, and then clearlyS ≤ nm. Solving the equation
m2 = nm, obtained by equaling the two possible upper bounds forS, yields as unique
m
−1
solutionmn , and therefore, in all cases,0 =n−1
S ≤ m2 n20 1 = nm0 = ,
n − 1 m0 −
with equality if and only if all
x
i
’s are equal with
n n−1 .
Remarks. Note that from this inequality, we can deduce the following problem of
S. Berlov, which appeared in 2005 at the Russian MO.
Real numbers
a
1
, a
2
, a
3
>
1
satisfy
a
1
+
a
2
+
a
3
=
S
and
a
2

i

ai−1 > S for i = 1, 2, 3. Prove that


1
+
1
>
1
.

92
a1 + a2
1 +
a2 + a3 a3 + a3
Problem 84 Let a, b, c, d be positive real numbers such that a + b + c + d = abc + bcd
+ cda + dab. Prove that
a2 + 1 + b2 + 12 + c2 + 1 + d2 + 12 ≤ (a + b + c + d)2 .
(Vo Quoc Ba Can)
Proof. The inequality is equivalent to
4 + 2 (a2 + 1)(b2 + 1) + 2 (c2 + 1)(d2 + 1) ≤ 2(ab + ac + ad + bc + bd + cd).
Sincea + b + c + d = abc + bcd + cda + dab,
(a + b)(a + c)(a + d)a2 + 1 = a + b + c + d
b2 + 1 = (b + a)(b + c)(b + d),a + b + c + d
and we thus have that
4(
a
2 2
2 + 1)(b2 + 1) = 4(a + b) (a + c)(b + d)(a + d)(b + c) (a + b + c + d)
2
[(a + c)(b + d) + (a + d)(b + c)]2 ≤ (a + b) , (a + b + c + d)2
which gives 2 (a2 + 1)(b2 + 1) ≤ (a + b)[(a + c)(b + d) + (a + d)(b + c)].a + b + c + d
Similarly, we get
2 (c2 + 1)(d2 + 1) ≤ (c + d)[(a + c)(b + d) + (a + d)(b + c)],a + b + c + d
and yields
2 (a2 + 1)(b2 + 1) + 2 (c2 + 1)(d2 + 1) ≤ (a + c)(b + d) + (a + d)(b + c).
It is thus enough to prove that
4 + (a + c)(b + d) + (a + d)(b + c) ≤ 2(ab + ac + ad + bc + bd + cd),
or equivalently, (a + b)(c + d) ≥ 4. In other words,
(a + b)(c + d)(a + b + c + d) ≥ 4[ab(c + d) + cd(a + b)], which can be fianlly rewritten
into (c + d)(a − b)2 + (a + b)(c − d)2 ≥ 0. This completes our proof.
Remarks. Note that this inequality is sharpening the following one due to Gabriel
Dospinescu:
a
+
b
+
c
+
d

a2 + 1+ b2 + 1+ c2 + 1+ d2 + 1 2 2 2 2 .
Problem 85 Let a, b, c be the sidelengths of a triangle. Prove that
a2 b − 1 + b2 c − 1 + c2 a − 1 ≥ 0.c a b
(Moldavian IMO Team Selection Test 2006)
Proof. The inequality is equivalent to
a2b b2c c2a
+ + ≥ a2 + b2 + c2 .c a b

93
By the Cauchy-Schwarz Inequality , we have
a2b +b2c +c2a a2 bc + b2 ca + c2 ab ≥ (a2 b + b2 c + c2 a)2 ,
c a b
hence it suffices to prove that
(a2 b + b2 c + c2 a)2 ≥ abc(a + b + c)(a2 + b2 + c2 ).
Without loss of generality, we may now assume that (a − b)(a − c) ≤ 0. Consider the
following real-valued quadratic polynomial:
f (x) = (a2 + b2 + c2 )x2 − 2(a2 b + b2 c + c2 a)x + abc(a + b + c). We have
f (a) = a(a − b)(a − c)(a − b + c) ≤ 0, and since,
lim f (x) = ∞,
x→∞
then there existsx0 such thatf (x0 ) = 0. Moreover, it now follows that the discriminant
of f (x) must be nonnegative, i.e.
(a2 b + b2 c + c2 a)2 ≥ abc(a + b + c)(a2 + b2 + c2 ).
This yields our conclusion.
Problem 86 Let x, y, z be positive real numbers such that x2 + y2 + z2 ≥ 3. Prove that
x5 − x2 y5 − y2 z5 − z2 x5 + y2 + z2 + y5 + z2 + x2 + z5 + x2 + y2 ≥ 0. (Vasile Cˆırtoaje)
Proof. The inequality is equivalent to
1 ≤3
x
2
+ y2 + z2. cyc x5 + y2 + z2
From this, we can easily deduce that it suffices to consider the inequality in the case
when x2 + y2 + z2 = 3; then our inequality becomes
1 ≤ 1.
5 2
cyc x − x + 3
By the AM-GM Inequality , we have
x
6 6 ≥ 2 . 2 2 2
5 = x 2x x x + 1 Put nowa = x , b = y , c = z . In this case,a + b + c = 3, and thus
we are left to prove
that 1 ≤ 1,2a3 − a + 3
cyc a+1
or equivalently,a + 1 ≤ 1,
3 2
cyc 2a − a + 2a + 3 i.e., n other words,
(a − 1)2 (−2a2 + 3a + 3)≥ 0. cyc 2a3 − a2 + 2a + 3
Without loss of generality, we may now assume that a ≥ b ≥ c, and soa ≥ 1 ≥ c.
We will consider the following two cases:
Case 1. Ifb + c ≥ 1; thena ≤ 2, and therefore
−2a2 + 3a + 3 > 0, −2b2 + 3b + 3 > 0, −2c2 + 3c + 3 > 0.
The result follows immediately.
Case 2. Ifb + c ≤ 1; thena ≥ 2, and so we have
(2
a
3 − a2 + 2a + 3) − 5(a + 1) = 2a3 − a2 − 3a − 2 = a3 2 − 1 − 3 − 2 a a2 a3 ≥ a3 2 − 1 − 3 −

94
2 = 1a3 > 0,
2 22 23 2 which implies that
a + 1 ≤ 1, 2a3 − a2 + 2a + 3 5

hereby, it suffices to prove that


b + 1 + 3 + c + 1
2c3 − c2 + 2c + 3 ≤ 4. 2b3 − b2 + 2b 5
We will actually show more: for all positive realsx contained in the interval [0, 1], we
have
that x + 1 ≤ 2. 2x3 − x2 + 2x + 3 5
Indeed, this rewrites into
4x3 ≥ (x + 1)(2x − 1),
and note that ifx ≤ 1 , then it is trivial, or otherwise ifx ≥ 1 we get2 2
4x3 − (x + 1)(2x − 1) ≥ 4x3 − 2(2x − 1) = 2(2x3 − 2x + 1) ≥ 2(x2 − 2x + 1) = 2(x − 1)2
≥ 0.
This completes our proof.
Remarks. Notice that the inequality in question is refining Problem 3 from IMO
2005: for any positive realsx,y,z satisfyingxyz ≥ 1, we have that
x5 − x2 y5 − y2 z5 − z2
x5 + y2 + z2 + y5 + z2 + x2 + z5 + x2 + y2 ≥ 0.
Problem 87 Let a, b, c be the sidelengths of a given triangle. Prove that a + b + c + 3 ≥ 2
a + b + b + c + c + a .b c a b + c c + a a + b
(Vo Quoc Ba Can)
Proof. The inequality is equivalent to
a + 2c a b
+
c
≥ 2b + c + 3,
cyc b cyc cyc
or
ab
a
+ 2c a b
+
c
≥ ab 2b + c + 3 .
b
cyc cyc cyc cyc cyc
This rewrites asca2 bc2 abc c.cyc b + 2 b + c ≥ a2 + 2 b +cyc cyc cyc On the other hand,
according to Problem 85 , we have that ca2 2,
cyc
b
≥ a
cyc
hence it is suffice to prove that
bc2 abc c, b + c ≥ b +
cyc cyc

95
or in other words, bc(c − a)≥ 0, cyc b + c
which furthermore can be rewritten into
bc(c2 − a2 )(a + b) ≥ 0,
cyc
which is valid, because the AM-GM Inequality yields
a2 b3 ≥ a2 b2 c.
cyc cyc
Problem 88 Let x, y, z be positive real numbers. Prove that x + y+ y + z+ z + x− 4 x
y
+
z
+
y + z z x y z + x x + y ≥ 1 − 8xyz . (x + y)(y + z)(z + x) (Cezar Lupu and Cosmin
Pohoat¸˘a, La Gaceta de la RSME)
First proof. Consider the substitutionsx =1 (b + c − a),y =1 (c + a − b), andz =
1
2 2
(a + b − c). In this case, the inequality rewrites as2
a + b c
c
+
a

b
+
a
+
b

c
+ (
b + c − a)(c + a − b)(a + b − c) b + c − a abc ≥ b + c − a+ c + a − b+ a + b − c+ 1,a b c
wherea,b,c are the sidelenghts of a triangle. According to the well-known identities
( b + c − a)(c + a − b)(a + b − c) = 8sr2 abc = 4sRr,(1) whereR,r,s are the
circumradius, the inradius, and the semiperimeter of the triangle which has the
sidelenghtsa,b,c, the inequality is equivalent with
a + b c
c
+
a

b
+

96
a
+
b

c
+
2r
b + c − a R
≥ b + c − a+ c + a − b+ a + b − c+ 1,a b c
or in other words,
a b c + 4r s − a s − b s − c
+ + ≥ 4 · ( + + ) + 2.s − a s − b s − c R a b c
Moreover, the inequality from above is obviously equivalent with
a(s − b)(s − c) + b(s − a)(s − c) + c(s − a)(s − b)+4r ≥ 4s(ab + bc + ca) − 12abc +
2abc,sr2 R abc
which now can be rewritten as
s2 (a + b + c) − 2s(ab + bc + ca) + 3abc + 4r ≥ 4s(ab + bc + ca) − 10abc.sr2 R abc
From the fact thatab + bc + ca = s2 + r2 + 4Rr, and according to the identities from (1),
this last inequality can be reformulated into
4R − 2r + 4r ≥ s2 + r2 − 6Rr,r R Rr
i.e.
4R2 + 4Rr + 3r2 ≥ s2 . Note that this is the left hand side of the Gerretsen Inequality .
Second proof. The inequality is equivalent to
x
2 2
1 + 1 − 4 x(y + z ) − 6xyz y
+
z
≥cyc , cyc yz (x + y)(y + z)(z + x)
and thus, the problem reduces to proving that x(y − z)2 x(y − z)2
yz
(
y
+
z
)
≥ (x + y)(y + z)(z + x).
cyc
On the other hand,1 ≥ 1
(x + y)(x + z), yz
which yields
x(y − z)2 1 − 1
(
x

97
+
y
)(
x
+
z ) ≥ 0. cyc y + z yz
Note that this is equivalent with x(y − z)2 x(y − z)2
yz
(
y
+
z
)

(x + y)(y + z)(z + x),
cyc
fact from which we deduce now our conclusion.
Remarks. The inequality in question is sharpening the well-known inequality due to
Levinson (see [MiBaMaRa]):
x + y+ y + z+ z + x≥ 4 x
y + z + y + z . z x y z + x x + y
Problem 89 Let a, b, c be real numbers satisfying a2 + b2 + c2 = 9. Prove that
3 · min {a, b, c} ≤ 1 + abc.
(Virgil Nicula, Crux Mathematicorum)
Proof. Without loss of generality, we may assume thatc ≥ b ≥ a. In this case, notice
that the given inequality becomes
abc + 1 ≥ 3a. We split this problem in two separate cases:
Case 1. Ifa ≤ 0, then from the AM - GM Inequality , we get
a
(
b
2 + c2) a(9 − a2) a + 1)2
+ 1 − 3a = + 1 − 3a = (
abc + 1 − 3a ≥(2 − a) ≥ 0,2 2 2
proving then our inequality.
Case 2.
If
a

0
,
and since
c

b

98

a,
we obtain that
3
√ 3 ≥ a ≥ 0. Now, ifa ≤ 1 , then we are done. Now, if√ 3 ≥ a ≥ 1 , we can easily check
that3
bc ≥ a b2 + c2 − a2 = a 9 − 2a2 ,
and therefore, the problem reduces to showing that
a2 9 − 2a2 + 1 ≥ 3a,
or equivalently, f (a) = 2a6 − 9a4 + 9a2 − 6a + 1 ≤ 0. We (re)split this now into two
more cases: if 1 ≥ a ≥ 1 , then3
2f (a) = a2 (a − 1) 4a(a − 1)2 + 12a2 − 18a + 7 + (1 − 3a)(7a2 − 6a + 3) ≤ 0. Otherwise,
if√ 3 ≥ a ≥ 1, we have
f (a) ≤ 2a6 − 9a4 + 9a2 − 5 = (a2 − 1)(2a4 − 7a2 + 2) − 3 ≤ −3 < 0.
This completes our proof. Note that the equality occurs if and only if (a, b, c) ∈ {(−1,
2, 2), (2, −1, 2), (2, 2, −1)}.
Problem 90 Let x1 , x2 , . . . , x3n be positive real numbers. Prove that
n3n 1 + x2 3n

2n ·k ≥ 1 + x1/n .k
1 + xk
k=1 k=1
(Mihaly Bencze, Math. Magazine)
Proof. Let us first prove that ifx,y,z are positive real numbers, then 2 1 + y2 1 + z2 1 +
xyz 1 + x
· 1 + y · 1 + z ≥ 2 .1 + x
To show this, notice that the above inequality rewrites as
(1 + x2 )(y − z)2 + (xyz − 1)(x − 1)(y − 1)(z − 1) ≥ 0,
cyc
and therefore it trivially holds when 0 ≤ x, y, z ≤ 1 orx, y, z ≥ 1. By symmetry we
have two remaining cases to check:x ≥ 1 ≥ y ≥ z andx ≥ y ≥ 1 ≥ z. Setting f (x) = (1 +
x2 )/(1 + x) thenxf (1/x) = f (x) and in the first case:
1 +
yz/x
=
x
+
yz
≥ 1 +
xyz
f (x)f (y)f (z) = xf (1/x)f (y)f (z) ≥ x ·2 2 2 ,
where the last inequality holds because (x − 1)(1 − yz) ≥ 0. On the other hand, in the
second case:
f
(

99
x
)
f
(
y
)
f
(
z
) =
xf
(1
/x
)
yf
(1
/y
)
f
(
z
)

xy
·
1 + z/(xy)= xy + z≥ 1 + xyz 2 2 2 where the last inequality holds because (z − 1)(1 −
xy) ≥ 0.
We now make use of the Huygens Inequality . Thus,
☐3
k
=

2
·
3
i
+
j
1 +
x
2 ☐ n−1 ☐ 3 ☐ 3n 1 + x2 n−1
2n · 1 + x ☐ ≥ ☐1 + x3i+j☐
k i=0 j=1 1 + x3i+j
k=1 i=0 j=1 ☐ ☐n−1 3 ☐1/n ☐n n3n

100
≥☐ 1 +☐ x3i+j ☐☐ = 1 + x1/n .☐ i=0 j=1 ☐ k
k=0
Remarks. It is interesting to see that the more general inequality
nan 1 + x2 an 2n ·k ≥ 1 + x1/n ,
k
k=1 1 + xk k=0
where a is a positive integer andx1 , x2 , . . . , xan ≥ 0 holds if and only ifa = 1, 2, 3
(fora ≥ 4, takexk = 2 and note that 2 · 5a < 3a + 6a ). This motivates the choice of the
number 3 in the initial statement.
Problem 91 Given an integer n ≥ 2, find the largest constant C(n) for which the
inequality n
xi ≥ C(n) 2xi xj +√ xi xj
i=1 1≤j<i≤n
holds for all real numbers xi ∈ (0, 1) satisfying (1 − xi )(1 − xj ) ≥ 1 for 1 ≤ j < i ≤ n. 4
(Bulgarian IMO Team Selection Tests 2007)
Proof. Let us see what happens ifx1 , = x2 = . . . = xn = 1 . We have that2
n ≥ C(n) 2 · 1 · n(n − 1) + 1 · n(n − 1),2 4 2 2 2
and thus,
C(n) ≤1 .n − 1
We will now show that this is the value we are looking for. That is, we are left to prove
that
n
(n − 1) x√ xi xj .i ≥ 2xi xj +
i=1 1≤j<i≤n
By the AM-GM Inequality , we have
√ x x ≤ 1 (x + x ) =n − 1 n
i j i j
1≤j<i≤n 2 1≤j<i≤n 2 i=1
xi ,
and furthermore
n 2 n
2
x
i
x
j
=
x
i

x
2 i 1≤j<i≤n i=1 i=1
n 2 1 n 2

101
≤ xi
i=1
xi −n i=1
n 2
= n − 1xi .n i=1
Hence the problem reduces to
n n − 1n 2 n − 1n (n − 1) x + x ,
i i
i=1
xi ≥ n i=1 2 i=1

or equivalently,
n
xn .i ≤2i=1
For this, note that the AM-GM Inequality is giving us
2 − xj − xi = (1 − xj ) + (1 − xi ) ≥ 2 (1 − xj )(1 − xi ) ≥ 1,
for all integersi, j satisfying 1 ≤ j < i ≤ n. This yields
1 − xj − xi ≥ 0,
and therefore (1 − xi − xj ) ≥ 0,
1≤j<i≤n
from which we now can conclude that
n
xn .i ≤2i=1
This proves our inequality, and therefore
max
C ( n ) =
1 n−1 .
Problem 92 Let a, b, c be nonnegative real numbers, from which at least two are
nonzero and satisfying the condition ab + bc + ca = 1. Prove that
a3 + a + b3 + b + c3 + c ≥ 2√ a + b + c.
(Iran TST, 2008)
First proof. From the H¨older Inequality , we have
2 b2 c2

a3 + a + b3 + b + c3 + c2 a 3 ,a2 + 1 +b2 + 1 +c2 + 1 ≥ (a + b + c)


hence it suffices to prove that
(
a
+
b
+
c
)
2
a2

102
4
a
2
+ 1 +
b2 c2
,
b2 + 1 +c2 + 1
or equivalently, (a + b + c)2 a2 b2 c2 ab + bc + ca ≥ 4 (a + b)(a + c) + (b + c)(b + a) + (c
+ a)(c + b) . This can be rewritten into
(a + b + c)2 4 a2 (b + c) + b2 (c + a) + c2 (a + b),ab + bc + ca ≥ (a + b)(b + c)(c + a)
or in other words,
a2 + b2 + c2 8abc . ab + bc + ca − 1 ≥ 1 − (a + b)(b + c)(c + a) Now since

a2 + b2 + c2 (a − b)(a − c) + (b − c)(b − a) + (c − a)(c − b) ab + bc + ca − 1 = ab + bc +


ca
1

8abc
(
a
+
b
)(
b
+
c
)(
c
+
a
)
=
a2 (b + c) + b2 (c + a) + c2 (a + b) − 6abc (a + b)(b + c)(c + a)
= (a − b)(a − c),
cyc (a + b)(a + c)
the problem reduces to proving that
x(a − b)(a − c) + y(b − c)(b − a) + z(c − a)(c − b) ≥ 0,
where 1
1
a
2
x = ab + bc + ca − (a + b)(a + c) = (ab + bc + ca)(a + b)(a + c) ≥ 0 andy,z are defined
similarly.
For this, we proceed with assuming (without loss of generality) thata ≥ b ≥ c; then
x − y = 1 a − b1 − (a + b)(a + c) = (a + b)(b + c)(c + a) ≥ 0, (b + c)(b + a)
and therefore

103
x (a − b)(a − c) + y(b − c)(b − a) + z(c − a)(c − b) ≥ x(a − b)(a − c) + y(b − c)(b −
a) ≥ y(a − b)(a − c) + y(b − c)(b − a) = y(a − b)2 ≥ 0.
This proves our inequality.
Second proof. By squaring the both sides, the inequality to prove becomes
a3 + 2 ab(a2 + 1)(b2 + 1) ≥ 3 a.
cyc cyc cyc
By the Cauchy-Schwarz Inequality , we now have that
(
a
2
+ 1)(
b
2
+ 1) = (
a
+
b
)
2
(
a
+
c
)(
b
+
c
)

(
a
+
b
)
2

2
ab + c , and thus, it follows that
ab (a2 + 1)(b2 + 1) ≥ (a + b)√ ab √ ab + c = ab(a + b) + c√ ab(a + b) ≥ ab(a + b) +
2abc.
In this case, it suffices to prove that
a3 + 2 ab(a + b) + 12abc ≥ 3 a ab ,
cyc cyc cyc cyc
which is obviously true according to the Schur Inequality .

104
Problem 93 Let a, b, c , d be nonnegative real numbers such that a + b + c + d = 3.
Prove that
ab(a + 2b + 3c) + bc(b + 2c + 3d) + cd(c + 2d + 3a) + da(d + 2a + 3b) ≤ 6√ 3.
(Pham Kim Hung)
Proof. Set f (a, b, c, d) = ab(a + 2b + 3c).
cyc
In this case, we have that
f (a, b, c, d) − f (a + c, b, 0, d) = c(b − d)(a + c − b − d),
and furthermore
f (a, b, c, d) − f (0, b, a + c, d) = −a(b − d)(a + c − b − d).
Thus, if (b − d)(a + c − b − d) ≤ 0, thenf (a, b, c, d) ≤ f (a + c, b, 0, d), and moreover if
f (b − d)(a + c − b − d) ≥ 0, thenf (a, b, c, d) ≤ f (0, b, a + c, d); hence
f (a, b, c, d) ≤ max{f (a + c, b, 0, d), f (0, b, a + c, d)}.
Similarly, we now get
f (a + c, b, 0, d) ≤ max{f (a + c, b + d, 0, 0), f (a + c, 0, 0, d)} f (0, b, a + c, d) ≤ max{f
(0, b + d, a + c, 0), f (0, 0, a + c, b + d)}
Now since
max{f (a + c, b + d, 0, 0), f (a + c, 0, 0, d)} ≤ 6√ 3 max{f (0, b + d, a + c, 0), f (0, 0, a +
c, b + d)} ≤ 6√ 3, we conclude that f (a, b, c, d) ≤ max{f (a + c, b, 0, d), f (0, b, a + c,
d)} ≤ 6√ 3,
which proves our inequality.
Notice that the equality occurs if and only ifa = d = 0, b = 3 − √3, c = √ 3 (or, of
course, if any other cyclic permutation takes place).
Problem 94 For n ∈ N, n ≥ 2, determine
n n
max (1 − xi ), for xi ∈ R+ , 1 ≤ i ≤ n, x2 = 1.i
i=1 i=1
(Romanian IMO Team Selection Tests 2007)
Proof. (by Dan Schwarz) Let us analyzeE(x, y) = (1−x)(1−y),x, y ≥ 0,x2 +y2 = k2
, 0 < k ≤ 1.
Takex = k sin θ,y = k cos θ,θ ∈ 0, π . Now,2
x + y = k(sin θ + cos θ) = k√ 2 sin θ +π = k√ 2 cos θ −π ,4 4
and
xy = k2 sin θ cos θ = k 2 k2 π 2 sin 2θ = 2 cos 2θ −2
= k 2 π ,2 cos 2 θ −4
and hence,2 xy = k2 cos2 θ −π −k .4 2
Takeu = cos θ − π , sou ∈1 , 1 , and then4 √ 2
E
(
x, y
) =
E

105
(
u
) =
k
2 z2 − k√ 2z + 1 −k2 .
2
Its minimum value is reached for
u
0
=
1
k

2

1
√ , and therefore2

for
1
<
1 1 √
2 − 1 , the maximum value forE(u)k √ 2 +k √ 2 −1 = 2−
√ k , i.e.k < 22 k √ 2
is reached foru = √ 1 , i.e. whenθ ∈ {0, π}, which meansx ory being zero;

for
k

2

2
2 − 1 , the maximum value forE(u) is reached foru = 1, i.e. whenθ = π ,4 which meansx
= y.
Now, forx2 + y2 + z2 = 1, there will be two, for which we have that
2 < 2 √ 2 − 12 x2 + y2 ≤
3
(we have assumed them to bex andy), so first case applies forx2 + y2 = k2 ,k < 2 √ 2 −
1 (casek = 0 is trivial), therefore the maximum value is reached when one of the three
variables is zero.
So, whenn n
x2 = 1, E = (1 − xi )i
i=1 i=1
takes maximum value when all but two variables (be them x, y) are zero; thenE =
E(x, y) = (1 − x)(1 − y), withx2 + y2 = 1, and second case applies, yielding max E = 1
− √ 1 2 1 .2 , forx = y = √ 2

106
Remarks. The result forn = 2,x2 +y2 = k2 , may be obtained through other
methods, e.g. by partial differentiation ofE(x, y) = (1 − x)(1 − y). Also, an alternative
solution using Lagrange multipliers is available, but must be used with utmost care, as
the maximum is to be obtained on the boundary.
Problem 95 For integers n ≥ 2 and real s > 0, show that
n ☐n 1 ☐n
(s + i) 1 .☐s + j ☐< (n + 1) s + k −2i=0 j=0 k=1
(Robert E. Shafer, Amer. Math. Monthly )
Proof. First we prove by induction the inequality
n n
(s + i) < s +n · s + k − 1 .
i=0 2 k=1 2
Forn = 1, this iss(s + 1) < s2 + s + 1/4 = (s + 1 )2 . For the induction step, it suffices2
to show
(s + n/2)(s + n + 1) < s +n + 1 s + n + 1 ,2 2
which reduces ton + 1 > n.2
Now we prove the original inequality by induction. Forn = 2, the inequality becomes
s(s + 1)(s + 2)1 +s + 1 +
1 1
2
s + 2 = 3s + 6s + 2s
< 3s2 + 6s + 9
4
= 3 s + 1 s + 3 .
2 2
Now assume thatn ≥ 2 and that the original inequality holds forn. By the induction
hypothesis and
n n
(s + i) < s +n · s + k − 1 ,
i=0 2 k=1 2
we get
n+1 ☐ n+1 1 ☐ n+1 ☐ n 1 ☐ n
(s + i) ☐s + j ☐ = (s + i) ☐s + j ☐ + (s + i)
i=0 j=0 i=0 j=0 i=0
n 1 < (s + n + 1)(n + 1) s + k −
2k=1
n n 1 + s + s + k −
2
2
k=1
n+1
= (n + 2) s + k − 1 .
k=1 2
This completes our proof.

107
Problem 96 Let a, b, c be real numbers contained in the interval 0, 3 and also
satisfying 5
the condition a + b + c = 1. Determine the maximum value that the following
expression can reach:
P (a, b, c) = a3 + b3 + c3 + 3abc.
4
(Vo Quoc Ba Can, after Chinese Northern MO 2007)
Proof. Without loss of generality, we may assume thata ≥ b ≥ c, and therefore
☐ 3 ≥ a ≥ 1 ☐
5 3☐ 1 ≥ b ≥ 1 .
☐2 5
☐1 ≥ c ≥ 03
We furthermore have that
P = a3 + b3 + c3 + 3abc
4
= (a + b + c)3 − 3(a + b + c)(ab + bc + ca) + 15abc
4
= 1 − 3(ab + bc + ca) + 15abc
4
= 1 − 3c(a + b) + 3ab5 c − 14
= 1 − 3c + 3c2 + 3ab5 c − 1 ,4
moment in which we split our problem in two different cases: Case 1. If 1 ≥ a, then 1 ≥
b, and so we will show that2 2
P (a, b, c) ≤ 1 ,4
or in other words,
ab(4 − 5c) − (1 − 2c)2 ≥ 0. Indeed, we have a − 1 b − 1 ≥ 0,2 2
and hereby,
ab ≥ 1 1 4 = 14(1 − 2c),2(a + b) −
hence
ab
(4

5
c
)

(1

2
c
)
2

108
1
≥ 4 (1 − 2c) (4 − 5c) − (1 − 2c)2 = 3c(1 − 2c) ≥ 0.
4
Case 2. If 3 ≥ a ≥ 1 , then we will show that5 2
P (a, b, c) ≤7 .25
Indeed, since the expression is symmetric with respect toa, b, c,, using the same idea
as above, we write
P (a, b, c) = 1 − 3a + 3a2 + 3bc5 a − 1 ,4
and thus our inequality is equivalent to
1 − 3a + 3a2 + 3bc5 a − 1 ≤7 ,4 25
which can be rewritten into
3
3
bc(4 − 5a) + 25(3 − 5a)(5a − 2) ≥ 0,4
which we see that it is true, because 3 ≥ a ≥ 1 .5 2
Note that from the two cases we’ve studied, we can now conclude that
P (a, b, c) ≤7 ,25
and moreover, since we have equality for
(a, b, c) =3 , 2 , 0 ,5 5
this means that max P (a, b, c) =7 .25 Remarks. In the same fashion, one can prove that
a3 + b3 + c3 + 4abc ≤9 ,32
for all real numbersa, b, c contained in the inverval 0,1 , and satisfyinga + b + c = 1.2
Problem 97 Let a, b, c, d be positive real numbers such that a ≥ b ≥ c ≥ d and abcd = 1.
Prove that
1 1 1 ≥ 3 .
a3 + 1 +b3 + 1 +c3 + 1 abc + 1
(MathLinks Contest 2008)
Proof. Firstly, note that ifx,y are positive real numbers satisfyingxy ≥ 1, then
2
(
xy
− 1)
≥ 0, + 1 +
x2
1 12 (x − y)

y2 + 1 − xy + 1 = (xy + 1)(x2 + 1)(y2 + 1) and thus1 1 ≥ xy + 1.


≥ √
x
2
+ 1 +
2 y2 + 1
Now, using it fort2 = ab abcd = 1, we get

109
1 1 ≥
t3 + 1 , a3 + 1 +2
b3 + 1
hence it suffices to prove that
2 1 ≥ , 3
t2c + 1 t3 + 1 +
c3 + 1
or in other words,
(t3 c + 2t2 c2 − 2t − c)(t − c)2 ≥ 0. For this, we proceed as follows:
t3 c + 2t2 c2 − 2t − c = t3 c + 2t2 c2 − (2t + c)(t2 cd)3/4
≥ t3 c + 2t2 c2 − (2t + c)(t2 c2 )3/4
= t3 c + 2t2 c2 − tc(2t + c)√ tc ≥ t3 c + 2t2 c2 − 1 tc(t + c)(2t + c)2
=1 tc2 (t − c) ≥ 0.2
Notice that the equality occurs if and only ifa = b = c = d = 1.
Problem 98 Let a, b, c be nonnegative real numbers, from which at least are nonzero.
Prove that
a2 (b + c)2 b2 (c + a)2 c2 (a + b)2
b2 + c2 +c2 + a2 +a2 + b2 ≥ 2(ab + bc + ca).
(Vo Quoc Ba Can, Cosmin Pohoat¸˘a)
Proof. Notice that the inequality is equivalent to
a
2
− 2 ab + 2abca
b
2 + c2 ≥ 0.
cyc cyc cyc
By the Cauchy Schwarz Inequality , we get
2
a
a ≥cyc

a
(
b
2 + c2 ),

cyc
b2 + c2
cyc
and thus, the problem reduces to proving that
2
2abc a
a
2 − 2 ab +cyc

a
(

110
b
2 + c2 ) ≥ 0.

cyc cyc cyc


Since the inequality is homogenous, we can assume now thata+b+c = 1, and
furthermore putx = ab + bc + ca. In this case, the to prove inequality becomes
1 − 4x +2abc ≥ 0.x − 3abc
On the other hand, the Schur Inequality , written for the fourth degree, gives us
a2 (a − b)(a − c) + b2 (b − c)(b − a) + c2 (c − a)(c − b) ≥ 0,
or equivalently (in terms ofx), abc ≥ (4x − 1)(1 − x).6
Therefore, it follows that
1

4
x
+
2 (4x−1)(1−x)
x − 3abc ≥ 1 − 4x + x − 3 · (4x−1)(1−x) = (1 − 3x)(4x − 1) 2abc 2 ·
6
≥ 0, 6 3(1 − 3x + 4x2 )
which proves our inequality. The equality holds if and only if (a, b, c) ∈ {(t, t, t), (0, t,
t), (t, 0, t), (t, t, 0)}, wheret is an arbitrary positive real number.
Problem 99 Let a, b, c be nonnegative real numbers, from which at least two are
nonzero. Prove that
(
a
+
b
+
c
)
2 a(b + c) b(c + a) c(a + b) (a + b + c)2 ≥ a2 + bc b2 + ca c2 + ab ≥ a2 + b2 + c2. ab +
+ +
bc + ca
(Tran Quoc Anh)
Proof. Firstly, we will show that
2 a(b + c) b(c + a) c(a + b) (a + b + c)
+
ab + bc + ca ≥ a2 + bc + b2 + ca c2 + ab. Since
(a + b + c)2 3 = (a − b)(a − c) + (b − c)(b − a) + (c − a)(c − b),ab + bc + ca − ab + bc + ca
and
a(b + c) − 3 = − (a − b)(a − c)− (b − c)(b − a)− (c − a)(c − b) a2 + bc a2 + bc b2 + ca c2 +
ab ,cyc
we may write the inequality in question as
1 +1
a
2 + bc (a − b)(a − c) ≥ 0, cyc ab + bc + ca

111
or in other words,
(a + b)(a + c) + bc(a − b)(a − c) ≥ 0. cyc a2 + bc
This means that our problem reduces to showing that
(a2 − b2 )(a2 − c2 )+ abc (a − b)(a − c)≥ 0. cyc a2 + bccyc a3 + abc
For this, assume wihthout loss of generality thata ≥ b ≥ c. Then, the inequality from
above obviously follows from the following two other inequalities:
(a2 − b2 )(a2 − c2 )≥ (a2 − b2 )(a2 − c2 ) + (b2 − c2 )(b2 − a2 ) cyc a2 + bc a2 + bc b2 + ca
c
(
a

b
)
2 (a + b)(a2 + b2 + ab + bc + ca − c2 = ) ≥ 0, (a2 + bc)(b2 + ca)

(a − b)(a − c)≥ (b − c)(b − a) + (c − a)(c − b) a3 + abc b3 + abc c3 + abccyc


2
= (b − c) (b + c)(ab + ac − bc)≥ 0.bc(b2 + ca)(c2 + ab)
For the right hand side of the inequality, we proceed as in the beginning of the
previous proof. Since
(a + b + c)2 2(a − b)(a − c) + 2(b − c)(b − a) + 2(c − a)(c − b),a2 + b2 + c2 − 3 = − a2 + b2 +
c2
and
a(b + c) − 3 = − (a − b)(a − c)− (b − c)(b − a)− (c − a)(c − b) a2 + bc a2 + bc b2 + ca c2 +
ab ,cyc
the problem reduces to showing that
2 −1
a
2 2 2
2 + bc (a − b)(a − c) ≥ 0, cyc a + b + c
or equivalently, t = (a − b + c)(a + b − c)(a − b)(a − c)≥ 0, cyc a2 + bc
which is obviously valid, since
2
t
=
(
a

b
)(
a
+
b

c

112
)
(a − b + c)(a − c) + (b + c − a)(b − c) a2 + bc b2 + cacyc
=
(c2 + ab)(a − b)2 (a + b − c)2 (
a
2 + bc)(b2 + ca)≥ 0.
cyc
Problem 100 Let a, b, c be positive real numbers. Prove that a2 − bc + b2 − ca c2 − ab
4b2 + 4c2 + a2 + 4c2 + 4a2 + b2 ≥ 0. 4a2 + 4b2 + c2
(Vasile Cˆırtoaje, Math. Reflections)
Proof. The inequality is equivalent to
(2
b
+
c
)
2 (2c + a)2 (2a + b)2 3 ≥ 4a2 + 4b2 + c2 + 4b2 + 4c2 + a2 + 4c2 + 4a2 + b2. Now, from the

Cauchy-Schwarz Inequality , we have that (2b + c)2 (2b + c)2 4a2 + 4b2 + c2 = 2(a2 +
2b2 ) + 2a2 + c2 2
b
2 c 2
≤ a2 + 2b2 + c2 + 2a2. (2c + a)2 2c2 a2

4 b2 + 4c2 + a2 ≤ b2 + 2c2 + a2 + 2b2. (2a + b)2 2a2 b2


4c2 + 4a2 + b2 ≤ c2 + 2a2 + b2 + 2c2, and therefore
(2b + c)2 2b2 c2 2c2 a2 4
a
2 + 4b2 + c2 ≤ a2 + 2b2 + c2 + 2a2 + b2 + 2c2 + a2 + 2b2

cyc
2
a
2 b 2
+ 2
c + 2a2 + b2 + 2c2 = 3.
Remark. Generally, for any real numbersa,b,c and any positive realp, the following
inequality holds:
a2 − bc + b2 − ca c2 − ab 2pb2 + p2 c2 + a2 + 2pc2 + p2 a2 + b2 ≥ 0. 2pa2 + p2 b2 + c2
We invite the reader to find a simple proof. As a hint, we suggest starting with the
following identity:
a2 − bc
= (2pa2 + p2 b2 + c2 )2pa2 + p2b2 + c2
cyc cyc
1 (pb − c)2 pab + ca − b2 − pc2 2 + 3p(a2 − bc)2 .
2 cyc

113
Problem 101 Let a, b, c be nonnegative real numbers, from which at least two are
nonzero. Prove that
a(b + c)+ b(c + a) c(a + b) a(b + c) + b(c + a) + c(a + b)
b2 + c2 c2 + a2 + a2 + b2 ≥ a2 + bc b2 + ca c2 + ab. (Vo Quoc Ba Can, Cosmin
Pohoat¸˘a)
Proof. The inequality in question can easily be rewritten as 2 + bc − b2 − c2 a(b + c) a≥
2 2 2
0. cyc (a + bc)(b + c )
On the other hand, for all positive numbersa,b satisfyinga ≥ b, we have that
1
c
(
a

b
)
a

c
2 2 2
(a + bc)(b2 + c2 )
2
2 + b − c 2 + ab + c2 1 − (b2 + ca)(c2 + a2 )= (a2 + bc)(b2 + ca)(a2 + c2 )(b2 + c2 ) ≥ 0.
Therefore, under the assumption thata ≥ b ≥ c, we get
1 ≥ 1 1 (b2 + ca)(c2 + a2 ) ≥ (c2 + ab)(a2 + b2 ). (a2 + bc)(b2 + c2 )
Moreover, since
a(b + c)(a2 + bc − b2 − c2 ) = a(b + c) (a2 − b2 ) + c(b − c) ≥ 0
and c(a + b)(c2 + ab − a2 − b2 ) = c(a + b) c2 − b2 + a(b − a) ≤ 0, it follows that a(b + c)
a2 + bc − b2 − c2 a(b + c) a2 + bc − b2 − c2 (a2 + bc)(b2 + c2 )≥ (b2 + ca)(c2 + a2 )
and c(a + b)(c2 + ab − a2 − b2 ) c(a + b)(c2 + ab − a2 − b2 )
≥ .(c2 + ab)(a2 + b2) (b2 + ca)(c2 +
a2)
Hence
a
(
b
+
c
)
a
2
+ bc − b2 − c2 a(b + c) a2 + bc − b2 − c2 cyc = 0, cyc (a2 + bc)(b2 + c2 )≥ (b2 + ca)(c2 +
a2 )
which completes our proof.
Note that the equality occurs if and only ifa = b = c orabc = 0.
Problem 102 Let n be a positive integer. Find the minimum value of 2n+1 + (b − c)2n+1 +

114
(c − a)2n+1 (a − b)
(a − b)(b − c)(c − a) for distinct real numbers a, b, c with bc + ca ≥ 1 + ab + c2 .
(Michel Bataille, Math. Magazine)
Proof. (by Roberto Tauraso) We shall prove that the minimum is 4n − 1. Lettingx = a−
b, y = b − c,z = c − a, thenx = −y − z and the problem is equivalent to finding the
minimum value of ) = (y + z)2n+1 − z2n+1 − y2n+1

F (y, z yz(y + z)

fory = 0,z = 0,y + z = 0, andyz ≥ 1.


Sinceyz ≥ 1 andF (y, z) = F (−y, −z) we can assume thaty, z > 0. In this case, we
write2n−1
F (y, z) =2n − (−1)k y2n−k zk .
k=1 k
On the other hand, we have that
2n−1 2n 2n−1
− (−1)k = 4n − 1, and k ·2n − (−1)k = n(4n − 1),
k=1 k k=1 k
and therefore, according to the AM-GM Inequality ,
n
1/(4 −1)
2n−1 )((2n )−(−1)k) F (y, z) ≥ (4n − 1) (y2n−k zk k
k=1
n − 1) yn(4n−1) · zn(4n−1) 1/(4n−1) n − 1)(yz)n ≥ (4 = (4 ≥ 4n − 1 = F (1, 1),
where in the last step we have used the fact thatyz ≥ 1. This proves our problem.
Problem 103 Let a, b, c be positive real numbers. Prove that
ab+c bc+a ca+b 3.
(b + c)2 + (c + a)2 + (a + b)2 ≥ 4
(Vo Quoc Ba Can, Math. and Youth magazine)
Proof. Without loss of generality, we may assume thata ≥ b ≥ c. Then, we will
consider the following two cases:
Case 1. b + c ≥ 1; we can easily check that the following inequality holds for all
nonzero positive real numbersx:1 ≥ 1 .x2 x2x
Thus,
4a b+c 4b c+a 4c a+b a b+c b c+a c a+b
(
b
+
c
)
2 + (c + a)2 + (a + b)2 = b+c 2 + c+a 2 + a+b 2
2 2 2

b
+
c
b

115
+
c
+
bc+a ca+b ab+c
c+a c+a + a+b a+b 2 2 2
2
a
b+c 2b c+a 2c a+b
= b + c+ c + a+ a + b, and moreover sinceb + c = min {b + c, c + a, a + b} ≥ 1, by the

Bernoulli Inequality , we get


2
a
b
+
c
2
a
− 1 ≥ 1 + (b + c)b + c − 1 = 1 + 2a − b − c.b + c = 1 +
b+c 2a
b + c
Similarly, we have that
2b c+a 2c a+b
c + a ≥ 1 + 2b − c − a,a + b ≥ 1 + 2c − a − b,
and therefore,
b +c 4bc+a 4ca+b 2a b+c 2b c+a 2c a+b4a + (c + a)2 + (a + b)2 ≥ b + c+ c + a+ a + b≥ 3. (b +

c)2
Case 2. b + c < 1; we will split this case itself in two separate parts: Case 2.1. a ≥ 1;
then
ab+c b+c
≥ 1 > 3 .(b + c)2 ≥ a4
Case 2.2. a < 1; then
a
b+c = a ·1
2
a1−b−c ≥ a ≥ a b
c+a = b2 ·1

b2−c−a ≥ b2
c
a+b = c2 ·1

c2−a−b ≥ c2,
and therefore
ab+c bc+a ca+b a2 b2 c2 3. (b + c)2 + (c + a)2 + (a + b)2 ≥ (b + c)2 + (c + a)2 + (a + b)2 ≥ 4
This completes our proof.
Remarks. Note that for any integern and positive real numbersa1 , a2 , . . . , an , we

116
have that
a
1 2 n
s−a1 as−a2 as−an n , (s − a )2 + (s − a )2 + . . . + (s − a )2 ≥ (n − 1)2 wheres = a + a + . . .
1 2 n 1 2
+ an .
Problem 104 Find the least real number c such that if n ≥ 1 and a1 , . . . , an > 0 then
n k n
≤ c ak .k
k=1 j=1 1/aj k=1
(Joel Zinn, American Math. Monthly)
Proof. Putak = 1 for allk = 1, 2, . . . , n. In this case, the inequality becomesk
n n 1 n k n 1
k
k ≤ c k, or in other words,
k(k+1) ≤ c k.k=1 j=1j k=1 k=1 2 k=1
This rewrites as
n 1 2n 1
2
n 1
≤ c k, or equivalently,n + 1 − 2 ≤ (c − 2) k.k=1 k + 1 k=1 k=1
From this, we can deduce thatc ≥ 2. Indeed, if for example we havec < 2, then by
letting n near to infinity, we know that
n
lim
n 1
= ∞, and thereforen→∞ (c − 2)1 = −∞,n→∞k=1 k limkk=1
but, on the other hand
lim2 n 1 →∞ n + 1
2 − 2 = −2, and we thus have
n + 1 − 2 > (c − 2)
n k, k=1
which contradicts the fact that
n 1
n + 1
2 − 2 ≤ (c − 2) k.
k=1
Now, we will show that 2 is the value we are looking for, i.e. we are left to prove that
n k n
≤ 2 ak .k
1/a
k=1 j=1 j k=1
By the Cauchy-Schwarz Inequality ,
b
2 1 + 1 + · · · +1 ≥ (b + b + · · · + b )2 , a +b2
1 2 k 1 1
2 a
2 k a2 + · · · +b k a1 a2 ak
wherebi are arbitrary positive real numbers for alli = 1, 2, . . . , n. Therefore k≤ k

117
(
b
1
+
b
2
+
· · ·
+
b
k
)
2
· b2 a1 +b2 a2 + · · · +b2 ak ,k 1/a 1 2 k
j
j=1
from which we deduce that
n k n
≤ ci ai ,k
1/aj
k=1 j=1 i=1
where
kb
2 (k + 1)b2 nb2
k k k
c k = (b 2 + 2 + · · · + (b + b + · · · + b
1 + b2 + · · · + bk ) (b1 + b2 + · · · + bk+1 ) 1 2 k+1
2, 3 2 2
) for allk = 1, 2, . . . , n. Choosing nowbk = k, we get k k (k + 1) + · · · + k nck = k 2 +
k+1 2 n 2
i i i
i=1 i=1 i=1
= 4k2 1 11 + (k + 1)(k + 2)2 + · · · + n(n + 1)2 k(k + 1)2
=
4
k
2 1 1 − − · · · − · · · + 1 1
(k + 1)2 (n + 1)2k(k + 1) +
n(n + 1)
< 4k1 12 1 1 +(k + 1)2 + · · · + 11 +(n + 1)2 −2 k2 2 n2
− 11 − · · · −
(n + 1)2 (k + 1)2
= 4k2 1 1 1 11 + 2(n + 1)2 + (k + 1)2 + · · · + 1 − (k + 1)2 − · · · − (n + 1)2 2k2 n2

= 4k2 11 −2(n + 1)2 < 2,2k2


for allk = 1, 2, . . . , n. In this case, it follows that
n k n
≤ 2 ai ,k
k=1 j=1 1/aj i=1

118
and so our proof is completed. We conclude thatcmin = 2.
Problem 105 Let a, b, c be nonnegative real numbers satisfying a + b + c = 1, and
moreover from which at least two are nonzero. Prove that
a 4b2 + c2 + b 4c2 + a2 + c 4a2 + b2 ≤ 3 .4
(Vo Quoc Ba Can)
Proof. With the help of the Cauchy-Schwarz Inequality , we get
2 a(4b2 + c2) a 4b2 + c2 ≤ a(2b + c)
2b + ccyc cyc cyc
a
(4 b2 + c2 = 3 ab) , cyc cyc 2b + c
hence it suffices to prove that
3 ≥ a(4b2 + c2 )+ b(4c2 + a2 )+ c(4a2 + b2) 16(ab + bc + ca) 2b + c 2c + a 2a + b , or
equivalently,
3 ) + 4abc 1 1 1

2b + c + 2c + a + 2a + b≥ 3(ab + bc + ca) 16(ab + bc + ca


Using again the Cauchy-Schwarz Inequality we see that
1 + 1 1 3
2c + a + 2a + b ≥ a + b + c = 3, 2b + c
and so, we are left to show that
1 ) + 4abc ≥ ab + bc + ca.
16(ab + bc + ca
Settingx = ab + bc + ca, notice that 0 ≤ x ≤ 1 . From the Schur Inequality , applied
for3
the fourth degree, we have
a4 + b4 + c4 + abc(a + b + c) ≥ ab(a2 + b2 ) + bc(b2 + c2 ) + ca(c2 + a2 ),
and thus, in terms ofx, we get abc ≥ (4x − 1)(1 − x).6
Therefore
1 ) + 4abc − ab − bc − ca =1 − x + 4abc
16(ab + bc + ca 16x
≥1 − x + 23(4x − 1)(1 − x)16x
=
(3 − 8x)(1 − 4x)2 48x ≥ 0.
The equality holds if and only if (a, b, c) ∈ (1 ,1 , 0), (0,1 ,1 ), (1 , 0,1 ) .2 2 2 2 2 2
Remarks. Actually, the slighly more general statement:
√k2 + 1 ,k + 1 a k2 b2 + c2 + b k2 c2 + a2 + c k2 a2 + b2 ≤ max
3 4
holds for all positive reals a,b,c satisfying the conditiona + b + c = 1. However,
the proof is quite similar to the one we presented. We shall leave it (again) as an
exercise for the reader.
Problem 106 Let a, b be real numbers such that a + b = 0 and let x, y > 1 be some
given constants. Determine the minimum value of the following expression:
) = (
a
2 + 1)x (b2 + 1)y

119
f (a, b (a + b)2 .
Proof. According to the Weighted AM-GM Inequality , we have that
(
a
2 + 1)x = xx 1 x + y − 2 x1 · a2 + 1
x + y − 1 + 1 −
x x· (x + y − 1)(x − 1) x
x (x + y − 2)x−1
≥ 1 , (x + y − 1)x−1 (x − 1)x−1 a2 + x + y − 1
and similarly, we get
yy (x + y − 2)y−1 1(b2 + 1)y ≥ (x + y − 1)y−1 (y − 1)y−1 b2 + x + y − 1 .
We now set xx yy (x + y − 2)x+y−2 k = (x + y − 1)x+y−2 (x − 1)x−1 (y − 1)y−1,
and thus
(
a
2 + 1)x (b2 + 1)y ≥ k a2 +1 1 x + y − 1 x + y − 1 + b2 a
b
2
≥ k √ x + y − 1 +√ x + y − 1
x
x y
y (x + y − 2)x+y−2
= 2. (x + y − 1)x+y−1 (x − 1)x−1 (y − 1)y−1(a + b) This yields x
x yy (x + y − 2)x+y−2 f (a, b) ≥ (x + y − 1)x+y−1 (x − 1)x−1 (y − 1)y−1.
y−1

This lower bound is attained if and only if
☐ |a| = (x−1)(x+y−1)
☐ |b| = x−1
(
y

1)( x + y − 1)
.

☐ ab > 0
Problem 107 It is given that real numbers x1 , x2 , . . . , xn (n > 2) satisfy
n
xi > 1, |xi | ≤ 1 (i = 1, 2, . . . , n).
i=1
Prove that there exists a positive integer k such that k xi −n xi ≤ 1. i=1 i=k+1 (Chinese
Western MO, 2005)
Proof. Setg(0) = −n xi ,g(k) =k xi −n xi (1 ≤ k ≤ n − 1),g(n) =
n
i=1 i=1 i=k+1
xi . Theni=1

120
|g(1) − g(0)| = 2|x1 | ≤ 2, |g(k + 1) − g(k)| = 2|xk+1 | ≤ 2, k = 1, 2, . . . , n − 1, |g(n) − g(n
− 1)| = 2|xn | ≤ 2. So for each 0 ≤ k ≤ n − 1, |g(k + 1) − g(k)| ≤ 2.
If the conclusion is not true, by the condition for each k, 0 ≤ k ≤ n, we have |g(k)|
> 1. Now, if there is ani, 0 ≤ i ≤ n−1, such thatg(i)g(i+1) < 0, we may assume thatg(i) >
0 andg(i + 1) < 0. Henceg(i) > 1 andg(i + 1) < −1, and therefore|g(i + 1) − g(i)| > 2.
This contradicts the fact that |g(k + 1) − g(k)| ≤ 2, for eachk such that 0 ≤ k ≤ n − 1.
Thusg(0),g(1),. . .,g(n) have the same sign. Butg(0) + g(n) = 0. The contradiction
implies that the conclusion is true.
Problem 108 Let x, y, z be real numbers with sum 0, which are contained in the
interval [−1, 1]. Prove that
1 + x + y2 + 1 + y + z2 + 1 + z + x2 ≥ 3.
(Phan Thanh Nam)
First proof. Squaring the both sides, the inequality to prove remains
x2 + 2 (1 + x + y2 )(1 + y + z2 ) ≥ 6.
cyc cyc
From the Cauchy-Schwarz Inequality , we get
(1 + x + y2 )(1 + y + z2 ) ≥ (1 + x)(1 + y) + |yz|
cyc cyc cyc
≥ (1 + x)(1 + y) − yz,
cyc cyc
and thus we are left to show that
x2 − 2 xy + 2 (1 + x)(1 + y) ≥ 6.
cyc cyc cyc
Consider now the substitutionsa2 = 1 + x, b2 = 1 + y, c2 = 1 + z. Following the fact
thata2 + b2 + c2 = 3, the previous inequality becomes
a4 − 2 a2 b2 + 2 a2 + 2 ab − 9 ≥ 0,
cyc cyc cyc cyc
or in other words,
a4 − 2 a2 b2 + 2 ab − a2 ≥ 0.
cyc cyc cyc cyc
This can be rewritten into
3 a4 − 6 a2 b2 + 2 ab − a2 a2 ≥ 0,
cyc cyc cyc cyc cyc
or
a4 + a2 bc + ab(a2 + b2 ) − 4 a2 b2 ≥ 0,
cyc cyc cyc cyc
which is valid, according to the Schur Inequality applied for the fourth degree. Note
that the equality holds if and only ifx = y = z = 0.
Second proof. (by Dung Tran Nam) Firstly, see that ifa ≥ −1, b ≥ −1, a + b ≥ −1 and
ab ≥ 0, then √ 1 + a +√ 1 + b ≥ 1 +√ 1 + a + b.

Now, notice that there are at least two numbers amongx + y2 , y + z2 , z + x2 , which
have the same sign. Without loss of generality, assume that these two arex + y2 , y + z2
, and thus x + y2 y + z2 ≥ 0. Then, via the previous inequality, we get 1 + x + y2 + 1 +

121
y + z2 + 1 + z + x2 ≥ 1 + 1 + x + y + y2 + z2 + 1 + z + x2

1 + z 2 + x2 = 1 + z2 − z + 12 + y2 +

1 +
z
+ (
x + y )
2

1 +
z
2
2
− z + 1 +
=
1 +
z
2

z
+ 1 +

2
1 + z + z2 ,
and since
z2 − z + 1 +√ 1 + z2 + z2 ≥ 4, we conclude that
1 + x + y2 + 1 + y + z2 + 1 + z + x2 ≥ 3.
Problem 109 Let ai be nonzero positive real numbers, for all i = 1, . . . , n, satisfying
a1 + 1 + . . . +1 .1 + a2 + . . . + an =a1 a2 an
Prove that
1
+
. . .
+
1

1
.
n − 1 + a1
1 + n − 1 + a n − 1 + a
2 n
(Vasile Cˆırtoaje, Amer. Math. Monthly)
Proof. Notice that the inequality is equivalent to
n
n

122

1
≥ 1, or n − 1 + a
ii=1 n − 1 + a
n
1 −ai ≥ 1.
i i=1

Setbi =1 ; then the condition from the hypothesis remains invariant, but in terms ofbi
’sai
rewrites as
b1 + 1 + . . . + 1 .1 + b2 + . . . + bn =b1 b2 bn
On the other hand, the inequality to prove becomes now
n 1 ≥ 1. (n − 1)b + 1
i=1 i
We proceed now by making use of the contradiction method. In this case, assume that
there exist some numbersb1 , b2 , . . . , bn satisfying the above condition, but such that
n 1 < 1.

i=1 (n − 1)bi + 1
Then there also exists somek < 1 such that
n 1 + 1 = 1.

i=1 (n − 1)kbi
Furthermore, setting
x
i
=
1 1−x i

(n−1)kbi+1 < 1, i.e.bi =(n−1)kxi , we have thatx1 + x2 + . . . + xn = 1, and the given


condition becomes
n 1 − xn

i = (n − 1)kxi,
i=1 (n − 1)kxi i=1 1 − xi
and sincek < 1, we also have that
n 1 − xn

i < (n − 1)xi. i=1 (n − 1)xi i=1 1 − xi On the other hand, from the Cauchy-Schwarz
Inequality we get
n
1

xj n ☐ ☐n ☐ ☐ n xi ,
i
=1
x
i
i

123
=1
j
=
i
x
j
☐ ≥☐ xi · (n − 1)2j=i = ☐xi ☐ xj ☐ = (n − 1)2
i
=1
j=i i=1 j=i
xj
and therefore
n n
j=i ≥ (n − 1)2
x
j
n
x
i , or equivalently, (n − 1)x
i i=1 1 − xii=1 xi i=1
n
1 − xi ≥ (n − 1)xi.xj
j=i i=1
This obviously comes in contradiction with
n
1 − xn
i < (n − 1)xi, i=1 (n − 1)xi i=1 1 − xi completing then our proof.
Remarks. Similarly, we can show the following inequality:
For any positive real numbers a1 , a2 , . . . , an satisfying the condition
a1 + 1 + . . . +1 .1 + a2 + . . . + an =a1 a2 an
Then 1 + 1 1
n

1 +
a
2
+ . . . + n − 1 + a2 ≤ 1. n − 1 + a2

1 2 n
A possible proof for this variation can follow the same steps as in our previous
solution. However, we leave the reader the opportunity to deduce it.
Problem 110 Let a, b, c , d be real numbers contained in the interval (0, k]. Prove that
+ b4 + c4 + d4 (2k − a)4 + (2k − b)4 + (2k − c)4 + (2k − d)4 a≥ (2k − a)(2k − b)(2k − c)(2k − d)
.abcd
(Taiwanese MO 2002)
Proof. The inequality is equivalent to
(a2 − b2 )2 + (c2 − d2 )2 + 2(a2 b2 + c2 d2 )

124
abcd

(a − b)2 (4k − a − b)2 + (c − d)2 (4k − c − d)2 + 2 (2k − a)2 (2k − b)2 + (2k − c)2 (2k −
d)2 (2k − a)(2k − b)(2k − c)(2k − d). Hence, in order to prove this inequality, it suffices
to show the following three inequalities: (a2 − b2 )2 (a − b)2 (4k − a − b)2
abcd≥ (2k − a)(2k − b)(2k − c)(2k − d),
(c2 − d2 )2 (c − d)2 (4k − c − d)2 abcd≥ (2k − a)(2k − b)(2k − c)(2k − d),
and a2 b2 + c2 d2 (2k − a)2 (2k − b)2 + (2k − c)2 (2k − d)2 abcd≥ (2k − a)(2k − b)

(2k − c)(2k − d).


Without loss of generality, we may now assume thata ≥ b ≥ c ≥ d. The first one is
equivalent to
(2
k

c
)(2
k

d
)
2
k 2k 4k2 − 1 ,
cd a − 1b − 1 ≥a + b
and since (2k − c)(2k − d)
≥ 1,cd
it suffices to prove that
2
k 2k 4k2 − 1 . − 1 − 1 ≥
a b a + b
This is true, because by the AM-GM Inequality , we have 2k 2k = 2k(2k − a − b)+ 1 ≥
8k(2k − a − b)+ 1 = a + b − 1 .a − 1 b − 1 4k2 ab (a + b)2
Since the proof for (c2 − d2 )2 (c − d)2 (4k − c − d)2 abcd≥ (2k − a)(2k − b)(2k − c)(2k
− d) is similar to the previous one, we are left to show that
a2 b2 + c2 d2 (2k − a)2 (2k − b)2 + (2k − c)2 (2k − d)2 abcd≥ (2k − a)(2k − b)(2k − c)
(2k − d), or equivalently, f ab≥ f (2k − c)(2k − d) .cd (2k − a)(2k − b)
wheref is the real-valued function such thatf (x) = x + 1 . Now, sincef (x) is monotonix
cally increasing for allx ≥ 1, and moreover since
ab ≥ 1, (2k − c)(2k − d) ≥ 1,cd (2k − a)(2k − b)
we only need to show that
ab ≥ (2k − c)(2k − d), or a(2k − a) · b(2k − b) ≥ 1.cd (2k − a)(2k − b) c(2k − c) d(2k − d)
But this is obviously true, because
a(2k − a)− 1 = (a − c)(2k − a − c)≥ 0 and b(2k − b)− 1 = (b − d)(2k − b − d)≥ 0.c(2k − c) c(2k
− c) d(2k − d) d(2k − d)
Problem 111 Let a, b, c be positive real numbers. Prove that a + b c3

125
b + c + c + a ≥ √ 2 · ab + bc + ca. a + b a2 + b2 + c2 (Nguyen Van Thach)
Proof. Since 2a2 a2 + c2 a2 − c2 a2 + c2 a
+
c
= a + c+ a + c= a + c+ (a − c)
cyc cyc cyc cyc cyc
a
2 + c2
= c ,
a + cyc
and because
3
a
2 − a2a2 2 − aa2 + b2
a + c = 3 a a + b
cyc cyc cyc cyc cyc cyc
2 2
c(a + b )= a2 − a + bcyc cyc
ab
(
a

b
)
2
= (a + c)(b + c) ≥ 0,
cyc
it follows that3 a2
cyc ≥ 2a2

a c.
cyc cyc a + Combining this with the H¨older Inequality , we get
3 a2 2
cyc a a(a + b)(a + c)
a cyc a + b cyccyc
a
2 a2 ≥ 2 a(a + b)(a + c) a + c a + b
cyc cyc cyc
4
≥ 2 a ,
cyc
hence it suffices to prove that
4 2
4 a ≥ 27 ab a(a + b)(a + c) ,
cyc cyc cyc
or equivalently,
4 2
4 a ≥ 27 ab a2 a + 3abc .

126
cyc cyc cyc cyc
Now, assume (without loss of generality) thata + b + c = 1, and furthermore putx = ab
+ bc + ca. We now have that 1 ≥ x ≥ 0, and thus3
x2 − 3abc = (ab + bc + ca)2 − 3abc(a + b + c)
=
1 c2 (a − b)2 + a2 (b − c)2 + b2 (c − a)2 2
≥ 0.
Therefore,
4 − 27x(1 − 2x + 3abc) ≥ 4 − 27x(1 − 2x + x2 ) = (4 − 3x)(1 − 3x)2 ≥ 0,
which proves our inequality.
Problem 112 Let a, b, c be nonnegative real numbers, from which at least two are
nonzero.
a3 b3 c3 3(a2 + b2 + c2). a2 + b2 + b2 + c2 + c2 + a2 ≥ 2
(Vo Quoc Ba Can)
Proof. From the H¨older Inequality , we get
2 3a3 2 + b2 )2 (a2 + c2 )3 ≥ a2 (a2 + c2 )
cyc
a
2
+ b2 (a
cyc cyc 3
=1 (a2 + b2 )2 ,8 cyc
hence it suffices to prove that
3
1 (a2 + b2 )2 ≥ 3 a2 (a2 + b2 )2 (a2 + c2 )3 .
8 cyc 4 cyc cyc
Letting √ x = a2 + b2 , √ y = b2 + c2 , √ z = c2 + a2 the previous inequality can be
rewritten into
(x + y + z)3 ≥ 3 √ x +√ y +√ z xy√ x + yz√ y + zx√ z .
Now since the Cauchy-Schwarz Inequality gives us that
√ x +√ y +√ z ≤ 3(x + y + z),
we are left to show that
(x + y + z)5/2 ≥ 3√ 3 xy√ x + yz√ y + zx√ z .
Furthermore, notice that we can assume thatx + y + z = 3. The inequality now
becomes
xy√ x + yz√ y + zx√ z ≤ 3,
and by using (again) the Cauchy-Schwarz Inequality ,
2
xy√ x ≤ xy x2 y ,
cyc cyc cyc
from which we deduce that the problem reduces to
xy x2 y ≤ 9.
cyc cyc
This rewrites as xy x x2 y ≤ 27,

127
cyc cyc cyc
or equivalently,
☐2 ☐ xy ☐ x3 y + xy − 3xyz☐ ≤ 27
cyc cyc cyc
According now to the Cˆırtoaje Inequality , it is enough to prove that
2 2 ☐
xy
cyc
☐1 x2 + xy − 3xyz☐ ≤ 27,☐
3 cyc cyc
or equivalently, if we lett = xy + yz + zx,
t[(9 − 2t)2 + 3t2 − 9xyz] ≤ 81 i.e. t 7t2 − 36t + 81 − 9xyz ≤ 81.
Now, note that the Schur Inequality , applied for the third degree, gives us
3xyz ≥ 4t − 9,
hence
t 7t2 − 36t + 81 − 9xyz − 81 ≤ t 7t2 − 36t + 81 − 12t + 27 − 81 = t(7t2 − 48t +
108) − 81 = (t − 3) 7t2 − 27t + 27 ≤ 0.
This completes our proof.
Problem 113 Let a, b, c, d be nonnegative reals. Prove that
a k b k c k d k 1 , 4 , a + b + c+ b + c + d+ c + d + a+ d + a + b≥ min 1, 2k−1 3k for any
nonnegative real number k. (Vo Quoc Ba Can)
Proof. Ifk ≤ 1, then
a k a c> a + b + c + d = 1.
cyc
cyc
a
+
b
+
c

a a + b +cyc
If 2 ≥ k ≥ 1, then from the H¨older Inequality , we have
k k+1 ak 2k ,
cyc
(
a
+
b
+
c
)
k a(a + b + c) ≥ ak+1
cyc cyc
and therefore

128
k+1
a 2k k+1k+1 a 2k + b 2k + c 2k + d 2k a

k cyc k+1 k+1 k+1 k+1


= [(a + c)2 + (b + d)2 + (a + c)(b + d)]k

cyc
(a + b + c)k ≥ k
a(a + b + c)
cyc
Since in this case2k ≥ 1, according to the Power Mean Inequality , we getk+1
2k 2k
a 2k + c 2k ≥ 2 a + c b + d

k+1 k+1 k+1
k+1 2k
+ d 2k ≥ 22 ,2 , bk+1 k+1
thus
a
k
2
a
+
c
k+1
2k b+d 2k

k+1 + 2
2
(a + b + c)k ≥
k+1 2

[(
a
+
c
)
2 + (b + d)2 + (a + c)(b + d)]k

cyc
1
(
a
+
c
)
2k + (b + d) 2k k+1

k+1 k+1
= 2k−1 · [(a + c)2 + (b + d)2 + (a + c)(b + d)]k. Without loss of generality, we may now
assume thata + c ≤ b + d. Furthermore, set
1

t =a+c k+1 ≤ 1. Then,b+d


ak 1 · (t2k+2 + tk+1 + 1)k = 2k−1f (t),

129
cyc
(
a
+
b
+
c
)
k

(t2k + 1)k+1 1
2k−1
where
f
(
t
) =
(t 2k +1) k+1

(t2k+2+tk+1+1)k . Notice now that


k 2k k 2k k+1 + 2tk−1 − 1)
f (t) = kt (k + 1)(t + 1) (t − 2t ,(t2k+2 + tk+1 + 1)k+1
and thus
f (t) = 0 ⇔ t2k − 2tk+1 + 2tk−1 − 1 = 0 t
2k + 2tk−1
⇔ g(t) = − 1 − 2 = 0.tk+1
Now, see that
2k k−1 + k + 1 h(t)
g (t ) = (k − 1)t − 4t = ,tk+2 tk+2
whereh(t) = (k − 1)t2k − 4tk−1 + k + 1. In this case, we further have that
h (t) = 2(k − 1)tk−2 ktk+1 − 2 ≤ 0.
It follows now that h(t) is decreasing, and thusg (t) has at most one root contained
in the interval (0, 1]. The Rolle Theorem yields thatg(t) has at most two roots in the
same interval (0, 1]. Note that 1 is always a root ofg(t), and therefore
k+1
f (t) ≥ min {f (0), f (1)} = min 1, 2 .3k
This means thatak 1 , 4 ,
k ≥ min 2k−1 3k hence
cyc (a + b + c) a2 4.
2 ≥
cyc (a + b + c) 9 In the last case whenk ≥ 2, from the Power Mean Inequality , we
get
cyc a
k

a
2

130

k 2
k 2 (a+b+c) (a+b+c) 1,4 ≥ ☐☐cyc ☐☐ ≥ 3k4

and soak 4.
k ≥ k
cyc (a + b + c) 3 This completes our proof.
Problem 114 Let a, b, c be positive real numbers such that abc = 1. Prove that for all
(strictly) positive k’s we have
1 + 1 1
1 + b + ck + 1 + c + ak ≤ 1. 1 + a + bk
(Vasile Cˆırtoaje)
Proof. We will split the problem into two separate cases:
Case 1. Ifk ≥ 1, then by expanding, we see that the inequality in question is equivalent
with
ak+1 b + ak bk+1 ≥ ak + a.
cyc cyc cyc cyc
By the Weighted AM-GM Inequality , we get
k
a
k
+1
b
+
a
k
b
k
+1
=
a
k
+1
(
kb
+
c
k
)

(
k
+ 1)
a
k +1
b
κ k k+1 c k+1 cyc cyc cyc cyc

131
=
(
k
+ 1)
a
k
+
1

k+1
cyc
and
ak+1 b + k ak bk+1 = ak ab +k2 − 1 bk+1 +k2 + k + 1 k cyc cyc cyc 2k + 1 2k + 1 c
1 k

ab
·
b
(
k
+1)(
k
2 k+1

−1) · c k(k2+k+1)

≥ (k + 1) a2k+1 2k+1
cyc 1 k
ab
k
(
k
2+
k
+1)
k(k2+k+1) k+1
= (k + 1) a2k+1 c2k+1
cyc
=
(
k
+ 1)
a
k
+
k
+1

k(k2+k+1)
= (k + 1) ak2+k+11
(k+1)(2k+1) 2k+1 . cyc cyc

Summing up these two inequalities, it follows that


a
k
+1
b

132
+
a
k
b
k
+1

a
k 2+ 1 2k+1
k+1 + ak+k+1 .
cyc cyc cyc cyc
Moreover, since the Chebyshev Inequality gives us now that
3
a
k
+
1 1 1
k+1 ≥ ak ak+1 ≥ 3(abc)3(k+1) ak = 3 ak
cyc cyc cyc cyc cyc
and
3
a
1)
k 2+ k+1 = 3 a1+ k(k− k(k−1) 2k+1 2k+1 ≥ a a 2k+1 cyc cyc cyc cyc

3(
abc
)
k(k−1)
3(2k+1) a = 3 a,

cyc cyc
we conclude that
a
k 1

2k+1
2+k+1
+ ak+k+1 ≥ ak + a,
cyc cyc cyc cyc
which yields
ak+1 b + ak bk+1 ≥ ak + a.
cyc cyc cyc cyc
Case 2. If 0 < k ≤ 1, then 1 ≥ 1, and therefore, from Case 1, we have thatk
1+ 1 1
1 +
y
+
z

133
1 + 1 + z + x 1 ≤ 1 1 + x + y 1

k k k

for any positive realsx,y,z with product 1.


By letting nowx = ak , y = ck , z = bk , we obtain that 1 + 1 1 1 + b + ck + 1 + c + ak ≤ 1.
1 + a + bk
Notice that the equality occurs if and only ifa = b = c = 1.
Problem 115 Let a, b, c be nonnegative real numbers, from which at least two are
nonzero. Prove that

a + √ b c
5√
b + c + √c + a ≤ a + b + c.a + b 4
(Jack Garfunkel, Crux Mathematicorum)
First proof. By the Cauchy-Schwarz Inequality , we get
2 a√ a ≤ a(5a + b + 9c)
(a + b)(5a + b + 9c)cyc a + bcyc cyc
2
= 5 aa , cyc cyc (a + b)(5a + b + 9c) and we thus need to show that
a
a
(
a
+
b
)(5
a
+
b
+ 9
c
)
≤5 .
cyc cyc 16
Note that this is true, since
5 − a (a + b)(5a + b + 9c)=A + Ba

16 C ,cyc cyc
where
A = ab(a + b)(a + 9b)(a − 3b)2 ≥ 0
cyc
B = 243 a3 b2 c + 835 a2 b3 c + 232 a4 bc + 1230a2 b2 c2 ≥ 0
cyc cyc cyc
C = 16(a + b)(b + c)(c + a)(5a + b + 9c)(5b + c + 9a)(5c + a + 9b) > 0.
Second proof. From the same Cauchy-Schwarz Inequality , we have that 2 a(3a + 3b
+ c) a√ a≤ a + b cyc3a + 3b + ccyc a + b cyc

134
=
2
a
+
a(a + b + c) a a + bcyc 3a + 3b + ccyc cyc
a a= a 3a + 3b + ccyc a + b + 2 ,
cyc cyc
hence it suffices for us to prove that
a a + 2 ≤ 25 . 3a + 3b + c a + b 16
cyc cyc

Setting now A = a3 + b3 + c3 B = a2 b + b2 c + c2 a C = ab2 + bc2 + ca2


r = abc,
we get a = 3A + 19B + 15C + 36r 3a + 3b + c9 A + 39B + 39C + 82r.cyc
a + 2 = 4B + 3C + 5r a + b B + C + 2r ,cyc
and thus, the problem reduces to proving that
3A + 19B + 15C + 36r · 4B + 3C + 5r≤ 25,9A + 39B + 39C + 82r B + C + 2r 16
or in other words,
1220r2 + (210A + 176B + 1072C)r + 33AB + 81AC + 255C2 − 241B2 + 78BC ≥ 0.
Now, since AB = a3 a2 b = a5 b + a2 b4 + Br.
cyc cyc cyc cyc
AC = a3 ab2 = ab5 + a4 b2 + Cr.
cyc cyc cyc cyc
2
C2 = ab2 = a2 b4 + 2Br.
cyc cyc
2
B2 = a2 b = a4 b2 + 2Cr.
cyc cyc
BC = a2 b ab2 = a3 b3 + Ar + 3r2 ,
cyc cyc cyc
we have that
33AB + 81AC + 255C2 − 241B2 + 78BC
= 33 a5 b + 81 ab5 − 160 a4 b2 + 288 a2 b4 + 78 a3 b3
cyc cyc cyc cyc cyc
+(78A + 543B − 401C)r + 234r2
= ab(11a + 9b)(3a + b)(a − 3b)2 + (78A + 543B − 401C)r + 234r2 ,
cyc
and therefore the inequality to prove becomes
1454r2 + (288A + 719B + 671C)r + ab(11a + 9b)(3a + b)(a − 3b)2 ≥ 0,
cyc
which is obviouly nonnegative.
Problem 116 Let a, b, c, d be nonnegative real numbers, from which at least two are
nonzero, and which also satisfy the condition that a + b + c + d = 1. Show that

135
√ b c da + √ b + c + √ c + d + √ d + a ≤ 3. a + b 2
(Mircea Lascu)
Proof. Without loss of generality, we may assume thata+c ≥ b+d, and sox = a+c ≥ 1 .2
According to Problem 114 , we get

a + √ b c
5√ √
b + c + √c + a ≤ a + b + c = 5 1 − d,a + b 4 4 and therefore
√b ca + √b + c ≤ 5√ 1 − d − √c + a.a + b 4
Similarly, we obtain that

c
c
+
d
+

d ≤ 5√ 1 − b − √a d + a 4 a + c, hence

√a ≤5 √ 1 − b +√ 1 − d −√a + c ≤ 5 2(2 − b − d) −√c + a cyc a + b 4 4


=5 2(x + 1) −√x4
= (√x − 1) (17√x − 7) + 3 ≤ 3. 2√ 2 5√ x + 1 +√ 2 (2√ x + 3) 2 2
This completes our proof. Moreover, note that the equality does not occur.
Problem 117 Let a, b, c, d be positive real numbers, from which at least three are
nonzero. Prove that

a b c d
a + b + c + √ b + c + d + √ c + d + a + √ d + a + b ≤ 5√a + b + c + d. 4
(Vo Quoc Ba Can)
Proof. Without loss of generality, assume thatd = min{a, b, c, d}. If we letx = a + d,
then we have that

a + b + c x + a + √ d a d
d + a + b ≤ √ a + b + d + √ d + a + b = √ x b, and since

b + c + d
b ≤ √b b + c, the problem reduces to showing that

x + √b c
5√
b + c + √c + d + a ≤ a + b + c + d.x + b 4
But, notice that this is just Problem 114 applied to the variablesx,b andd. Note
that the equality occurs if and only if a = b = c = d or if any other cyclic permuta3 1 0 0
tion of this condition takes place.
Remarks. Generally (but apparently provable in the same manner), the following

136
inequality holdsx1 , x2 , . . . , xn

x1 + x2 +
x1 + √ x2
. . . + xn−1 x2 + x3 + . . . + xn
+ . . . + √xn ≤ 5√ x1 + x2 + . . . + xn .xn + x1 + . . . + xn−2 4
Problem 118 Let a1 , a2 , . . . , an be real numbers satisfying a2 + a2 + . . . + a2 = 1. 1 2 n
Determine the maximum value of the following expression:
min |ai − aj | .
i=j
(Nguyen Kim Cuong, Math. and Youth Magazine)
Proof. Assume (without loss of generality) thata1 ≤ a2 ≤ . . . ≤ an . Furthermore, set
n−1
a2 = a1 + s1 , a3 = a2 + s2 = a1 + s1 + s2 , . . . , an = a1 + si , wheresi ≥ 0, for all i = 1, 2, .
. . , n, and also puti=1
P = min |ai − aj | .
i=j
From the condition we have in the hypothesis, we get
n−1 2
1 = a2 + (a1 + s1 )2 + . . . + a1 + si ,1
i=1
and thus ☐ n−1 i ☐ n−1 ☐ i ☐2

na2 + 2☐ sj ☐a1 + ☐ sj ☐ − 1 = 0.1


i=1j=1 i=1 j=1
View the left hand since of the above equation as a quadratic polynomial. More
exactly,
take ☐ 2
n−1 i ☐n−1 ☐i ☐
f (x) = nx2 + 2☐ sj ☐x + ☐ sj ☐ − 1.
i=1 j=1 i=1 j=1
Since (the real number)a1 is a root for this polynomial, we deduce that its discriminant
must be nonnegative. That is,
☐n−1 i ☐2 ☐ n−1 ☐i ☐2 ☐ ∆f =☐ sj ☐ − n☐☐i=1 j=1 ☐☐ ≥ 0,☐ sj ☐ − 1
i=1j=1
which can be rewritten into
☐i ☐2 n−1 ☐i ☐2 ☐n−1 i ☐2 n−1
n − ☐ sj ☐ ≥ (n − 1) ☐ sj ☐ −☐ sj ☐ .
i=1 j=1 i=1 j=1 i=1 j=1
On the other hand, the Lagrange Identity tells us that
☐ i ☐2 ☐ n−1 i ☐2 n−1 j i 2 n−1
(n − 1) ☐ sj ☐ −☐ sj ☐ = sk − sk ,
i=1 j=1 i=1 j=1 1≤i<j≤n−1 k=1 k=1

137
and so, it follows that
n−1 ☐ i ☐2 ☐ n−1 i ☐2 n−1
(n − 1) ☐ sj ☐ −☐ sj ☐ ≥ P2 (i − j)2 ,
i=1 j=1 i=1j=1 1≤i<j≤n−1
or equivalently,
n−1 ☐ i ☐2 n−1
n − ☐ sj ☐ ≥ P2 (i − j)2 .
i=1 j=1 1≤i<j≤n−1
Moreover, because
n−1 ☐ i ☐2 n−1 n−1 n−1
☐ sj ☐ ≥ P2 i2 , we have that n − P2 i2 ≥ P2 (i − j)2 ,
i=1 j=1 i=1 i=1 1≤i<j≤n−1
and since n−1 = (n − 1)n(2n − 1)
i2
6
i=1
n−1 n−1 j n−1 j j
(i − j)2 = (i − j)2 = j3 − 2j i + i2
1≤i<j≤n−1 j=1 i=1 j=1 i=1 i=1
n−1
j(j + 1)(2j + 1)= j3 − j2(j + 1) + 6j=1
n−1
=1 2j3 − 3j2 + j6j=1
1 (n − 1)2 n2 (n − 1)n(2n − 1) + (n − 1)n = 6 2 − 2 2
n
(
n

2)(
n

1)
2
=12 ,
we can now conclude that n− (n − 1)n(2n − 1)P2 ≥ n(n − 2)(n − 1)2 12.6 12 P2, or in other
words,P ≤ n(n2 − 1)
As we can see, this upper bound is attained (for example) fors1 = s2 = . . . = sn = 12 ,a1
= −3(n−1) , and so the maximum value we are looking for is12
n(n 2 −1) . n(n 2 −1) n(n+1)
Problem 119 Let a2 , a3 , . . . , an be positive real numbers and s = a2 + a3 + . . . + an .
Show that n
a1−1/k √ s.k < s + 2
k=2
(George Tsintsifas, Amer. Math. Monthly )

138
Proof. In fact, we will show a slightly stronger statement:
For all positive real numbers b1 , b2 , . . . , bn , the following inequality holds:
n k n 2(π2 − 3)n b .
k k
k=1 k=1
bk+1 ≤ b k +9 k=1
We begin with a preliminary result:
Lemma . If
mk = 1, ifk = 1 , 2(k−1)k−1 ,ifk > 1
(k+1)k+1
thenk
bk+1 ≤ bk + mk bk .k
Ifk = 1, this is trivially true. Otherwise, ifk > 1, the inequality rewrites as
2
k 1 k−1 1
bk+1 2 .k 1 − bk+1 ≤ mk bk , or equivalently, bk+1 1 − bk+1 ≤ mkk k k
Indeed, ifbk > 1, this follows from the definition ofmk . Otherwise, ifbk is contained in
the interval [0, 1], we make use of the AM-GM Inequality as follows:
☐ bk+1 ☐k−1☐ 1 − bk+1☐2
2 1 1
k−1 1
b
k+1
1 − bk+1 = 4(k − 1)k−1 ☐k k
k − 1☐ ☐ 2☐k k

b k+1 + 2 1−b k+1 ☐k+11 1
(
k

1)
k k

☐☐☐☐☐
k−1 2

4(
k

1)
k−1
k + 1 ☐☐☐☐☐
=
4(k − 1)k−1
(k + 1)k+1 = m2,k
and so the Lemma is proven.
Returning to our problem, we see that it suffices to show that
n 2(π2 − 3)n
bk .

139
k=1
mk bk ≤9 k=1
But the Cauchy-Schwarz Inequality gives us that
n n n
mk bk ≤ m2 bk ,k
k=1 k=1 k=1
and so the problem actually reduces to proving
n
m2 1 n (k − 1)k−1 2(π2 − 3) , or in other words, 4 +k=2 (k + 1)k+1 ≤π2 1 .k ≤9 18 −6k=1
Now, because (
k
+ 1)
k+1
2 k + 1k−1 2k−1 = (k + 1)k − 1 = (k + 1)2 1 +k − 1 ,(k − 1)k−1
the Bernoulli Inequality yields
(k + 1)k+1 2 ,
(k − 1)k−1 ≥ 3(k + 1)
and thus
1 nn (k − 1)k−1 1 4 + 1 n+1 1 − 1 .4 + 1
k=2 (k + 1)2 = 1 k=2 (k + 1)k+1 ≤
3
k=1 k2 6
On the other hand,
lim
1 + 1
+ 1
+
. . .
+
1
=
ζ
(2) = π
2
,
2 2 2
n→∞ 2 3 n 6
and therefore it follows that
1 n (k − 1)k−1 π2 1, 4 + k+1 ≤ 18 − 6 which finishes our proof. Notice also that
k=2 (k + 1)
the equality does not occur.

Glossary
1. Abel’s Summation Formula
Ifa1 , a2 , ..., an , b1 , b2 , ..., bn are real or complex numbers, and
Si = a1 + a2 + ... + ai ,i = 1, 2, ..., n,
then n n−1
ai bi = Si (bi − bi+1 ) + Sn bn .

140
i=1 i=1
2. AM-GM (Arithmetic Mean-Geometric Mean) Inequality Ifa1 , a2 , ..., an are
nonnegative real numbers, then
n1 a ≥ (a a ...a ) 1 ,
i 1 2 n n n
i=1
with equality if and only ifa1 = a2 = ... = an . This inequality is a special case of the
Power Mean Inequality .
3. Arithmetic Mean-Harmonic Mean (AM-HM) Inequality Ifa1 , a2 , ..., an are
positive real numbers, then
1
ai ≥n
n 1
n i=1 1 1 n ai i=1
with equality if and only ifa1 = a2 = ... = an . This inequality is a special case of the
Power Mean Inequality .
153
4. Bernoulli’s Inequality
For any real numbersx > −1 anda > 1 we have (1 + x)a ≥ 1 + ax.
5. Cauchy-Schwarz’s Inequality
For any real numbersa1 , a2 , ..., an andb1 , b2 , ..., bn
(a2 + a2 + ... + a2 )(b2 + b2 + ... + b2 ) ≥1 2 n 1 2 n ≥ (a1 b1 + a2 b2 + ... + an bn )2 , with
equality if and only ifai andbi are proportional,i = 1, 2, ..., n.
6. Cauchy-Schwarz’s Inequality for integrals
Ifa, b are real numbers,a < b, andf, g : [a, b] → R are integrable functions, then
b 2 b b
f (x)g(x)dx ≤ f2 (x)dx · g2 (x)dx .
a a a
7. Chebyshev’s Inequality
Ifa1 ≤ a2 ≤ . . . ≤ an andb1 , b2 , . . . , bn are real numbers, then
n 1 n n
1)Ifb1 ≤ b2 ≤ . . . ≤ bn then ai · bi ;
i=1
aibi ≥ n i=1 i=1 n n n
1
2)Ifb1 ≥ b2 ≥ . . . ≥ bn then ai · bi ;
i=1
aibi ≤ n i=1 i=1

8. Chebyshev’s Inequality for integrals


Ifa, b are real numbers,a < b, andf, g : [a, b] → R are integrable functions and having
the same monotonicity, then
b b b
(b − a) f (x)g(x)dx ≥ f (x)dx · g(x)dx
a a a

141
and if one is increasing, while the other is decreasing the reversed inequality is true.
9. Cˆırtoaje’s Inequality
For any positive real numbersa,b,c, we have that
a2 + b2 + c2 2 ≥ 3 a3 b + b3 c + c3 a .
This inequality has been published in 1992 as Problem 22695 in the Romanian
(mathematical) journal Gazeta Matematic˘a. Notice that the following similar
inequality holds:
a 2 + b 2 + c 2 2 ≥ 3 ab 3 + bc 3 + ca 3 .
10. Convex function
A real-valued functionf defined on an intervalI of real numbers is convex if, for anyx,
y inI and any nonnegative numbersα, β with sum 1,
f (αx + βy) ≤ αf (x) + βf (y).
11. Convexity
A functionf (x) is concave up (down) on [a, b] ⊆ R iff (x) lies under (over) the line
connecting (a1 , f (a1 )) and (b1 , f (b1 )) for all
a ≤ a1 < x < b1 ≤ b.
A function g(x) is concave up (down) on the Euclidean plane if it is concave up
(down) on each line in the plane, where we identify the line naturally withR. Concave
up and down functions are also called convex and concave , respectively. Iff is
concave up on an interval [a, b] andλ1 , λ2 , ..., λn are nonnegative numbers wit sum
equal to 1, then
λ1 f (x1 ) + λ2 f (x2 ) + ... + λn f (xn ) ≥ f (λ1 x1 + λ2 x2 + ... + λn xn )
for anyx1 , x2 , ..., xn in the interval [a, b]. If the function is concave down, the
inequality is reversed. This is Jensen’s Inequality .
12. Cyclic Sum
Letn be a positive integer. Given a functionf ofn variables, define the cyclic sum of
variables (x1 , x2 , ..., xn ) as
f (x1 , x3 , ..., xn ) = f (x1 , x2 , ..., xn ) + f (x2 , x3 , ..., xn , x1 )
cyc
+... + f (xn , x1 , x2 , ..., xn−1 ).
13. H¨older’s Inequality
If r, s are positive real numbers such that 1 + 1 = 1, then for any positive real
numbersa1 , a2 , . . . , an ,r s
b1 , b2 , . . . , bn ,
n

n

1 ☐ n ☐1

r s

142
a
i
b
i

a
r bs i ☐ ☐ i ☐ i=1 ≤ ☐ i=1 ☐ ☐ i=1 ☐ .n ☐ n ☐ ☐ n ☐ ☐ ☐ ☐ ☐
☐ ☐ ☐ ☐
14. Huygens Inequality
Ifp1 , p2 , . . . , pn , a1 , a2 , . . . , an , b1 , b2 , . . . , bn are positive real numbers withp1 +
p2 + . . . + pn = 1, then
n n n
(ai + bi )pi ≥ api + bp .i i
i=1 i=1 i=1
15. MacLaurin’s Inequality
For any positive real numbersx1 , x2 , . . . , xn ,
S1 ≥ S2 ≥ . . . ≥ Sn ,
where xi1 · xi2 · . . . · xik
1≤i 1 <i 2 <...<i k ≤n .k = kS n
k
16. Lagrange’s Identity
For any two sets{a1 , a2 , . . . , an } and{b1 , b2 , . . . , bn } of real or complex number
(or more generally, elements of a commutative ring), we have that
2
n n n n−1 n
a2 b2 − ak bk = (ai bj − aj bi )k k
k=1 k=1 k=1 i=1 j=i+1
1
n n
=2 (ai bj − aj bi )2 .
i=1 j=1
17. Minkowski’s Inequality
For any real number r ≥ 1 and any positive real numbers a1 , a2 , . . . , an , b1 , b2 , . . . ,
bn ,
n
1 n 1 n 1 r r r

(ai + bi )r ≤ ar + br .i i
i=1 i=1 i=1
18. Power Mean Inequality
For any positive real numbersa1 , a2 , . . . , an with sum equal to 1, and any positive
real numbersx1 , x2 , . . . , xn , we defineMr = (a1 xr + a2 xr + . . . + an xr )1 ifr is1 2 n r a

143
non-zero real andMa xa . . . xan M∞ = max{x1 , x2 , . . . , xn },M−∞ =0 = x1 2 n ,
min{x1 , x2 , . . . , xn }. Then for any realss ≤ t we haveM−∞ ≤ Ms ≤ Mt ≤ M∞ .
19. Root Mean Square Inequality
Ifa1 , a2 , ..., an are nonnegative real numbers, then
a
2 + a2 + ... + a2

1 2 n a1 + a2 + ... + an,n ≥ n
with equality if and only ifa1 = a2 = ... = an .
20. Schur’s Inequality
For any positive real numbersx, y, z and anyr > 0,xr (x − y)(x − z) + yr (y − z)(y − x) +
zr (z − x)(z − y) ≥ 0. The most common case isr = 1, which has the following
equivalent forms:
1) x3 + y3 + z3 + 3xyz ≥ xy(x + y) + yz(y + z) + zx(z + x);
2) xyz ≥ (x + y − z)(y + z − z)(z + x − y);
3) if x + y + z = 1 then xy + yz + zx ≤ 1 + 9xyz ;4
4)
9xyz
2 + y2 + z2 .
x + y + z ≥ 2(xy + yz + zx) − x
Together with these equivalent forms, the Schur Inequality has lots of
generalizations and extensions, which we refer to as a same generalized Schur
Inequality . More exactly, for any realsa,b,c, letx,y,z be three nonnegative numbers.
Then, the inequality
x(a − b)(a − c) + y(b − c)(b − a) + z(c − a)(c − b) ≥ 0
holds if one of the following (sufficient) conditions is fullfilled:
a) We havea ≥ b ≥ c andx ≥ y.
b) We havea ≥ b ≥ c andz ≥ y.
c) We havea ≥ b ≥ c andx + z ≥ y.
d) The realsa,b,c are nonnegative and satisfya ≥ b ≥ c andax ≥ by. e) The realsa,b,c are
nonnegative and satisfya ≥ b ≥ c andcz ≥ by. f) The realsa,b,c are nonnegative and
satisfya ≥ b ≥ c andax + cz ≥ by. g) The realsx,y,z are the sidelengths of a triangle.
h) The realsx,y,z are the squares of the sidelengths of a triangle. i) The realsax,by,cz
are the sidelengths of a triangle.
j) The realsax,by,cz are the squares of the sidelengths of a triangle. k) There exists a
convex functiont : I → R+ , whereI is an interval containing
the realsa, b, c, such thatx = t(a), y = t(b), z = t(c) (here by R+ we have denoted the set
of all nonnegative real numbers.
21. Suranyi’s Inequality
For any nonnegative real numbersa1 , a2 , . . . , an ,
n n n n
(n − 1) an + n ak ≥ ak · an−1 .k k

144
k=1 k=1 k=1 k=1
22. Turkevici’s Inequality
For any positive real numbersx, y, z, t,
x4 + y4 + z4 + t4 + 2xyzt ≥ x2 y2 + y2 z2 + z2 t2 + t2 x2 + x2 z2 + y2 t2 .
23. Weighted AM-GM Inequality
For any nonnegative real numbersa1 , a2 , . . . , an , ifw1 , w2 , . . . , wn are nonnegative
real numbers (weights) with sum 1, then
w1 a1 + w2 a2 + ... + wn an ≥ aw aw . . . aw ,1 2 n with equality if and only ifa1 = a2 = ...
= an .

References
[ACDL] T. Andreescu, V. Cˆırtoaje, G. Dospinescu, M. Lascu, Old and New
Inequalities , Vol. 1, GIL, 2004.
[BB] E. Beckenbach, R. Bellman, Inequalities , Springer Verlag, 1961.
[BCLP] I. Boreico, V. Q. B. Can, M. Lascu, C. Pohoat¸˘a, Introduction to Inequalities ,
GIL, 2008.
[BDJMV] O. Bottema, R. Z. Djordjevic, R. R. Janic, D. S. Mitrinovic, P. M. Vasic,
Geometric Inequalities , Wolters-Noordhoff Publ., Groningen 1968.
[Ci] V. Cˆırtoaje, Algebraic Inequalities: Old and New Methods , GIL, 2006.
[Hun] P. K. Hung, Secrets in Inequalities , Vol. 1: basic inequalities, GIL, 2007.
[Hun1] —–, Secrets in Inequalities , Vol. 2: advanced inequalities, GIL, 2008.
[Kaz] N. D. Kazarinoff, Geometric Inequalities , Random House, New York, 1961.
[Liu] A. Liu, Hungarian Problem Book III , New Mathematical Library, Vol. 42,
MAA, 2001.
[Loz] E. Lozansky, C. Rousseau, Winning Solutions , Springer, 1996.
[MBMR] D. S. Mitrinovic, E. S. Barnes, D. C. B. Marsh, J. R. M. Radok, Elementary
Inequalities , Wolters-Noordhoff Publ., Groningen 1964.
[PBL] L. Panaitopol, V. B˘andil˘a, M. Lascu, Inegalit˘at¸i , GIL, 1995.
[Ste] J. M. Steele, Cauchy-Schwarz Master Class , Cambridge University Press ,
2004.
159

145
Table of Contents
Content 2

146

You might also like